Vous êtes sur la page 1sur 113

To Our Readers

Dear Readers,
It gives us great pleasure and satisfaction to present to you the August issue of
your favourite and frontline magazine ‘Competition Science Vision’. It is generally
claimed by the toppers and high ranking successful candidates of all pre-medical
examinations that each issue of this magazine is highly useful, unique and unbeaten
in matter of its contents and way of presentation. Traditionally, we always try to
improve the extent and quality of the subject matter and make it more and more
examination-oriented keeping in view the changes introduced in the examination
pattern.
CSV meets fully your requirements in all the four subjects. It has been marked
as second to none in its field by its readers. It covers all pre-medical tests held
throughout the country at present.
Hardwork under proper guidance, constant practice and revision have been
widely claimed by successful candidates as the core elements of their success. In
matters of guidance CSV stands matchless in the worthy estimation of our wise
readers.
Read CSV regularly and intelligently. It gives you the power to master
your career and shape your destiny.
With best wishes for your all-round success.
Sincerely yours,
Mahendra Jain
(Editor)

FORTHCOMING COMPETITIVE EXAMS.


2009
Chhattisgarh Sashastra Bal/Bharat Rakshit Vahini Constable Jharkhand High School Teachers Exam. (Aug. 9)
Recruitment Test (General Duty) (July 10) Bank of India Clerk Exam. (Aug. 9)
Haryana VLD Diploma Entrace Exam. (July 11) Central Bank of India Probationary Officer Examination (Aug. 16)
Gurgaon Gramin Bank Officers (Scale-I) Exam. (July 12) IGNOU B.Ed. Entrance Exam., 2009 (Aug. 16)
(Closing Date : 17 July, 2009)
Jawahar Navodaya Vidyalaya Entrance Exam., 2009 (Class IX) (July 12)
Delhi Fire Service, Fire Operator Exam. (Aug. 23 & 30)
Ordnance and Ordnance Equipment Factories Trade
PNB Management Trainee Exam. (Aug. 30)
Apprentices Training Selection Test (July 12)
UPSC National Defence Academy and Naval Academy
Madhya Pradesh Jail Department Guard Recruitment Exam. (July 12) Examination (II), 2009 (Aug. 30)
Rajasthan Sanskrit Education Department, Teachers Indian Air Force Airman Selection Test Group ‘Y’ Trade (Aug.)
Grade-III Exam. (July 22) SSC Central Police Organisation Sub-Inspectors Exam., 2009 (Sept. 6)
Rajasthan Sanskrit Education Department, Senior Teachers EPFO Social Security Assistant Exam., 2009 (Sept. 6)
Exam. (July 23-24) (Closing Date : 8 July, 2009)
Haryana School Teachers, Eligibility Test, 2009 for Elementary Combined Defence Services Examination (II), 2009 (Sept. 13)
Teachers (July 24) PNB Clerical Cadre Exam. (Sept. 20)
Haryana School Teachers, Eligibility Test, 2009 for Lecturers (July 25) Delhi SSSB Trained Graduate Teachers (English, Mathe-
R.A.S. (Mains) Examination (July 25–Aug. 12) matics, Natural Science) Exam. (Sept. 27)
UPSC Special Class Railway Apprentices Exam., 2009 (July 26) Bihar Telecom Technical Assistant Exam., 2008 (Oct. 4)
Haryana School Teachers, Eligibility Test, 2009 for (Closing Date : 20 July, 2009)
Master/Mistress (July 26) Delhi SSSB Trained Graduate Teacher (Social Science) Exam. (Oct. 10)
M.P. Guruji Eligibility Exam. (July 26) UPSC CPF Assistant Commandants Exam., 2009 (Oct. 11)
Chandigarh Police Constable (India Reserve Battalion) Delhi SSSB Trained Graduate Teachers (Sanskrit, Hindi, Urdu,
Recruitment Test (July 26) Punjabi) Exam. (Oct. 11)
S.S.C. Data Entry Operator Exam., 2009 (Aug. 2) Indian Economic Service/Indian Statistical Service
Uttarakhand B.T.C. Entrance Exam. (Aug. 2) Examination, 2009 (Nov. 21)
Madhya Pradesh Assistant Seed Certified Officer Selection Rajasthan State Eligibility Test (SET) (Nov. 22)
Test, 2009 (Aug. 2) (Closing Date : 31 July, 2009)

C.S.V. / August / 2009 / 666


There is always a section of examinee who is always ready to then sit for study for the period as
students which wants that the take the examination is apparently full fixed in your time table. This would do
examination dates be postponed for of self confidence and as such is you two benefits—there would remain
sometime so that they may make a always full of hope and optimism. We a continuity in your study and
thorough revision of the whole course believe that you also would like to secondly you will be free from the
or complete the chapters which have make the best of your time and com- tension which is the result of irregular
been left unfinished. Behind this plete your course within the definite study. One who makes regular
demand there is only one argument period of time and keep always studies keeps also some hours for
that they did not have enough time for ready to take the examination. extra study which makes him full of
proper preparations. On the other enthusiasm. Regular studies will do
side, there is another section of you one more good that you will get
examinees which is always ready to pleasure in doing constructive work
take their examination and wants and will never feel depressed.
that the examination may finish at
The continuous study creates
the earliest. It does not look nice that
boredom which needs to be removed.
the examinations may be postponed
For this the candidate must make a
once a date has been fixed for them.
change. For example, after reading
In our daily life also we find two for sometime if one sits to write some-
sections of people—one section of thing he will see that boredom is
the people seems to be having no released to a great extent. For
time for anything, while the other recreation it seems to be more proper
section of the people has time for that we develop the habit of reducing
to writing what we read. This makes
every useful thing. The people of this
what you read a part of your per-
section never complain of having no
sonality and you become exact in
time for anything. These people have your expression. As has been rightly
time to complete all assignments such said, “Reading makes a man perfect
as, complete their studies, attend and writing makes a man exact.”
games, go to important meetings and
We want to emphasize that if we
also join important functions of their
keep a constant eye on our time table
friends and relations etc. There are
we shall see that all our things are
some people in this section who have
done in their own way and without
to earn their living as well.
To make the best of time means resistance and then we will have no
As opposed to this the people of that we do not waste our time in complaint of having no time. If we
the section one have only one com- useless things and gossipping. Every work regularly and with full devotion
plaint to make that they had no time. examinee should first of all draw out we will always be ready to take the
They even make their business a his time table as to for how many
examination and will never think of its
cause of great excuse and make it hours he has to devote to his studies
postponment. This is a known fact
convenient to forget many things and also the timings of his studies.
He should be clear in his mind that that without proper tests no one gets
under their cover. If proper account is perfect knowledge. So you should
particular hours he must devote to his
kept of work, they have done during make up your mind to welcome
books and do nothing else during that
the twenty four hours, it will be seen examinations and as such be always
period. Such students may have to
that they have hardly devoted two to face criticism of many types such as ready for it. Also, do things syste-
three hours to serious work. It is no he does not talk to anybody, he tries matically. Hotch-potch working is the
surprise if such persons are not to show himself, a studious man, he result of hotch-potch of ideas. Syste-
successful in anything in their lives. is very proud etc. but he has to ignore matise your ideas so as to acquire a
A student may lag behind in his such things and follow his time table system in your working and then your
preparations on account of his limi- with full determination. Then and then
success is assured. That would give
tations or circumstances but in his alone will he be able to make himself
you confidence to go ahead and open
heart of hearts he also wants to be ready to take the examination.
Develop the habit of leaving your bed the gates for progress.
one of those who are always ready
to welcome the examinations. The in the morning at a particular time and ●●●

C.S.V. / August / 2009 / 667 / 1A


UGHTS FOR THE MON Subscription
T HO TH Rates
❥ An independent judiciary is vital to equal opportunity.
❥ Truth is above everything, but higher still, is the living of truth.
One Copy Rs. 35/-
❥ Everything appears coloured to the jaundiced eye.
One Year Rs. 335/-
❥ A vocal minority can create the impression of being the majority.
Two Years Rs. 630/-
❥ Our greatest evils flow from ourselves.
❒ Please send your Member-
❥ Genius is often perseverance in disguise.
ship fee by M.O. or Bank
❥ He that knows least commonly assumes most. Draft. Cheques will not be
❥ Handsome is that handsome does. accepted.

❥ A highbrow is a person educated beyond his intelligence. ❒ Please inform your


Correct Address together
❥ They never die who live for others. with the period, the month
❥ Men without the minimum restraint, dignity and integrity cannot be from which you are
architects of a new India. becoming a member.

❥ On the highest throne in the world, we still sit only on our own ❒ M.O. or Bank Draft will
bottom. be accepted only in the
name of M/s. Pratiyogita
❥ Fear of death is worse than death itself. Darpan, Agra.
❥ One crime is everything, two nothing.
❥ If you cannot be free, be as free as you can. ORDER FORM
❥ The devil hath powers to assume pleasing shapes. I want to be a regular
subscriber for Competition
❥ First deserve, then desire.
Science Vision. Kindly send
my copy on the following
address :
Name ........................................

Address.....................................
..................................................
..................................................
If you are appearing at any examination for admission to
.................................................
MBBS course of AFMC, AIIMS, CPMT, CBSE (Medical) and
any other Medical College/Institute you have a chance to win Pin
an attractive prize. I am sending the M.O./Bank

All that you are to do is to send us the list of multiple Draft for Rs. ...........................
choice objective questions which were asked in that examination Dated ......................................
on the basis of your memory. The questions should be complete Signature
with all the alternatives for answering them. We shall include . .................................................
all the entries in the contest. Participants with maximum number
of correct questions will be awarded. COMPETITION
SCIENCE VISION
M/s. Pratiyogita Darpan
PRIZES
2/11A, Swadeshi Bima Nagar,
(i) First three participants sending maximum number of correct Agra–282 002
questions will receive Rs. 200, 100 and 60 as prize. Phone : 4053333, 2530966, 2531101
(ii) No prize is admissible for less than 75% questions. Fax : (0562) 4031570, 4053330

C.S.V. / August / 2009 / 668


Climate Change—Imme- walking and cycling, it would tend to Europe, who inhabited the ancient
diate and Biggest Threat lower stress levels, reduce obesity, forests of the Carpathian Mountains
lessen heart and lung diseases and in what is the present day Romania,
to Health stroke risks. about 35,000 years ago.
Top experts of medical sciences The reconstruction of the face
Indian Space Shuttle will that could be male or female is based
of U.K. have published a report
be a Milestone on the skull and jaw-bone found in a
warning that climate change is the
biggest threat to global health of the cave where bears were known to
An Indian space shuttle will be hibernate.
21st century. Rising global tempera- test-fired from the spaceport at
tures would have a catastrophic effect Sriharikota within a year. Reusable
on human health and patterns of Launch Vehicle-Technology Demons-
infection would change with insect- trator (RLV-TD), as it is called, will be
borne diseases such as malaria and a rocket-aircraft combination : the
dengue fever spreading more easily. aircraft with a winged body, which is
the RLV, will sit vertically on the
rocket.
The engineering model of the
aircraft is ready at the Vikram
Sarabhai Space Centre (VSSC) in
Thiruvananthapuram. The first stage
of the Satellite Launch Vehicle-3 will
form the booster rocket. Weighing
nine tonnes, it is called S-9. After it Face of an ancient European
takes off like a rocket, the booster will The facial features indicate the
Insect Invasion : Patterns of infection release the unmanned aircraft, which close affinity of these early Europeans
would change, with insect-borne disea- will go into space. At the end of the to their immediate African ancestors,
ses such as malaria and dengue fever mission, the aircraft will land on the although it was not still possible to
spreading more easily. sea. determine the person’s sex.
The report says that the poorest Next year, the prototype of the Professor Richard Neave, who
people in the world will be worst RLV-TD will be ready for flight-testing.
affected. Although the carbon footprint reconstructed this face, based his
This will be a milestone for ISRO. The
of the poorest billion people is about 3 assessment on a careful and minute
RLV will open a new dimension in the
per cent of the world’s total footprint, launch vehicle technology and trans- measurement of the bone fragments
loss of life is expected to be 500 portation system of ISRO. Ground and his deep understanding of how
times greater in Africa than in the testing of the booster rocket is done the soft tissues of the face are built
wealthy countries. The impact of heat recently at Sriharikota. around the bones of the skull.
waves, flooding and food shortages The aircraft will stand over the The reconstruction is intended to
will be felt globally. rocket, nosetip up, and its tail will be record, how human origins and evolu-
Climate change is an immediate interfaced with the rocket. In other tion from our birth place in Africa to
danger. It is going to affect you and it words, the entire RLV will stand the long migratory routes, led us to
will inevitably affect your children. vertically on top of the booster.
populate even the most distant parts
The report says the evidence of The booster rocket will take the of the globe.
greenhouse gas emissions, tempera- RLV to a specific altitude, release the
ture and sea-level rises, the melting RLV and fall into the sea. On re-entry It is impossible from the bones to
of icesheets, ocean acidification and into the Earth’s atmosphere, the RLV determine the skin colour of the indi-
extreme climatic events suggest that will land in the sea, to be recovered. vidual, although scientists speculate it
the forecasts by the Intergovernmental was probably darker than modern day
panel on Climate Change might be First European Face Europeans, reflecting a more recent
too conservative. Unveiled African origin.
There is an awful lot we can do. Taken together, the material is
Reducing carbon emissions would A scientist of U.K. succeeded to the first that securely documents what
encourage people to cut use of reconstruct the face of the first modern humans looked like when
vehicles, and if that led to more anatomically modern human to live in they spread into Europe.

C.S.V. / August / 2009 / 669


Extended Sleep Improves The geology is complex, but it food and nutrition, not only helps
Performance boils down to this : Relieved of tackle obesity but cuts cancer risk
billions of tons of glacial weight, the also. Professor Soo-Yeun Lee from
Wondering how star athletes win land has risen much as a cushion. the University of Illinois has come up
world-level championships as recently The land is ascending so fast that the with a cinnamon-flavoured soy cereal
Roger Federer won the French Open rising seas—a ubiquitous byproduct that can cut the risk of prostate and
Tennis Championship. The secret of global warming—cannot keep breast cancer.
may lie in increased sleep. In accor- pace.
dance with a study, by the researchers As a result, the relative sea-level
Giant Space Tornadoes
of Stanford University in California, is falling, at a rate ‘among the highest Power the Northern
the athletes, who extended their night- ever recorded’, according to a report Lights
sleep, reported improvements in by a panel of experts.
various drills conducted after every Different cultures have attributed
regular practice. The research stated The Ape Laughs Like their spectacular light show to fire-
that many of the athletes, who parti- Humans breathing dragons, dancing gods and
cipated in this study, for the first time ghostly clans at war. Now research
realized the importance of sleep and What happens if you tickle a has found that Northern Lights or
how it affects their performance during gorilla ? The ape laughs like humans, aurora borealis, are powered by giant,
competitions. according to a research, which electrical tornadoes spinning at more
suggests that the origins of laughter than a million miles an hour and
While most athletes and coaching
can be traced back more than 16 stretching thousands of miles into
staff may believe that sleep is an
million years. Findings of a research space.
important contributing factor in sports
team of the University of Portsmouth Scientists used a set of five
and peak performance can only occur
(Britain) indicate that humans have satellites designed to measure the
when an athlete’s sleep and sleep Earth’s magnetic field to generate the
inherited ability to laugh from the last
habits are optimal. first images of the whirling vertices.
common ancestor from which they
Five healthy students between and great apes evolved. This investi- They show how vast quantities of
the age-group of 18 to 21 participated charged particles emitted by the Sun
gation is the first phylogenetic test of
in the study. A record of their sleep/ first pile up in huge clouds about
the evolutionary continuity of a human 40,000 miles above the night side of
wake pattern for a three-week-period emotional expression. It supports the
was recorded. Athletic’s performance, Earth. Then as the energy they hold
idea that there is laughter in apes. becomes too great, the particles
including sprinting and hitting drills,
explode downwards towards Earth,
was recorded after every practice. spinning as they go.
Athletes extended their sleep to 10
‘‘When these space tornadoes
hours a night for six weeks. Mood and
reach the upper atmosphere, their
daytime sleepiness were recorded.
enormous energy heats the air so
Furthermore, daily sleep/wake acti-
strongly that it starts glowing. That is
vities were monitored using sleep
what generates the aurorae’’, said
actigraphy.
Professor Karl Heinz Glassmeier
Results suggested that sleep (Germany), the leader of research
extension in athletes was associated team.
with a faster sprinting drill, increased Humour in Evolution : Origin of The Northern lights and their
hitting depth in drill and increased laughter can be traced back to more Southern equivalent, the aurora
hitting accuracy. According to the than 16m years. australis, create spectacular moving
research team the findings of this Professor Marina Davila Ross led displays of different shades and
study could be relevant in all walks of the research team. The experts who colours of light.
life. The results of this study were carried out the research on gorillas, ●●●
presented at the 23rd Annual Meeting chimpanzees and orangutans, believe
of Associated Professional Sleep that it started out as a ‘grunt like’ noise
Societies. with our distant ancestors and
gradually turned into the more sophi-
As Alaska Glaciers Melt, sticated chuckles and guffaws, we
Sea Level Falls know today. The research proves that
laughter evolved gradually over the
Global warming conjures images last 10 to 16 million years of primate
of rising seas that threaten coastal evolutionary history. Useful for Various Competitive Exams.
areas, but in Juneau, Alaska, as
almost nowhere else in the world, Super Cereal Fights By : Dr. Lal, Mishra & Kumar
climate change is having the opposite Obesity, Cancer Code No. 1624 Rs. 250/-
effect. As the glaciers melt there, the
A new study has shown that soy UPKAR PRAKASHAN, AGRA-2
land is rising, causing the sea to E-mail : publisher@upkar.in Website : www.upkar.in
retreat. cereal, developed by a professor of

C.S.V. / August / 2009 / 670


AWARDS/HONOURS Best Playback Singer (Female)
—Shreya Ghoshal—Teri Ore (Singh BOOKS
is King)
10th IIFA Awards, 2009 Twilight of the Tigers— G. H.
Outstanding Achievement by Peiris (The book deals with what
The 10th IIFA (International an Indian in International Cinema— would be the next for Tamils in Sri
Indian Film Academy) Awards cere- Aishwarya Rai Bachchan Lanka. The dismissal of devolution of
mony for 2009 was concluded in Lifetime Achievement Award— power as a solution has dark fore-
Macau, China. Major names from Rajesh Khanna boding. After three decades holding a
Bollywood transcended on this beauti-
Best Dialogue—Manu Rishi— gun to Sri Lanka’s head, the
ful island of China. The function took
Oye Lucky ! Lucky Oye ! Liberation Tigers of Tamil Eelam has
place in the Venetian Macao Resort
Best Screenplay—Neeraj finally been defeated militarily and its
Hotel, Macau. The event continued
Pandey—A Wednesday ! leader Velupillai Prabhakaran is dead.
for three days.
The decisive battles between the
Bollywood director Ashutosh Sound Editing—Ghajini
LTTE and the Sri Lankan military were
Govarikar’s historical love epic Jodhaa Sound Recording—Ghajini fought since January 2009 till the
Akbar swept the 10th IIFA awards, Best Action—Ghajini middle of May 2009. The simple point
taking home trophies for the best is that in Sri Lanka, Sinhalese are 73
Special Effects—Ghajini
picture, best director and best actor. per cent and Tamils are 12 per cent.
Best Sound Recording—Rock Sri Lankan Tamils are liberated from
Awardees are—
On !! the LTTE, but are afraid of not having
Best Film—Jodhaa Akbar
Best Editing—Jodhaa Akbar a strong voice to speak up for them.
Best Actor in a Leading Role
Best Make-up—Jodhaa Akbar The book deals with these facts).
(Male)—Hrithik Roshan—Jodhaa
Akbar Art Direction—Jodhaa Akbar World of Work— Pub. by Inter-
Best Costume Award—Jodhaa national Institute for Labour Studies,
Best Actor in a Leading Role
Akbar Geneva (The book gives the details
(Female)—Priyanka Chopra—
that globalisation has resulted in the
Fashion Best Background Score—A. R. widening of income–inequality across
Best Actor in Supporting Role Rahman and within the countries.)
(Male)—Arjun Rampal—Rock On Star of the Decade (Male)— India’s Energy Security—Ed. by
Best Actor in a Supporting Shahrukh Khan Ligia Noronha and Anant Sudarshan
Role (Female)—Kangana Ranaut— Star of the Decade (Female)— (The book draws the attention to the
Fashion Aishwarya Rai Bachchan need for formulating holistic policies
Best Actor in a Comic Role— Music Director of the Decade— on energy.)
Abhishek Bachchan—Dostana A. R. Rahman Democracy and Human Deve-
Best Actor in a Negative lopment in India— Naresh Gupta
Role—Akshaye Khanna—Race Man Booker International (The book explains how the demo-
Best Debutant Star (Male)— Award, 2009 cracy and human development are
Farhan Akhtar Canadian short story writer, Alice interrelated.)
Best Debutant Star (Female)— Munro, beat Mahasweta Devi and
Asin host of other literary heavyweights, DAYS
Best Director—Ashutosh Gowa- including Nobel Laureate V. S.
riker—Jodhaa Akbar Naipaul, to win the £ 60,000 Man July 1—Doctors’ Day
Booker International Prize. July 6—Zoonoses Day
Best Story—Neeraj Pandey—A
Wednesday Ms. Munro (78) is regarded as July 11—World Population Day
one of Canada’s most celebrated
Best Music—A. R. Rahman— writers. She was amazed and
Jodhaa Akbar delighted to win the prize.
Best Lyrics—Javed Akhtar— The prize, different from the Ustad Akbar Ali Khan—Sarod
Jashn-E-Bahara (Jodhaa Akbar) annual Booker Prize for Fiction, is maestro Ustad Akbar Ali Khan (88)
Best Playback Singer (Male)— awarded once every two years to a passed away in Francisco (U.S.A.) on
Javed Ali—Jashn-E-Bahara (Jodhaa living author for his or her lifetime June 19, 2009 after a prolonged
Akbar) achievements. kidney ailment. He is survived by his

C.S.V. / August / 2009 / 671 / 2


wife Mary, three sons and a daughter. Banerjee. Prior to being appointed the A. K. Kembhavi (New Director,
He was born in 1922 in Comilla Solicitor-General, he was Advocate- IUCAA )—Ajit Keshav Kembhavi has
district, now in Bangladesh. General of Maharashtra till June 2004. been appointed as the Director of
A recipient of Padma Bhushan Mr. Vahanvati is the first Muslim Inter-University Centre for Astronomy
and Padma Vibhushan, the Ustad to occupy the top law officer’s post in and Astrophysics (IUCAA), Pune.
was a colossus in the world of Indian the country in the last six decades. IUCAA is one of the five inter-
classical music for the last five Pradeep Vasant Naik (New university research centres of the
decades. He was admired by both Chief of the Air Staff)—Air Chief University Grants Commission. Prof.
eastern as well as western musicians Marshal Pradeep Vasant Naik (60) Khembhavi will take charge on Sep-
for his brilliant compositions and his took over as the 19th Chief of the tember 1, 2009 from Professor Naresh
mastery of the 25 string instrument. Indian Air Force succeeding Air Chief Dadhich.
He also composed music for Indian Marshal Fali Homi Major. With more Professor Kembhavi took his
films of Chetan Anand, Satyajit, Tapan than four decades of distinguished Ph.D. degree in 1979 under the
Sinha and others. guidance of Professor Jayant Narlikar.
services, Air Chief Marshal Naik took
His research interests included
Habib Tanvir—Noted playwright serious part in Indo-Pak war in 1971. quasars and other active galaxies,
Habib Tanvir passed away on June 8, He was honoured with the Vishisht intermediate red-shift galaxies, galaxy
2009. He was one of the greatest Seva Medal. He was commissioned morphology, tidal capture binary stars
stalwarts of the Indian stage, known into the IAF in 1969 as a fighter pilot. in globular clusters, astronomical data
for blending theatre, folk art and So far he has clocked more than bases and the virtual observatory
poetry in his works. He left an indelible 3000 hours of flying. (VO).
mark on the minds of the viewers. He P. K. Barbora ( New Vice-Chief P. Varghese (New Australian
was born on September 1, 1923 in of Air Staff )—Air Marshal P. K. Envoy)—Australia has named Peter
Raipur. He took M.A. degree from Varghese as the next High Commis-
Barbora took over as the new Vice-
sioner to India with concurrent acredi-
Aligarh Muslim University and joined Chief of the Air Staff. tation to Bhutan. He succeeds John
All India Radio, Bombay as Director Nirmal Kumar Verma (New McCarthy. Mr. Varghese will take
in 1945. His full name was Habib Navy Chief)—Vice-Admiral, Nirmal charge in August 2009. Mr. Varghese
Ahmed Khan and he had adopted the Kumar Verma, will be the next Chief served as the High Commissioner in
pen name ‘Tanvir’. He also wrote of the Naval Staff. He will take charge Malaysia and had been also a part of
songs for Hindi films. He won Sangeet from Admiral Sureesh Mehta who will Australian missions Vienna, Washing-
Natak Akademi award in 1969. He ton and Tokyo.
retire on August 31, 2009. Vice-
was honoured with Bhushan. He had Admiral Verma, currently Flag Officer
been a Member of Rajya Sabha Commanding-in-Chief, Eastern Naval
(1972–1978). Command, commanded aircraft carrier
Rajeev Motwani—Rajiv Motwani Meira Kumar—History was made
INS Viraat. He was decorated with the
(47) was talented mathematician on June 3, 2009 when diplomat-
Param Vishisht Seva Medal.
whose contribution to the world of turned-politician Meira Kumar became
R. Khullar (New Commerce the first woman
computer science influenced the
Sec.)—Union Government, on June 5,
development of algorithm-based Speaker of the Lok
2009, appointed Disinvestment
search technology. He passed away Sabha, with the
Secretary, Rahul Khullar as
in a freak drowning accident in his otherwise fractious
Commerce Secretary. He will replace
swimming pool on June 7, 2009. In House setting aside
G. K. Pillai a 1975 batch IAS Officer,
Atherton, California. He was its differences to
Mr. Khullar was shifted to the
professor at Stanford University. He elect the Dalit leader
Department of Disinvestment from
is survived by his wife Asha Jadeja
the Commerce Ministry. unanimously. The Meira Kumar
and the daughters Naitri and Anya.
Google’s founders Sergey Brin Father’s Name Late Shri Jagjivan Ram
and Laurence page, were mentored Mother’s Name Smt. Indrani Devi
by Dr. Motwani as he conducted Date of Birth 31.03.1945
research to launch what would Place of Birth Patna (Bihar)
become an iconic service. His contri- Marital Status Married
bution to frame the ‘world wide web Date of Marriage 29 Nov., 1968
(www)’ is incomparable. Spouse’s Name Shri Manjul Kumar
No. of Son 1
No. of Daughters 2
Educational Qualifications M.A., LL.B., Advanced Diploma in Spanish Educated
G. E. Vahanvati ( New Attorney at Indraprastha College and Miranda House, Delhi
Gen. )—Goolam E. Vahanvati (60) till (Delhi)
recently the Solicitor-General of India, Profession Social Worker, Advocate, Civil Servant
is appointed as the next Attorney- Permanent Address D-1029, New Friends Colony, New Delhi–110 065
General of India for a period of three (011) 26910618, 26910639, 9810630165 (M)
years. He will succeed Milon K. Fax—91-11-26910618

C.S.V. / August / 2009 / 672


name of the 64-year-old Congress
leader from Bihar was proposed by Iran’s Political Turmoil
party President Sonia Gandhi and
seconded by the Prime Minister
Manmohan Singh and BJP leader
L. K. Advani together led her to the
podium and later paid tribunes to a
woman.
This is Ms. Meira Kumar’s fifth
term in Lok Sabha. At present she
represents Sasaram (Bihar).
Karia Munda—Seven-time
Member of Parliament, Karia Munda,
was unanimously elected Deputy
Speaker of the Lok Sabha. A dozen of
nominations were
filed in Mr. Munda’s
favour. The motion
was first moved by
the opposition
leader L. K. Advani
and seconded by
BJP President Karia Munda
Rajnath Singh. Subsequently, leaders
representing other parties, including The purported victor’s triumpha- personnel and CPI(M) workers and
Congress, moved similar motions. A lism at a huge victory rally was sympathisers. With tribal folk as
32-year-old unbroken tradition of immediately challenged on the streets human shield, they have now shought
having the Deputy Speaker from the by close to a million supporters of Mir to create ‘liberated zones’ in the
opposition—which began in 1977, the Hosain Mausavi, the principal district. The offensive, time to take
very year Mr. Munda entered the Lok opponent. They braved a ban order advantage of the electoral debacle of
Sabha—has been preserved with the and choked a nine-kilometre stretch CPI(M) in the recently concluded Lok
unanimous election of Mr. Munda. leading to Azadi (freedom) Square, a Sabha election, would not have been
This sentiment was echoed by the prominent Tehran landmark. The possible without the support of the
entire house. Mr. Munda is elected to unrest has now spread to other cities, main opposition party, the Trinamool
this 15th Lok Sabha from Khunti in including Tabriz, Shiraj and others. Congress. Either directly or indirectly,
Jharkhand on BJP ticket. the Police Santrosh Birodhi
Only a fair, free and credibly Janashadharaner Committee or the
Mr. Karia Munda was born on monitored fresh election can heal a People’s Committee against police
April 20, 1936 in Ranchi district of nation that is treading a thin line bet- atrocities, which spread heads the
Jharkhand. He took his M.A. degree ween fear of theocratic authoritarian- agitation, has drawn substance from
from Ranchi University. He is a senior ism and hope of genuine reform that the opportunism of the Trinamool
leader of Bhartiya Janta Party and is not in conflict with revolution’s Congress and its partner the
had been a Cabinet Minister in fundamentals. Congress. Union Government is
Vajpayee’s Government. Lalgarh (West Bengal)—Social reluctant to extend full assistance to
unrest is known to manifest itself as West Bengal Government.
problems of law and order. But the The situation in West Midnapore
reverse can also be true sometimes, is too serious to allow such crass
‘IRAN’ on the Boil—Thirty years politically inspired violence seeks the politicking. Manmohan Singh Govern-
after momentous events brought cover of socio-economic grievance. ment must not lose any more time in
Ayatollah Ruhollah Khomeini to the Maoist, outlaws who went on the coming to the aid of West Bengal’s
fore as Iran’s man of destiny, his rampage in the Lalgarh of West Left Front Government to tackling the
Islamic revolution has skidded into Midnapore district of West Bengal, Maoists and their surrogates. Else
uncharted territory. The official carrying out murderous attacks on the there will be a heavier cost to pay.
ruling Communist Party of India
declaration of incumbent President
(Marxist) workers and destroying their SPORTS
Mohamoud Ahmadinejad as a run
houses and party offices, were looking
away winner in a hotly disputed and to settle political scores in the guise
possibly rigged presidential election of protesting ‘police atrocities’. Using Badminton
has generated a white heat of popular the neighbouring State of Jharkhand Indonesian Open—Ace Indian
anger that has no precedent in Iran’s as the base, they established reign shuttler, Saina Nehwal, on June 21,
post-revolution history. of terror and drove out security 2009 in Jakarta, scripted history by

C.S.V. / August / 2009 / 673


becoming the first Indian to win a Federer achieved this victory session of Parliament on June 4,
Super Series tournament after she without his having to combat the 2009, the President Pratibha Patil
clinched the Indonesian Open title formidable rivals such as Rafael unveiled the new Union Government’s
with a stunning victory over the higher Nadal, Andy Murrray and Novak agenda—
rankeded Chinese Lin Wang. Saina Djokovic, all of whom suffered shock
beat Wang 12-21, 21-18, 21-9 in a defeats. Manmohan’s Agenda
thriller that lasted for 49 minutes. ● Internal security and preservation of
French Open 2009 communal harmony.
Men’s Singles—Switzerland’s ● Economic growth in agriculture
Roger Federer beat Robin Soderling of manufacturing and services.
Sweden 6-1, 7-6, 6-4 in the men’s ● Consolidation of the existing flag-
singles final of the French Open 2009 in ship programmes.
Paris (France) on June 7, 2009. This
● Concerted action for the welfare of
victory took him level with great friend
women, youth, children, other back-
Pete Sampras of USA as the holder of
ward classes, SCs, STs, minorities,
14 Grand Slam titles. With this title
differently-abled and elderly.
Federer has joined the elite club of five
others—Fred Perry (Britain), Don Budge ● Governance reform.
(USA), Rod Laver (Australia), Roy ● Creation and modernisation of
Emerson (Australia) and Andre Agassi infrastructure and capacity addition.
(USA)—who have won all four Grand
Big Moment : Saina Nehwal scripted a ● Prudent fiscal management.
Slams.
remarkable victory over her higher- ● Energy security and environment
ranked opponent China’s Wang Lin in Women’s Singles—Russia’s
protection.
the Indonesian Open. Svetlana Kuznetsova claimed the
women’s singles title by defeating her ● Constructive and creative engage-
In the final, World No. 8 Saina ment with the world.
came from behind to outsmart World compatriot Dinara Safina 6-4, 6-2 on
No. 3 Wang and avenge her last June 6, 2009. By defeating her she ● Promotion of a culture of enterprise
week’s Singapore Open Super Series claimed her second Grand Slam title. and innovation.
loss to her. Men’s Doubles—India’s Leander ‘‘The yearning of our young people
Paes and the Czech Republic’s Lukas for inclusiveness—economic, social
Hailing Saina’s victory as an
Dlouhy won the men’s doubles title and cultural—and the rejection of
important milestone, Badminton
outclassing South Africa’s Wesley the forces of divisiveness…con-
Association of India announced a
Moodie and Belgium’s Dick Norman. tinues as both its (govt’s) inspiring
cash prize of Rs. 2 lakh as a recog-
Women’s Doubles—Anabel vision and unfinished business.
nition of here feat.
Medina Garrigues and Virginia Ruano —President, Pratibha Patil
The results (finals) : Women : Pascual of Spain clinched the women’s
Saina Nehwal (Ind) bt Lin Wang (Chn) doubles crown by defeating Victoria New Apex Body for Higher
12-21, 21-18, 21-9. Azarenka of Belarus and Elena Vesnina Education—The creation of a
Doubles : Eei Hui Chin & Pei Tty of Russia. National Commission for Higher
Wong (Mas) bt Shu Cheng & Yunlie Mixed Doubles—In the mixed Education and Research that will
Zhao (Chn) 21-16, 21-16. subsume as many as 13 existing
doubles, Liezel Huber and Bob Bryan of
USA beat Vania King of USA and profession councils and regulatory
Tennis Marcelo Melo of Brazil. agencies, including the University
Grants Commission, Medical Council
French Open Tennis Tourna- Cricket of India and the AICTE, is a key
ment 2009—Roger Federer gloriously
ICC World Twenty–20 Cricket recommendation of a committee
equalled Pete Sampras’ record of 14 Championship—Final of this tourna- headed by well-known educationist,
Grand Slam title, when he won his ment was played between Pakistan Professor Yashpal. A draft legislation
maiden French Open title in Paris on and Sri Lanka on June 21, 2009 at
and constitutional amendment are
June 7, 2009. Lord’s in London. Pakistan won the
recommended.
final by eight wickets.
Top Five Grand Slam Winners The proposal autonomous statu-
Man of the match—Shahid Afridi
tory body will comprise six members
14-Pete Sampras (U.S.) and Roger
Man of the series—Tillkaratne and a chairman appointed by the
Federer (Switzerland), 12-Roy Emerson
(Australia), 11-Bjorn Borg (Sweden) Dilshan President. State Higher Education
and Rod Laver (Australia), 10-Bill Councils, along the lines of those
Tilden (U.S.), 8-Andre Agassi (U.S.), existing in West Bengal, Kerala and
Jimmy Connors (U.S.), Ivan Lendl Andhra Pradesh, will form the second
(Czechoslovakia), Fred Perry (Britain) New Union Government’s tier of the system.
and Ken Rosewall (Australia). Agenda—While addressing the joint ●●●

C.S.V. / August / 2009 / 674


⎯⎯⎯⎯⎯⎯⎯⎯⎯⎯⎯⎯⎯⎯⎯⎯⎯ Inspiring Young Talent ⎯⎯⎯⎯⎯⎯⎯⎯⎯⎯⎯⎯⎯⎯⎯⎯⎯⎯
‘‘My hard working, teachers’ guidance, support of family members and
belief in God are the elements of my success.’’
—Rashmi Singh
Topper—U.P.-CPMT 2009 (1st Position)

[‘Competition Science Vision’ arranged an exclusive interview with Miss Rashmi Singh who has the
credit of standing first on the list of successful candidates in U.P.-CPMT, 2009. In addition she has also
cleared AIPMT (CBSE) and BHU Pre-medical Test with high ranks. For her brilliant success, she deserves
all praise and our heartiest congratulations. This important interview is presented here in its original form.]
CSV—Congratulations on your Rashmi—To both Botany and
brilliant success. Zoology I have given more weightage
Rashmi—Thank you. because they together make 50% of
CSV—Before knowing your result our question paper. These are
what did you think about those who scorable subjects but Physics and
achieve top positions ? Chemistry gave rank in exam.
Rashmi—I used to think that CSV—Did you make complete
God has given some special feature study of all topics or of some selective
in them. But now I am convinced that topics ?
nothing is special in them. Rashmi—I studied all the topics.
CSV—Achieving top position has CSV—How did you give final
come as surprise to you or were you touches to your preparation ?
confident of achieving it ? — CSV is really very helpful
in PMT examinations. It helps to Rashmi—Last time I prepared
Rashmi—It was really surprising the topics but I used to forget what I
to me. I had never thought of becom- understand basic concepts. Its
science tips, multiple choice read.
ing topper. But after giving examina-
tion of U.P.-CPMT and calculating my questions and assertion-reason Bio-Data
marks, I was expecting my position in type questions are really wonder-
ful for these tests. Name—Rashmi Singh
top ten and not of a topper.
Father’s Name—Sri Bhishma
CSV—What do you think is the —Rashmi Singh Pitamah Singh
secret of your success ?
Mother’s Name—Smt. Chanda
Rashmi—My hard working, CSV—What planning did you Singh
teachers’ guidance, my brother family make for preparation ? Please tell Educational Qualifications—
members support as well belief in something in detail. H.S./Std. X—85·4% (Raj English
God. School, Varanasi), 2006.
Rashmi—First, I used to make
CSV—In how many attempts did Inter/Std. XII—87% (Raj English
list of work. I have to do in a day. I
you get this success ? School, Varanasi), 2008.
used to use my full effort to complete
Rashmi—This was my 2nd Special achievements—
attempt. those pieces of work. I planned to
read and solve problems of NCERT ● 1st rank in U.P.-CPMT, 2009.
CSV—What were the shortcom- ● AIPMT (CBSE) Main—AIR-110
ings in your preparation for earlier text book at least for 3 times. Apart
(OBC-18)
attempts ? How did you make up for from this I used to solve objective
● BHU Screening—AIR 26th (OBC-5)
them this time ? after reading text book.
Rashmi—In earlier attempts, I did CSV—How much time did you CSV—Did you prepare notes ?
not prepare well any subject, I just devote daily and regularly for Physics, Rashmi—Yes, In coaching my
gave the exam for knowing pattern teachers helped me to prepare notes.
Chemistry, Zoology and Botany ?
etc. This time I had joined coaching
and from the starting of session I Rashmi—For me it was not CSV—What was your attitude for
started preparing for competitive possible to read all the four subjects solving numerical questions ? What
exam. daily. But I used to read three subjects weightage did you give them ?
CSV—From where did you get atleast. I devoted 3 hours for Botany, Rashmi—2 or 3 hours before
the inspiration of choosing a medical 3 hours for Zoology, 3 hours for commencement of any exam I used
career ? Botany. If time remained then 1 or 2 to solve numerical questions randomly
Rashmi—I belong to rural area. I hours for Physics. of any chapter. Due to this step and
used to see that many poor people tricks of numerical striked immediately
were not getting proper treatment. CSV—Out of the above four in mind. I gave much importance to
So, for them I thought that I have to subjects, to which subject did you give them because they helped me in
be a doctor. more weightage and why ? becoming topper.

C.S.V. / August / 2009 / 675


CSV—How much time is suffi- Rashmi— It was really very
cient for preparing for this exami- helpful to me. It helped me under-
Read Upkar’s
nation ?
Rashmi—Time is not an impor-
standing basic concepts. Science
Tips, multiple choice questions as
L EARN TO WRITE
CORRECT ENGLISH
tant factor. I did planning for a day. well as assertion-reason were very
Whatever the topics I had to complete wonderful for the medical students. (English-Hindi Medium)
them, I used to complete in specified CSV—Please suggest in what
time whether it took 2 hours or 10 way CSV can be made more useful
hours. for medical aspirants.
Rashmi—Latest discovery in
Personal Qualities various medical areas should be
Hobby—Watching cartoon published. For AIIMS something
Ideal Person—Dr. A.P.J. Abdul should be specially published.
Kalam CSV—Please mention your posi-
Strong Point—Hard work tion in the merit list as well as the
Weak Point—Nervousness marks obtained in different subjects.
What was your aggregate percentage Rs. 199·00
CSV—From what level of educa-
tion should an aspirant begin pre-
paring for it ?
of marks ?
Rashmi—Ist Position.
Physics—48/50
C ORRECT ENGLISH :
HOW TO WRITE IT
Rashmi—You must know basic Chemistry—48/50
concepts of all subject. You should try (English Medium)
Zoology—46/50
to feel the subjects.
Botany—50/50
CSV—What was your order of
Total—192/200, i.e., 96%.
preference for various branches for
which this test is held ? CSV—What books/magazines/
newspapers did you read for G.K.
Rashmi—MBBS, BDS, BAMS, preparations ?
BHMS.
Rashmi—Competition Science
CSV—Please mention various Vision.
books in each subject and magazines CSV—Whom would you like to
on which you based your preparation. give credit for your success ?
Rashmi—NCERT Text Book and Rashmi—I would like to give Rs. 195·00
Competition Science Vision magazine.
CSV—Did you take coaching in
credit of my success to my family,
relatives as well as my teachers.
L EARN TO WRITE
CORRECT ENGLISH
your preparation ? CSV—Please tell us something
Rashmi—Yes, JRS Tutorial, about your family. (English-Bangla)
Varanasi. I was very much impressed Rashmi—I have mummy, papa,
with the director of JRS Tutorial Mr. elder brother and sister. My father
A. K. Jha who taught me Physics in name is Mr. Bhishma Pitamah Singh
very scientific manner. I was also who is engineer in DRDA (Son-
impressed with Professor of Botany bhadra) and my brother is doing
Mr. Diwedi. MBBS, Allahabad and sister also
doing MBBS from Jhansi, my mother
CSV—What help do the science
is housewife.
magazines render in the preparations
for this examination ? CSV—What in your frank opinion
has been the biggest mistake in your
Rashmi—They helped me under- preparation for this test ?
standing basic concept in many
Rashmi—After giving any com-
topics.
petitive exam. I used to rest for Rs. 210·00
CSV—What will be your criterion atleast 3 days and I did not solve
for selecting a magazine for these By : Dr. B.B. Jain
Physics problem much.
examination ? CSV—What message would you As the Latest and All
Rashmi—Select those magazine like to give for our readers of CSV ? Comprehensive Books
which has contents of your syllabus Rashmi—Please read CSV very for
and latest G.K. seriously for easy selection in PMT as All Competitive
CSV—What is your opinion about it is very useful for medical students Examinations.
because all the four subjects are Purchase from nearest bookseller or get the copy by
our Competition Science Vision ? How V.P.P. sending M. O. of Rs. 100/- on the following address
much helpful and useful do you find given together in this magazine.
it ? ●●● UPKAR PRAKASHAN, AGRA–2

C.S.V. / August / 2009 / 676


⎯⎯⎯⎯⎯⎯⎯⎯⎯⎯⎯⎯⎯⎯⎯⎯⎯ Inspiring Young Talent ⎯⎯⎯⎯⎯⎯⎯⎯⎯⎯⎯⎯⎯⎯⎯⎯⎯⎯
‘‘Continuous hard work, Long practice and faith & belief in God are the
elements of my success.’’
—Anjali Singh
Topper—Uttarakhand, PMT–2009 (1st Position)

[‘Competition Science Vision’ arranged an exclusive interview with Miss Anjali Singh who has the
credit of standing first on the list of successful candidates in Uttarakhand PMT, 2009 in the very first
attempt. In addition, she has also cleared AFMC in the written examination. For this brilliant success she
deserves all praise and our heartiest congratulations. This important interview is presented here in its
original form.]
CSV—Congratulations on your sleep. Atleast 2 hours are required for
brilliant success. each subject and extra time is
Anjali—Thank you, sir. needed for solving MCQs.
CSV—Before knowing your result CSV—Out of the above four
what did you think about those who subjects, to which subject did you give
achieve top positions ? more weightage and why ?
Anjali—Same as now others Anjali—I payed more attention to
think of me. As brilliant and very hard- Physics and Botany. I gave maximum
working personalities. time to Biology as a whole as I think I
am a little weak in Biology (Botany
CSV—Achieving top position has
especially). I had a good command on
come as surprise to you or were you
Physics, but maintaining the same, it
confident of achieving it ?
—It is a very good magazine requires a lot of practice and a lot of
Anjali—As a surprise. But I was time.
and has influenced me much. It
confident of achieving a good rank in
contains good MCQs as well as
this PMT.
sound and brief material for
Bio-Data
CSV—What do you think is the revision. Interviews of toppers Name—Anjali Singh
secret of your success ? and high ranking candidates are Father’s Name—Mr. Desh Raj
Anjali—Continuous hardwork, very inspiring. Singh
long practice and faith & belief in God. Mother’s Name—Mrs. Vinay Singh
—Anjali Singh
CSV—In how many attempts did Educational Qualifications—
you get this success ? CSV—From when did you start H.S./Std. X—75% (St. Francis Sec.
Anjali—In Uttarakhand PMT, I the preparation for it ? School, Agra), 2003
had appeared 1st time but after 10 + 2 Anjali—I started my preparation Inter/Std. XII—77% (Holy Public
(2005), I had prepared for medical of medical entrance exam after School, Agra), 2005.
entrance exam for four years. intermediate. In XI and XII Std., I was Special achievements—
not so sincere and passed inter- ● 1st rank in state PMT
CSV—What were the shortcom-
mediate with both Maths and Bio. as ● I had been selected in AFMC
ings in your preparation for earlier
(written) also called for an inter-
attempts ? How did you make up for an average student.
view.
them this time ? CSV—What planning did you
Anjali—Earlier, I had not pre- make for preparation ? Please tell CSV—Did you make complete
pared some topics (Like Animal taxo- something in detail. study of all topics or of some selective
nomy, families of angiosperms etc.). Anjali—First I prepared my weak topics ?
Also, I was lazy at the last time after a topics in all subjects and had a Anjali—I studied all the topics
lot of hardwork in the beginning. But command on them. I did a lot of completely in the starting and solved
this year from the very starting. I pre- practice by solving MCQs of all all MCQs but at the time of exams, I
pared these topics well and continued subjects. I gave a lot of attention to left few topics of low weightage (Like,
my studies until the last exam. Botany and Physics. At the time of Aging, Virus etc.)
CSV—From where did you get exams, instead of solving MCQs, I CSV—How did you give final
the inspiration of choosing a medical concentrated on the revision of topics, touches to your preparation ?
career ? thoroughly. Anjali—By solving more and
Anjali—It was a dream for me CSV—How much time did you more multiple choice questions and
since my childhood. But when my devote daily and regularly for Physics, revising my topics frequently.
cousins and friends were selected, it Chemistry, Zoology and Botany ? CSV—Did you prepare notes ?
became an aim for me, which I had to Anjali—My study hours were not Anjali—I had taken coaching. So
achieve anyway. In childhood I liked fixed. I studied all the time but when I I already had notes. I wrote all the
the clinic of paediatricians very much. felt exhausted. I liked to take rest or a formulae and important points in

C.S.V. / August / 2009 / 677


Physics and Physical Chemistry. But Anjali—I used science maga- Anjali—My father is a Chemistry
did not prepared any notes in other zines CSV for solving MCQs and last lecturer. My mother is housewife. I
subjects, because biology, organic years exam questions. have one sister and two brothers, all
and inorganic chemistry don’t have are younger to me. My sister (Shilpi
CSV—What will be your criterion
any certainty from where questions Singh) had also qualified in CBSE
for selecting a magazine for these
may be asked. this year.
examination ?
Personal Qualities Anjali—I had not given much CSV—What in your frank opinion
importance to this aspect. But I admit has been the biggest mistake in your
Hobbies—Listening music (filmy
that for quick revision, magazines are preparation for this test ?
songs), cooking
Ideal Person—My mother and all helpful. Anjali—My unlimited sleep and
the selected students CSV—What is your opinion about laziness. When I lost my interest in
Strong Point—My continuous hard- our Competition Science Vision ? How studies in between and found it diffi-
work when I feel it is a requirement. much helpful and useful do you find cult to regain it. When I got good
Weak Point—My sleep and it ? ranks in tests (in watching), I became
silly mistakes while solving MCQs. quite loose.
Anjali—It contains good MCQs
CSV—What was your attitude for as well as a sound and brief material CSV—What message would you
solving numerical questions ? What for revision. It contain interviews of top like to give for our readers of CSV ?
weightage did you give them ? rankers, which is very much inspiring
Anjali—First make your concepts
for the students preparing for the
Anjali—I had Maths in XII, so I clear. Read the theory deeply, solve
same. It influenced me very much.
can do well in numerical questions. I MCQs, complete a book wholly,
gave them as much time as required CSV—Please suggest in what instead of solving few topics from
by using formulae and practising way CSV can be made more useful different books. Nothing is impossible
MCQs. for medical aspirants. if we have patience and strong will
Anjali—By adding NCERT based power. Impossible word itself says
CSV—How much time is suffi-
topics. Many new points had been ‘‘I am possible’’. Best of luck to all.
cient for preparing for this examina-
tion ? added in NCERT. So, I think, CSV ●●●
should contain a separate NCERT
Anjali—Two years study during
corner containing extra points for
XI and XII is sufficient if we study
revision. New Release
properly and seriously.
CSV—From what level of educa- CSV—Please mention your posi-
tion should an aspirant begin pre- tion in the merit list as well as the
paring for it ? marks obtained in different subjects.
Anjali—According to me, a
What was your aggregate percentage
of marks ?
Mathematical
student preparing for medical
entrance exam should begin the pre- Anjali—I got 1st rank in Formulae
paration from XI Std. Uttarakhand PMT (general) with 44 (Useful for Various Competitive Examinations)
CSV—What was your order of marks in Physics, 43 in Chemistry, 41
each in Zoology and Botany. All are Compiled by : Dr. N. K. Singh
preference for various branches for
which this test is held ? out of 50 on aggregate 169 marks out
of 200, i.e., 84·5%.
Code No. 1642

Anjali—Only MBBS.
CSV—What books/magazines/
Rs. 65/-

CSV—Please mention various


newspapers did you read for G. K.
books in each subject and magazines
preparations ?
on which you based your preparation.
Anjali—Elementary Biology, Anjali—I had read only daily
Pradeep Physics and Comprehensive newspaper (Amar Ujala) as only 20
Chemistry for subjective study. For questions in G. K. are asked in AIIMS.
preparing MCQs, I used objective So, I payed only a negligible impor-
Dinesh for Biology, CSV for all tance to G. K.
HINDI
subjects and Pradeep for Chemistry. CSV—Whom would you like to EDITION
CSV—Did you take coaching in give credit for your success ?
your preparation ? Anjali—I would like to give my
Anjali—Yes, I had taken coach- regards to God and my parents, who Code 248
ing for first two years (after XII) and had been always with me during my Rs. 76/-
then for one year at Agra. preparation. I am also very much
CSV—What help do the science thankful to all my respected teachers.
Upkar Prakashan, AGRA-2
magazines render in the preparations CSV—Please tell us something ● E-mail : publisher@upkar.in ● Website : www.upkar.in
for this examination ? about your family.

C.S.V. / August / 2009 / 678


12. The magnifying power of a telescope for image at
Physics infinity is
f
➠ M = – fo
e
1. Smaller is the potential gradient along a potentio-
13. The three characteristics of a heating wire are
meter wire, the more is the
➠ High resistivity, high melting point,
➠ Sensitivity of potentiometer heat resistant
2. The standard equation of trajectory of a projectile in 14. Intensity of electric field due to a point charge ‘q ’ at a
terms of range (R) is distance r
x
( )
➠ y = x 1 – R tan α →
➠ E =

1 q ^
. 2r
πε 0 r
3. Can Kirchhoff's laws be applied to both the direct and 15. Which of the two has more resistance : 100W
alternating currents ? tungsten bulb or 1000W heater, both marked for
➠ Yes 220 V.
4. What are the limitations of the Newton’s laws ? ➠ 100W tungsten bulb
→ → 16. What is the one basic difference between Biot-
➠ (a) The relation F m a would not hold good Savart’s law and Coulomb’s law ?
in case m does not remain constant. ➠ The charge element appearing in Coulomb’s
(b) The first two laws do not hold good in law is a SCALAR but the current element (idl )
each and every frame of reference. It is appearing in Biot-Savart’s law is a VECTOR
only in a very special frame of reference →
that the two laws of motion hold good. (i dl)
Such a frame of reference is called an 17. Kepler's third law is
inertial frame of reference. ➠ (Period of planet)2 ∝ (Distance from sun)3
5. What limits the energy that can be provided by a 18. The force between two magnetic poles of pole
cell ? strengths m and m′ separated by distance r is given
➠ Amount of reactants by
(chemicals used in the cell)
6. Excess pressure inside a liquid drop; inside a liquid
➠F=
μ0
π

( )
mm ′
r2
bubble, inside an air bubble in a liquid are
19. Planets are natural satellites of the
2T 4T 2T
➠ R ; R ; R ➠ Sun
20. What is the unit of ‘Magnetomotive force’ ?
7. Why is it necessary to keep the concentration of
CuSO 4 solution constant in Daniell cell ? ➠ Gilbert (Gb.)
➠ This helps to get a steady e.m.f.
8. What is the equation of a plane progressive simple Chemistry
harmonic wave ?
π

➠ y = a sin
λ
π
(vt – x ) = a sin 2π ( )
t

T λ
x
21. The true use of chemistry is not to make gold, but to
prepare medicines. Who said this ?
With usual meaning of notations ➠ Paracelsus (1493–1541)
9. How are the coils wound in a resistance box ? 22. The first super-conductor of family 1-2-3, discovered
➠ The resistance coil is doubly wound on a in 1987 is
bobbin to avoid electromagnetic induction ➠ Y Ba2Cu3O7
10. The equation of a stationary wave when the wave is 23. First modern text book of chemistry was written by :
reflected from a rigid boundary is ➠ Berzelius
πx
2π πt
2π 24. The reactions in which a single reagent undergoes
➠ y = 2a cos cos
λ T both oxidation and reduction are known as
11. Board of trade unit is defined as the amount of work ➠ Disproportionation reactions
done when a power of one kilowatt is consumed for 25. Meson theory of nuclear stability was given by :
➠ One hour ➠ Yukawa

C.S.V. / August / 2009 / 679 / 3


26. Isothermal gel-sol transformation brought about by 43. An union or joining of 2 or more arteries, veins or
shaking is termed as other vessels
➠ Thixothropy ➠ Anastamosis
27. The quantitative aspect of dealing with the mass and 44. What is called a unit of recombination ?
volume relation among the reactants and products is ➠ Recon
known as :
➠ Stoichiometry 45. Phenomenon in which synthesis of fatty acids and
keto acids takes place from amino acids
28. Fuel value of hydrogen is
➠ Ketogenesis
➠ 142 kJ/gm
46. What type of RNA carries a sequence of codons to
29. A mole is Chemists unit for counting atoms, mole-
ribosomes ?
cules, ions and other microscopic species. A mole
means : ➠ m RNA
➠ A collection of 6·022 × 1023 particles 47. Structures in which a bone fits into a socket of the
other.
30. An instrument used to detect and measure radiation
by the fluorescence is known as ➠ Gomphosis
➠ Scintillation counter 48. Which serous membrane lines the body cavity and
31. The ratio of charge (e) of the electron to its mass was covers the organs therein in many animals ?
found to be 1·76 × 108 coulomb/gm (5·28× 1017 ➠ Peritoneum
esu/gm). It was measured for first time by : 49. A hormone that stimulates growth of seminiferous
➠ Sir J. J. Thomson (1887) tubules and spermatogenesis in men
32. Two liquids which mix in all proportions are called ➠ Follicle stimulating Hormone
➠ Miscible 50. What is called the duct which derives from mesoderm
33. The amount of radiant energy in a photon is and conveys gametes or excretory products from the
proportional to the frequency of the radiation. This codon to the exterior ?
was the observation of : ➠ Coelomoduct
➠ Max Planck (1900) 51. Honey bees repair, filling and cementing cracks and
34. Bonding in which the bonding electrons are relatively crevices called
free to move throughout the three-dimensional ➠ Propolis
structure, is called
52. What is called an animal that warms his body mainly
➠ Metallic-bonding
by the heat of cellular respiration ?
35. The theory of quantum mechanics was independently
➠ An Ednotherm
and simultaneously formulated by
➠ Werner Heisenberg and Erwin 53. Abiotically polymerized amino acids that are joined in
a preferred manner
Schrodinger (1925-26)
➠ Proteinoid
36. A substance capable of behaving as either an acid or
a base is called 54. What is called the product made from swim bladder
➠ Amphoteric of fishes containing 90% gelatin ?
37. Cyclic aliphatic hydrocarbons are known as : ➠ Isinglass
➠ Alicyclic hydrocarbons 55. A group of structural and regulating genes that
38. A substance which acts as a proton donor is called functions as a single unit
➠ Bronsted-Lowry acid ➠ Operon
39. Organic compounds containing atoms such as O, S, 56. In which naturally occurring phospholipid complex,
N etc, as part of the ring system are known as : choline is bound to phosphoric acid group ?
➠ Heterocyclic compounds ➠ Lecithins
40. A cell used to obtain sodium metal by electrolysis of 57. A blood substance that produces vaso contraction
molten NaCl, is known as ➠ Angiotensin
➠ Downs cell 58. Which form of coelom is formed by the splitting of
embryonic mesoderm ?
➠ Schizocoel
Zoology 59. A pelagia free swimming, colonial luminiscent thalia-
cean is called
41. DNA that contains genes from more than one source ➠ Pyrosoma
➠ Recombinant DNA 60. Which portion of internal ear is coiled and contains
42. Who first discovered the lethal genes ? organ of hearing ?
➠ Cuenot ➠ Cochlea

C.S.V. / August / 2009 / 680


Botany Just Released

61. What are bacteriophages attacking E. coli called ?


➠ Coliphages
62. What often glucose metabolism provides ?
➠ Energy in the form of ATP
63. How many capsomeres are present in a virion of
φ ×174 ?
➠ Twelve
64. What causes separation of the complementary chains
of a DNA molecule ?
➠ Denaturation (For 10th Based Diploma Courses)
65. What is the time between infection of host and
assembly of new phages called ? By : Dr. M. B. Lal & J. P. Dixit
➠ Eclipse period
66. What is the chief means of achieving the genetic Code No. 1656 Price : Rs. 210/-
variation ?
➠ Mutation
Main Features
67. Who gave the concept of pure line ? ☞ Previous Year’s Solved
➠ W. L. Johanssen Paper
68. Which factor causes most mutation in nature ? ☞ Mathematics
➠ Radiation
☞ Science (Physics,
69. Which acid was first produced by fermentation ?
➠ Lactic acid Chemistry, Biology)
70. Who gave the concept of pure line ? ☞ General English
➠ W. L. Johanssen
71. What is Triticale ? ● E-mail : publisher@upkar.in
UPKAR PRAKASHAN, AGRA-2 ● Website : www.upkar.in
➠ A man-made cereal
72. What term is applied for specific sequence of deve-
12345678901234567890123456789012123
12345678901234567890123456789012123
lopment of a community relating to particular set of
physical and chemical conditions ? UPKAR’S 12345678901234567890123456789012123
12345678901234567890123456789012123
➠ Sere
73. What happens when a cell is placed in strong salt
Chhattisgarh
solution ?
➠ Water comes out by exosmosis, Pre-Medical Test
as a result the cell is shrinked
74. CO2, CFC, NO2 and CH4 causes global warming of ◆ Upkar’s Chhattisgarh PMT
atmosphere which is called Combined Guide–AT A Glance Rs. 435/-
➠ Green house effect Highlights
75. What type of inflorescence is found in Ocimum san- ☞ Objective Questions with Explanatory Answers.
ctum ? ☞ Quick Revision of Entire Syllabus.
➠ Verticillaster ☞ Help to Remember all Important Facts.
76. What are climbers growing on large trees and ☞ Provides Opportunity of Self-Assessment.
becoming woody due to secondary growth called ? ☞ Latest Facts and Discoveries.
➠ Lianas
77. What refers an allelomorph ? HINDI EDITION IS ALSO AVAILABLE
➠ Pairs of determiners
78. Where the main reservoir of nitrogen is present in Other Useful Books
atmosphere ? Upkar’s Practice Sets Medical Entrance Test 110/-
➠ Atmosphere Upkar’s Practice Work Book Medical Entrance Test 180/-
79. What is zeatin ? Upkar’s Quicker Numerical Physics 235/-
➠ A cytokinin-like substance isolated from milky Competition Science Vision Useful Extra Issues
endosperm of corn
Physics 165/- Zoology 115/-
80. What is that physiological process called where ions
move by mass flow and diffusion through the Chemistry 175/- Botany 115/-
apoplast ?
UPKAR PRAKASHAN, AGRA-2
➠ Passive uptake
● E-mail : publisher@upkar.in ● Website : www.upkar.in
●●●

C.S.V. / August / 2009 / 681


de-Broglie Hypothesis When the material particles like neutrons are in
The wave theory of light was capable of explaining thermal equilibrium at absolute temperature T, then they
the phenomena like reflection, refraction, interference, possess Maxwellian distribution of velocities and so their
diffraction and polarisation but it failed to explain photo- average kinetic energy is given by
electric effect and Compton effect. Quantum theory could 1
E = mv 2rms
explain these. Quantum theory says that a beam of light 2
of frequency ν consists of small packets each of energy 3
= kT
hν, called quanta or photons. These photons behave like 2
particles, thus, light sometimes behaves like a wave and where k is Boltzmann’s constant whose value is
sometimes like a corpuscle. 1·38 × 10–23 joule/K. So that
In 1924 de-Broglie proposed that a material particle h
λ =
such as an electron, proton, atom etc. might have a dual 3mk T
nature as light does. According to de-Broglie hypothesis a Davisson and Germer Experiment—This experi-
moving particle, whatever its nature, has wave ment performed by Davisson and Germer is the first
properties associated with it. de-Broglie proposed that experimental proof of the wave nature of material parti-
the wavelength λ associated with any moving material cles. The de-Broglie wavelength for an electron is less
particle of momentum p is ° A crystal lattice in which the atomic distances
than 1 A.
h between layers are of the order of 1 A ° is ideal for the
λ= ,
p purpose of studying the diffraction of electron waves. A
where h is Planck’s constant. nickel crystal C is taken. Electrons are made incident on it
The waves associated with a material particle are as a narrow beam. The incident electrons are produced
called matter-waves or de-Broglie waves and the wave- by an electron gun G. The gun consists of a tungsten
h filament F which is connected to a low tension battery.
length given by λ = is called de-Broglie wavelength. Electrons emitted from the filament F are made to pass
p
through pin holes under suitable accelerating potentials.
If m is the mass and v the velocity of the material
The accelerating potential is provided with the help of a
particle, then
high-tension battery. Ordinarily, we get an electron beam
p = mv of energy 50 eV from the gun.
h
or λ =
mv The resolving power of any microscope increases as the
If E is the kinetic energy of the material particle, then, wavelength used to illuminate the object decreases. Thus,
the electron waves have a smaller wavelength than light
1 p2
E = mv 2 = waves and so an electron microscope reveals much more
2 2m detail. The field ion microscope gives even greater resolution
because the waves associated with the helium ions used
or p = 2m E have an even shorter wavelength.
Therefore, the de-Broglie wavelength is given by
h The electron beam is made to fall normally on the
λ = crystal. The beam is diffracted by the crystal and received
2m E
at an angle θ by a detector D. The intensity of the diffrac-
Diffraction effects have been obtained with streams of ted electrons is measured by the detector as a function of
protons, neutrons and alpha particles, but it is evident from angle θ.
de-Broglie’s equation that the greater the mass of the In the original experiment, the detector was set at an
moving particle, the smaller is the associated wavelength
and so the more difficult detection becomes. Ordinary ob-
angle of 50° to the direction of the incident beam. The
jects have extremely small wavelengths. scattered electron current for different values of voltage V
was noted. The graph between the voltage and the
If a charged particle carrying charge q is accelerated detector current is as shown in figure on next page. The
through a potential difference of V volts, then existence of peak in graph can be explained as due to
Kinetic energy E = q V constructive interference of waves scattered from atoms
in different planes of the crystal. The peak occurs at 54 V.
In that case,
The observed phenomenon is similar to the diffraction of
h X-rays. So the experiment establishes the wave nature of
λ =
2mq V electrons.

C.S.V. / August / 2009 / 682


The detector current is maximum when Electron
diffraction
2d sin θ = n λ, rings
where n is the order of diffraction, d is the atomic spacing
Electron gun Thin
between successive crystal planes and θ is the angle at platinum
which strong reflection takes place. The value of d as foil Photographic
determined by X-ray reflection by nickel crystal, comes plate
° The value of θ comes out to be 65°.
out to be 0·91 A. Electron and neutron diffraction phenomenon is now
° commonly used for studying crystal structure like X-rays.
For n = 1, λ = 2d sin θ = 2 × 0·91 × sin 65° = 1·65 A. The electron beams can be conveniently produced and
We know that controlled as compared to X-rays.
h
λ = Study of Crystal Structure—The electrons interact
2me V
with the surface atoms much more intimately. Therefore,
In the given experiment electron diffraction is superior to X-ray diffraction to study
V = 54 volt the surface structure of crystals. The great advantage of
neutron diffraction over X-ray diffraction lies in the fact
6·63 × 10–34 that the neutron diffraction gives better information, for
λ =
(2 × 9 × 10 –31 × 1·6 × 10–19 × 54)1/2 neutrons being neutral particles can penetrate deeper into
° the nucleus. Moreover, neutron diffraction can give
= 1·67 A. information regarding lattice vibrations also and neutron
So, the wavelength determined by two different beam can be produced with very much smaller energy
methods comes out to be the same. This confirms that than is needed for the production of X-rays.
electrons are diffracted in the same way as the de-Broglie
Q. What is the de-Broglie wavelength of a neutron
waves. whose energy is 2 electron-volt ?
F Mass of neutron = 1·676 × 10–27 kg
Planck’s constant = 6·62 × 10–34 joule-sec
G
Solution :
Detector current

D h
λ =
θ 2m E
6·62 × 10–34
=
2 × 1·676 × 10–27 × 2 × 1·6 × 10–19
C 54 V = 0·202 × 10–10 m
Voltage V °
= 0·2 A
Fig. : Davisson and Fig. : Variation of detector
Germer experiment current with voltage Electron Lens
It is a device used to focus an electron beam. It is
Calculation shows that electrons accelerated through a analogous to an optical lens but instead of using a
p. d. of about 100V should be associated with de-Broglie refracting material, such as glass, it uses a coil or coils to
waves having a wavelength of the order of 10–10 m. This is produce a magnetic field or an arrangement of electrodes
about the same as for X-rays and it was suggested that the
between which an electric field is created. Electron lenses
conditions required to reveal the wave nature of X-rays might
also lead to the detection of electron waves.
are used in electron microscopes.

Thomson’s Experiment Source of electrons

In this experiment a beam of electrons obtained from


an electron gun is made to fall normally on a thin platinum Condenser lens
foil. The foil is nearly 10 –8 m thick. The foil can also be of Object
aluminium or gold. A photographic plate is placed behind
the foil. Diffraction pattern in the form of concentric rings is Objective lens
obtained on the photographic plate. These rings are
clearly due to the diffraction of waves associated with
electrons. This is because of the randomly oriented Intermediate image
crystals in the foil. From the geometry of the apparatus
the voltage used to accelerate the electrons and the Projector lens
diameter of the electron diffraction rings. The wavelength
associated with the beam of electrons can be determined.
It is observed that the experimental value of wavelength is
Final image
in close conformity with the theoretical wavelength. Thus,
the de-Broglie hypothesis is verified experimentally. Fig. : Principle of Transmission electron microscope

C.S.V. / August / 2009 / 683


de-Broglie wavelength associated with than light; for example, electrons accelerated to an energy
charged particles of 105 electron volt have a wavelength of 0·04 nanometre
(de-Broglie wavelength) enabling a resolution of 0·2–0·5
(1) For electrons ( me = 9·1 × 10–31 kg) nm to be achieved. The Transmission electron micro-
h
λ = scope has an electron beam, sharply focussed by elec-
2mqv tron lenses, passing through a very thin metallised speci-
6·62 × 10–34 men (less than 50 nanometre thick) onto a fluorescent
= m
2 × 9·1 × 10–31 × 1·6 × 10–19 V screen, where a visual image is formed. This image can
12·27 ° be photographed.
= A
V
Note—The potential difference required to have an de-Broglie wavelength associated with
° uncharged particles
electron of wavelength λ A is
150·6 (1) For neutrons (m n = 1·67 × 10–27 kg)
V = (From above equation)
λ2 h 6·62 × 10–34
λ = =
(2) For protons (mp = 1·67 × 10–27 kg) 2m E 2 × 1·67 × 10 –27 E
0·286 ° 0·286 °
λ = A = A
V E (eV)
(3) For deuterons (md = 2 × 1·67 × 10–27 kg) (2) For thermal neutrons at ordinary temperatures
0·202 ° E = kT
λ = A
V h
∴ λ =
(4) For α-particles (mα = 4 × 1·67 × 10–27 kg) 2mk T
0·101 ° 30·835 °
λ = A = A
V T
(3) For gas molecules
Electron Microscope h
λ = m×C
rms
It is a form of microscope that uses a beam of
⇒ For gas molecules at T K
electrons instead of a beam of light (as in the optical
3
microscope) to form a large image of a very small object. E = 2kT
In optical microscopes the resolution is limited by the h
wavelength of light. High-energy electrons, however, can ∴ λ =
3mk T
be associated with a considerably shorter wavelength

SOME TYPICAL SOLVED EXAMPLES

Example 1. Calculate the de-Broglie wavelength Example 3. What voltage must be applied to an
associated with a proton moving with a velocity equal electron microscope to produce electrons of wave-
1 °
to th of the velocity of light. length 0·5 A ?
20
Solution :
Solution :

Example 2. Show that the electrons accelerated Example 4. Calculate the de-Broglie wavelength
through a potential difference of V volts will have a of an α -particle accelerated through a potential diffe-
12·27 ° rences of 4000 volt.
wave of wavelength A associated with them.
⎯V
√ Given Planck’s constant
h = 6·62 × 10–34 joule-sec.
Solution :
Mass of proton mp = 1·67 × 10 –27 kg
Electronic charge e = 1·6 × 10 –19 coulomb
Solution :

C.S.V. / August / 2009 / 684


Example 7. What is the energy of gamma photon
° ?
having a wavelength of 1 A

(Given : Planck’s constant h = 6·62 × 10 –34 joule-


sec and speed of light c = 3 × 108 m/sec.)

Solution :

Example 5. Find the energy of the neutron in units


of electron volt whose de-Broglie wavelength is
°
1 A.
(Given : Mass of the neutron = 1·674 × 10–27 kg
Planck’s constant h = 6·60 × 10–34 joule-sec)

Solution :

Example 8. For a moving electron with mass


m = 2m 0, calculate the de-Broglie wavelength in terms
of rest mass m 0 and velocity of light c.
Solution :

Example 6. Energy of a particle at absolute


temperature T is of the order of k T. Calculate the
wavelength of thermal neutrons at 27°C. (Given : Mass
of the neutron = 1·67 × 10–27 kg. Planck’s constant
h = 6·60 × 10–34 joule-sec, and Boltzmann’s constant
k = 8·6 × 10 –5 eV deg–1.)

Solution :

Example 9. Calculate the de-Broglie wavelength


of an electron which has kinetic energy equal to
15 eV.
Solution :

C.S.V. / August / 2009 / 685


OBJECTIVE QUESTIONS
1. Matter waves— neutron (mass m n ), their velo- 13. What is the momentum of an
(A) Are electromagnetic waves cities should be in the ratio— °
electron if its wavelength is 2 A ?
(B) Are transverse waves (electron velocity/neutron velocity)
(Given : h = 6·63 × 1 0–34 joule-
(C) Are longitudinal waves me
(A) One (B) sec.)
mn
(D) Exhibit diffraction (A) 3·22 × 10–20 kg ms–1
mn
2. Neglecting variation of mass with (C) me × mn (D) (B) 23·2 × 10–21 kg ms–1
me
velocity, the wavelength asso- (C) 2·32 × 10–23 kg ms–1
ciated with an electron having 8. Which of the following figures (D) 3·32 × 10–24 kg ms–1
the kinetic energy E is proportio- represents the variation of parti-
14. Calculate the de-Broglie wave-
nal to— cle momentum and associated
de-Broglie wavelength ? length of an α-particle accelera-
(A) E1/2 (B) E–1/2 ted through a potential difference
(C) E (D) E–2 P P of 2000 volt—
(A) (B) °
(A) 3·2 × 10–1 A
3. A proton when accelerated
through a potential difference of °
(B) 5·0 × 10–2 A
V volts has a wavelength λ asso- P °
(C) 2·3 × 10–3 A
P
ciated with it. An α -particle in °
(C) (D) (D) 1·1 A
order to have the same λ must
be accelerated through a poten- 15. The de-Broglie wavelength of an
tial difference of— 9. A beam of monoenergetic neu- ° The energy
electron is 1·224 A.
(A) V volt (B) 4V volt trons corresponding to 27°C is of electron is—
V allowed to fall on a crystal. A first (A) 1 eV (B) 10 eV
(C) 2V volt (D) volt order reflection is observed at a (C) 100 eV (D) 1224 eV
8
glancing angle 30°, calculate the 16. The de-Broglie wavelength of
4. An electron accelerated through
interplanar spacing of the crys- 150 eV electron will be—
a potential difference of V volts
tal—(Given : Planck’s constant h °
(A) 1 A °
(B) 12·27 A
has a wavelength λ associated
= 6·62 × 10–34 joule-sec, Mass of ° °
with it. Mass of proton is nearly (C) 0·5 A (D) 1·5 A
neutron m n = 1·67 × 10 –27 kg,
2000 times that of an electron. In 17. Obtain the de-Broglie wave-
Boltzmann’s constant, k = 1·38
order to have the same λ for length of a neutron of kinetic
× 10–23 joule/degree) energy 150 eV—
proton, it must be accelerated
through a potential difference °
(A) 1·78 A °
(B) 2·18 A (Given : Mass of neutron = 1·675
of— × 10–27 kg)
°
(C) 8·12 A °
(D) 3·12 A
(A) V volt (B) 2000 V volt (A) 3·351 × 10–12 m
V 10. Calculate the de-Broglie wave- (B) 2·331 × 10–10 m
(C) volt (D) 2000 V volt
2000 length of neutron of kinetic (C) 2·335 × 10–12 m
5. In an electron microscope if the energy 54 eV— (Given : Mass of (D) 5·332 × 10–12 m
potential is increased from 20 kV neutron = 1·67 × 10–27 kg)
18. Calculate the de-Broglie wave-
to 80 kV, the resolving power R °
(A) 0·4 A °
(B) 4·0 A length for electrons if their speed
of the microscope will become— is 105 ms–1—
°
(C) 0·04 A °
(D) 0·004 A (Given : h = 6·63 × 10–34 joule-
(A) R (B) 2R
(C) 4R (D) R/2 11. Calculate the de-Broglie wave- sec)
length associated with a proton (A) 7·3 × 10–9 m
6. A proton and an α-particle are
moving with velocity 3 × 107 m/s— (B) 3·7 × 10–9 m
accelerated through the same
potential difference. The ratio of (A) 1·32 × 10–14 m (C) 5·3 × 10–9 m
their de-Broglie wavelengths (D) 5·5 × 10–9 m
(B) 1·14 × 10–14 m
λp/λα is— 19. Calculate the de-Broglie wave-
(C) 1·14 × 10–32 m
(A) 1 (B) 2 length for protons if their speed
1 (D) 4·14 × 10–10 m is 105 ms–1—
(C) 8 (D)
8 12. Calculate the de-Broglie wave- (Given : h = 6·63 × 10–34 joule-
length associated with an elec- sec)
7. One can study crystal structure
tron of 50 eV energy— (A) 39·71 × 10–12 m
by electron diffraction as well as
by neutron diffraction. In order to ° ° (B) 3·97 × 10–12 m
(A) 17·3 A (B) 1·73 A
have the same wavelength λ for (C) 9·37 × 10–12 m
° °
the electron (mass me) and (C) 7·13 A (D) 3·17 A (D) 7·39 × 10–12 m

C.S.V. / August / 2009 / 686


20. Electron microscope works on
the principle of—
(A) Particle nature of electrons
(B) Wave nature of light
(C) Quantum nature of light
(D) Wave nature of moving ele-
ctrons

ANSWERS WITH HINTS

(Continued on Page 747 )

C.S.V. / August / 2009 / 687


Isothermal Process cylinder of a petrol engine, where there is a compression
and expansion of a gas as a piston goes up and down.
It is a thermodynamic process in which the pressure
and volume of system change but temperature remains C
constant. An isothermal process is carried out either by
supplying heat to the substance or by extracting heat from
it. A process has to be extremely slow to be isothermal. P
B net
Examples of Isothermal Process work
done
D
(1) The temperature of a substance remains constant
during melting. So, the melting process is an isothermal A
process.
(2) When a substance boils, its temperature remains V
constant. So, boiling is an isothermal process. Thermodynamic variables or parameters are the
(3) Consider an ideal gas enclosed in a conducting quantities like pressure, volume and temperature which help
cylinder fitted with a conducting piston. Let the gas be us to study the behaviour of a thermodynamic system.
allowed to expand very slowly. This will cause a very slow
cooling of the gas. But heat will be conducted into the A to B—Gas (air-petrol mixture) is compressed
cylinder from the surroundings. Thus, the temperature of adiabatically by the raising piston. This causes a rise in
the gas remains constant. temperature.
If the gas is compressed very slowly, heat will be B to C—Ignited by a spark, the mixture explodes.
produced. But this heat will be conducted to the The further rise in temperature causes a further rise in
surroundings. So, the temperature of the gas remains pressure.
constant.
C to D—The hot high pressure gas pushes down the
Adiabatic Process piston as it expands adiabatically and the temperature
falls.
It is the thermodynamic process in which pressure,
volume and temperature of the system change but there Thermodynamic process is said to take place if some
is no exchange of heat between the system and the change occurs in the state of a thermodynamic system i.e.
surroundings. A process has to be sudden and quick to be the thermodynamic variable of the system change with time.
adiabatic.
D to E—The warm waste gas is removed and
Examples of Adiabatic Process replaced by cooler, fresh gas mixture, ready for the next
(1) Consider a gas enclosed in a thermally insulated cycle.
cylinder fitted with a non-conducting piston. If the gas is Note—From A to B, work is done on the gas. From C
compressed suddenly by moving the piston downwards, to D work is done by the gas. The shaded area represents
some heat is produced. This heat cannot escape the the net work done during the cycle.
cylinder. Consequently, there will be an increase in the
temperature of the gas. ● Cyclic process is that thermodynamic process in which
(2) If the above gas is suddenly expanded by moving the system returns to its initial stage after undergoing a
the piston outwards, there will be a decrease in the series of changes.
temperature of the gas. ● Non-cyclic process is that process in which the system
(3) Bursting of a cycle tube. does not return to its initial stage.
(4) Propagation of sound waves in a gas. ● Isolated system is that system which is completely
isolated from its surroundings.
(5) Expansion of gases in internal combustion engine.
(6) Expansion of steam in the cylinder of a steam First law of thermodynamics : If some quantity of
engine. heat is supplied to a system capable of doing external
work. Then the quantity of heat (d Q) absorbed by the
Thermodynamical system is an assembly of an system is equal to the sum of the increase in the internal
extremely large number of particles (atoms or molecules) so
that the assembly has a certain value of pressure, volume energy (d U) of the system and the external work ( d W )
and temperature. done by the system, i.e.

Indicator diagrams : Pressure-volume graphs are dQ = dU+dW


called indicator diagrams. They can be used to show the Equation of isothermal process : The perfect gas
cycle of changes taking place in an engine. The diagram equation is
below shows in simplified form, what happens in a PV = RT,

C.S.V. / August / 2009 / 688


where R is gas constant in an isothermal process, T is Work done in an isothermal process : Consider
constant. Therefore, one mole of an ideal gas enclosed in a cylinder having

PV = constant ( R is constant) perfectly non-conducting walls
and a perfectly conducting
That is, the product of the pressure (P) and volume bottom. Let the cylinder be fitted
(V) of a given mass of a perfect gas remains constant in with a frictionless and insulating
an isothermal process. So, Boyle’s law is obeyed in an piston of cross-sectional area A.
isothermal process. Let d W be the work done by the
Isotherm : A graph between pressure and volume of gas when the piston moves up
a given mass of a gas at constant temperature is known through an elementary distance
as isotherm or isothermal of a gas. In the figure below, d x . Let P be the pressure of the
two isotherms for a given gas at two different tempera- gas. Then,
tures T1 and T 2 are shown. d W = P × A × dx Fig. : Work done in
(work = force × distance) isothermal process
or d W = Pd V
T2 > T1 where d V is the infinitesimally small increase in the
P
volume of the gas.
Let Wiso be the total work done by the gas when the
T2
gas expands isothermally from an initial volume V1 to the
T1
final volume V 2.

V ⌠ V2
⎮ Pd V
Fig. : Isotherms of a gas
Then Wiso = ⎮
⌡ V1
Application of first law of thermodynamics to
isothermal process : The first law of thermodynamics RT
But, PV = RT or P =
states V
dQ = dU + dW where T is the constant temperature at which isothermal
The internal energy of an ideal gas depends only on expansion takes place.
temperature. In an isothermal process, temperature
remains constant. Therefore ⌠ V2
⎮ RT d V
dU = 0 ∴ Wiso = ⎮
⌡ V1 V
and dQ = dW
When an ideal gas expands isothermally, it does ⌠ V2
⎮ 1d V
= RT ⎮
mechanical work d W and absorbs an equivalent amount ⌡ V1 V
of heat d Q from the surroundings. Similarly, when an
ideal gas is compressed isothermally by doing a V2
or Wiso = RT [1oge V ]
mechanical work d W on it, it rejects an equivalent quantity V1
of heat d Q to the surroundings.
= RT [loge V 2 – loge V 1]
Q. Calculate the work done by an expanding gas.
Solution : V2 V2
or Wiso = RT loge = 2·3026 RT log10
V1 V1

The graph below left shows the expansion of a gas at


constant pressure. The area under the graph gives the work
done by the gas (PΔV). The same principle applies when the
pressure is not constant, as shown below right :

P P
Above, a gas at pressure P exerts a force PA on the area
ea =
piston and moves it a short distance Δx. If the expansion of work
the gas is so small that the pressure does not change : done area
ea
= work done
Work done by the gas = Force × displacement V V
O O
= PAΔx
But, AΔx = ΔV, the increase in volume Let P 1 and P2 be the pressures corresponding to
So, Work done by gas = PΔV the volumes V1 and V2 respectively.

C.S.V. / August / 2009 / 689


V2 P1 constant volume. This is equal to change in the internal
Then, P2V2 = P1V1 or = energy of the gas.
V1 P2
P1 d U = Cv d T
∴ Wiso = RT loge But, dQ = dU+dW
P2
P1 In an adiabatic process, no exchange of heat
= 2·3026 RT log 10 between the system and the surrounding is allowed.
P2
∴ dQ = 0
For μ moles of an ideal gas
So, Cv d T + Pd V = 0
V2
Wiso = μRT log e For an ideal gas, PV = RT
V1
Differentiating, P d V + V d P = Rd T
V2
= 2·3026 μRT log 10 Pd V + V d P
V1 or dT =
R
If we consider one gram of an ideal gas, But, Cv d T + Pd V = 0
RT V2
Wiso =
M
log e
V1 ∴ Cv [
Pd V + Vd P
R ]
+ Pd V = 0

where M is the molecular weight of gas or Cv P d V + Cv V d P + RPd V = 0


V2 or (Cv + R) Pd V + C v V d P = 0
or Wiso = r T loge
V1 But, Cp –C v = R
V2 or C p = Cv + R
= 2·3026 r T log 10
V1
∴ Cp P d V + Cv V d P = 0
where r is the principal gas constant, i.e., gas constant Dividing both sides by C v PV, we get
for one gram of gas.
Cp P d V Cv V d P
+ = 0
Equations of Adiabatic Process Cv PV Cv PV
∴ Cp
(i) Adiabatic relation between P and V for ideal
gas : Consider one mole of a gas contained in a perfectly
or γ
d V dp
V
+
P
= 0 [ Cv ]

non-conducting cylinder fitted with a non-conducting γd V


piston. Let P, V and T be the Integrating, ∫ V
+ ∫ dpP = constant
pressure, volume and tempera-
ture respectively of the gas. Let or γ ∫ 1V d V + ∫ 1Pd P = constant
the gas be compressed adiabati-
or γ log V + log P = constant
cally so that the piston moves
γ
inwards through a distance dx. or log V + log P = constant
Let A be the cross-sectional γ
or log PV = constant
area of a piston. γ
or PV = antilog (constant)
Force acting on the piston = another constant K
Fig. : Work done in
= P×A adiabatic process γ
∴ PV = K
Work done, d W = Force × distance
which is the required relation between the pressure and
= PAdx = Pd V volume of a gas.
where d V is the decrease in the volume of the gas. If P1V1 be the initial and P2V2 be the final pressure
and volume respectively of the gas for an adiabatic
Q. A certain volume of gas suffers an expansion of change, then
0·25 m3 at a constant pressure of 10 3 Nm–2. Calculate γ γ
the work done.
P 1V1 = P2V2

Solution : The work done W = PΔV (ii) Adiabatic relation between volume and
temperature : For a perfect gas
where P = Pressure
PV = RT
ΔV = Change in volume
RT
∴ W = 103 × 0·25 = 250 joule or P =
V
γ
The heat generated due to compression causes a But, PV = K
rise of temperature d T. This heat energy is equal to RT γ
∴ V = K
Cv d T, where Cv is gram molecular specific heat at V

C.S.V. / August / 2009 / 690


γ–1
or RT V = K ∴ dU+dW = 0
γ–1 K or dW = –dU
or TV = = K1 (say)
R
But d U = Cv d T
γ–1
∴ TV = constant where Cv is the molar specific heat of gas at constant
(iii) Adiabatic relation between pressure and volume.
temperature : For a perfect gas, ∴ d W = – Cv d T
PV = RT
Let Wadia be the work done when the gas expands
RT adiabatically from temperature T1 to temperature T2.
or V =
P T2
γ
But PV = K
RT γ
Then, Wadia = – ∫ Cv d T
∴ P
P [ ] = K
T1
T2

or
RT
P γ
γ γ
= K
= – Cv ∫
T1
dT
P T2
γ γ γ = – Cv [ T ] = – Cv [T2 – T 1]
or P1 – · R T = K T1
γ γ K or Wadia = Cv [T1 – T 2]
or P1 – T = γ = K2 (say)
R
γ γ which is the expression for the work done for one mole of
∴ P1 – T = constant an ideal gas during adiabatic process.
An adiabatic is steeper than an isotherm : For an Now, Cp – C v = R
isothermal process, PV = constant where Cp is the molar specific heat at constant pressure
Differentiating, Pd V + V d P = 0 and R is the molar gas constant.
dP P Dividing both sides by C v , we get
or = –
dV V Cp Cv R
dP – =
But represents the slope of isotherm Cv Cv Cv
dV
R
or γ–1 =
∴ Slope of isotherm,
dP
dV ( ) iso
= –
P
V
Cv
R
γ or Cv =
For an adiabatic process, PV = constant γ–1
Differentiating,
γ–1 γ
R
P·γ V dV + V dP = 0 So, Wadia = (T1 – T 2)
γ–1
γ γ– 1
or V d P = – γ PV dV
which is another expression for the work done during
dP γ PVγ – 1 adiabatic process.
or = –
dV Vγ If we consider of one gram of ideal gas.
γP
= – r
V Then Wadia = (T – T 2)
–1 1
dP P
Hence, slope of adiabatic, = –γ
dV V where r is the principal gas constant, i.e., gas constant for
1 gram of gas.
= γ ( )

P
V
The work done by an ideal gas during adiabatic expansion
= γ ( )
dP
dV iso
, (or compression) is proportional to the fall (or rise) in the
temperature of the gas.
(From above)
But, γ > 1 Note :
● If the gas expands adiabatically, work is done by the
Therefore, the slope of adiabatic is greater than the
gas. So, Wadia is positive.
slope of isotherm
or an adiabatic is steeper than an isotherm. ∴ T1 > T2
Work done in an adiabatic process : From first law ● So, the gas cools during adiabatic expansion.
of thermodynamics. ● If the gas is compressed adiabatically, work is done
dQ = dU+dW on the gas. So, W adia is negative.
In an adiabatic process, no exchange of heat is ∴ T1 < T2
allowed between the system and the surroundings. So, the gas heats up during adiabatic compres-
∴ dQ = 0 sion.

C.S.V. / August / 2009 / 691


Comparison of Amounts of Work Done during Also d W = Pd V = P × 0 = 0
Isothermal and Adiabatic Process According to first law of thermodynamics
In the case of expansion, the work done in an dQ = dU+dW
isothermal process is more So, mL = dU
than the work done in an
A Isotherm
adiabatic process as is That is, the internal energy increases by m L during
shown in fig. below. AB is P the melting process.
B
the isotherm and AC, the First law of thermodynamics and boiling process :
corresponding adiabatic.
Adiabatic
C On boiling a liquid, it changes into vapour at constant
The shaded area gives the
temperature called boiling point.
excess of work done in
isothermal expansion over Let a liquid of mass m vaporise. Let Vl and Vv be the
the work done in corre- V volumes of the liquid and vapours respectively.
sponding adiabatic expan- Fig. : Works done in isother-
The work done in expanding at constant temperature
sion. mal and adiabatic expansion
and pressure P.
Isothermal lies above the abiabatic in the case of d W = Pd V = P (Vv – Vl )
expansion. But in case of compression, the adiabatic lies
above the isothermal. Heat absorbed during boiling process,
dQ = mL
In the case of compression, the work done in an
adiabatic process is more where L is the latent heat of vaporisation.
than the work done in an Change in internal energy,
C Adiabatic
isothermal process as d U = Uv – U l
P
shown in figure below. AB B A
where U v and U l are the internal energies of the liquid and
is the isotherm while AC is
Isotherm vapours respectively.
the corresponding adiabatic.
The shaded area gives the First law of thermodynamics states
excess of work done in dQ = dU+dW
adiabatic compression over
the work done in corre-
V ∴ m L = (Uv – U l ) + P (Vv – Vl )
Fig. : Works done in isother-
sponding isothermal com- mal and adiabatic com- or Uv – U l = m L – P(Vv – Vl )
pression. pressions
First law of thermodynamics and melting process : ● The latent heat of fusion of ice is 80 cal g– 1 or k cal kg– 1
When a substance melts, the change in volume ( d V) is at normal atmospheric pressure.
very small and is negligible. The temperature remains ● The latent heat of vaporisation of water is 540 k cal kg– 1
constant during the melting process. or 540 cal g– 1 under normal atmospheric pressure.
Let a mass m of a substance be melted.
Heat absorbed during melting process d Q = m L From which the gain in internal energy can be
where L is the latent heat of fusion of substance. computed.

SOME TYPICAL SOLVED EXAMPLES

Example 1. Calculate the work done in compress-


ing 3 moles of a gas from 4 litre to 1 litre at constant
temperature. (R = 8·3 J mol–1, K–1)
Solution :

Example 2. Keeping the temperature constant at


27°° C one mole of a perfect gas is allowed to expand
from 4 atmospheric pressure to 1 atmospheric pres-
sure. Calculate the work done by the gas.
(R = 8·314 JK– 1 mol– 1)

C.S.V. / August / 2009 / 692


Solution : Example 6. The volume of 1 mole of oxygen at
constant pressure and temperature is 22·4 litre.
Calculate the two specific heats of oxygen.
Solution :

Example 3. Having suddenly compressed the


volume of dry air is reduced to one quarter at
atmospheric pressure. What will be its pressure ?
(γγ = 1·5)
Solution :

Example 4. A gas is compressed adiabatically to


one quarter of its initial volume at 17°° C. Calculate the
resulting temperature. (γγ for gas = 1·5)

Solution :

Example 7. Dry air at 15°° C and 10 atmospheric


pressure is suddenly released at atmospheric pres-
sure. Find out the temperature of air. (γγ for air = 1·41)
Solution :

Example 5. The density of a gas at 27°° C and


105 N-m– 2 pressure is 1·775 kg m– 3 and its specific
thermal capacity at constant pressure is 846 J kg– 1
K – 1. Determine the ratio of specific thermal capacity
at constant pressure to that at constant volume.

Solution :

C.S.V. / August / 2009 / 693


Example 8. At N.T.P. certain gas expands adia-
batically from 1 litre to 2 litre. Change the new
temperature of the gas. (γγ = 1·5)
Solution :

Example 9. A certain mass of air is expanded Example 10. A cylinder fitted with a piston
adiabatically at 0°°C so that its volume gets threefold. contains 0·2 mole of air at temperature 27°° C. The
How much does its temperature fall ? (γγ = 1·4) piston is pushed so slowly that the air within the
Solution : cylinder remains in thermal equilibrium with the
surroundings. Find the work done by the system if the
final volume is twice the initial volume.
Solution :

OBJECTIVE QUESTIONS
1. The first law of thermodynamics batically. Now which of the follo- 6. Calculate the work done to
is essentially a restatement of wing is true ? compress isothermally 1 g of
the law of— (A) Padia = Piso hydrogen gas at NTP to half of
(A) Conservation of momentum (B) Padia > Piso its initial volume. Gas constant
(C) Padia < Piso R = 8·31 J mol– 1 K– 1. Also find
(B) Conservation of charge the amount of heat evolved and
(C) Conservation of spin (D) Padia << Piso
the change in internal energy.
(D) Conservation of energy 4. A motor tyre pumped to a (A) – 786·3 J, 187·21 cal, 100 J
pressure of 3 atmosphere sud-
(B) 786·3 J, 187·21 cal, 1000 J
2. A certain gas at atmospheric denly bursts. What is the fall in
pressure is compressed adiabati- temperature due to adiabatic ex- (C) – 786·3 J, 187·21 cal, zero
cally so that its volume becomes pansion ? The temperature of air (D) 786·3 erg, 187·21 cal, 500
half of its original volume. before expansion is 27°C. erg
Calculate the resulting pressure
(Given γ = 1·4) 7. Ten moles of hydrogen at NTP
in dyne cm – 2. (Given : γ = 1·4)
(A) 219·2 K (B) 300 K are compressed adiabatically so
(A) 4·0 × 103 dyne cm – 2 (C) 75 K (D) 80·8 K that its temperature becomes
(B) 26·7 × 106 dyne cm – 2 400°C. How much work is done
5. A gram molecule of a gas at
on the gas ? Also calculate the
(C) 2·67 × 106 dyne cm – 2 127°C expands isothermally until
increase in the internal energy of
its volume is doubled. Find the
(D) 2·67 × 108 dyne cm – 2 the gas.
amount of work done and heat
(Given R = 8·4 J mol–1 K –1, γ = 1·4)
3. Two samples of a gas initially at absorbed.
(A) 2·301 × 1010 erg, 500 k cal (A) – 8·4 × 104 J, 8·4 × 104 J
same temperature and pressure
are compressed from a volume V (B) 2·301 × 1010 J, 547·9 cal (B) 8·4 × 104 J, – 8·4 × 104 J
V
to · One sample is compressed (C) 2·301 × 1010 erg, 547·9 cal (C) 8·4 × 104 J, 8·4 × 104 J
2
isothermally and the other adia- (D) 2·301 J, 600 cal (D) – 8·4 × 104, – 8·4 × 104 J

C.S.V. / August / 2009 / 694


8. A certain volume of dry air at NTP adiabatically to half the volume. C
is allowed to expand four times Its equation can be written as—
A
of its original volume under iso- (A) PV5/ 3 = constant B
thermal conditions. Calculate the
(B) PV7/ 5 = constant P
final pressure and temperature. B
(C) PV3/ 2 = constant
(Given : γ = 1·4)
(D) PV4/ 3 = constant
(A) 19 dyne cm – 2, 273°C
13. If internal energy of a gas V
(B) 19 dyne cm – 2, 0°C
decreases by an amount equal to
(C) 19 Nm– 2, 273 K (A) A (B) B
the external work, the gas is
(D) 19 cm of Hg, 273 K undergoing— (C) C (D) D

9. A certain volume of dry air at NTP (A) Adiabatic expansion 18. Four curves A, B, C and D are
is allowed to expand four times (B) Adiabatic compression drawn in the fig. for a given
of its orginal volume under adia- amount of gas. The curves which
(C) Isothermal expansion represent adiabatic and isother-
batic conditions. Calculate the
final pressure and temperature. (D) Isobaric expansion mal changes are—
(Given γ = 1·4) 14. The pressure and volume of a
(A) 10·91 dyne cm– 2, 0 K given mass of a gas at a given
temperature are P and V B C
(B) 10·91 cm of Hg, 156·8 K
respectively. Keeping tempera- D
(C) 10·91 Nm– 2, 273 K P
ture constant, the pressure is A
(D) 10·91 cm of Hg, 200°C
increased by 10% and then
10. A litre of hydrogen at 27°C and decreased by 10%. The volume
106 dyne cm– 2 pressure expands now will be—
isothermally until its volume is V
(A) Less than V
doubled. Find the final tempera- (A) C and D respectively
(B) More than V
ture, pressure and work done. (B) D and C respectively
(C) Equal to V
(Given γ = 1·4) (C) A and B respectively
(D) Less than V for diatomic and (D) B and A respectively
(A) 27°C, 5 × 1010 dyne cm–2,
more than V for monatomic
6·9 × 105 erg 19. Check the correct statement—
(B) 27°C, 5 × 10 5
dyne cm– 2, 15. 1 mole of oxygen is mixed with 1 (A) Heat is a path function, while
mole of argon. The external work internal energy is not
6·9 × 108 erg
done when the mixture is heated (B) Internal energy is a path
(C) – 45·6°C, 1·895 × 105 dyne
at constant pressure through 1°C function, while heat is not
cm– 2, 6·06 × 108 erg (C) Both heat and internal
is—
(D) – 40°C, 2 × 104 dyne cm– 2, energy are path functions
(A) R (B) 2 R
7 × 103 erg (D) Both heat and internal
(C) 3 (D) 4 R energy are not path func-
11. The equation of a certain gas
16. When a gas is isothermally tions
can be written as
expanded so that its volume 20. When a gas expands adiabati-
( )
T7 1/ 5
P2
= constant becomes twice, the rms velo-
city—
cally—
(A) No energy is required for
The specific heat at constant
(A) Becomes twice expansion
volume of the gas is—
(B) Energy is required and it
(in J/mol K) (B) Becomes 2 times
comes from the wall of the
(A) 0·5 R (B) 1·5 R (C) Remains same container of the gas
(C) 2 R (D) 2·5 R (D) Becomes 0·7 times (C) Internal energy of the gas is
12. Consider a hypothetical gas 17. Four processes are shown in fig. used in doing work
whose temperature increases to The one representing adiabatic (D) Law of conservation of
2 times when compressed compression is— energy does not hold

ANSWERS WITH HINTS

C.S.V. / August / 2009 / 695 / 4


11. In circuit shown below, the resis-
tances are given in ohms and the
battery is assumed ideal with
e.m.f. equal to 3 volt. The voltage
across the resistance R4 is—

1. Four smooth steel balls of equal 6. If M = mass, L = length, T = time


mass at rest are free to move and I = electric current, then the
along a straight line without fric- dimensional formula for resis-
tion The first ball is given a velo- tance R will be—
city of 0·4 m/s. It collides head on (A) [R] = [M1L2T–3I –2]
with the second elastically, the (B) [R] = [M1L2T–3I 2] (A) 0·4 V (B) 0·6 V
second one similarly with the
(C) [R] = [M1L3T–2I –2] (C) 1·2 V (D) 1·5 V
third and so on. The velocity of
the last ball is— (D) [R] = [M1L–2T–3I 2] 12. The internal energy U of a gas, in
(A) 0·4 m/s (B) 0·2 m/s 7. A wire of length L and cross general, consists of—
(C) 0·1 m/s (D) 0·05 m/s sectional area A is made of a (A) Only translational K.E. of all
material of Young’s modulus Y. If its molecules
2. A toy gun uses a spring of force the wire is stretched by an
(B) Only translational and vibra-
constant k. When charged before amount x, the work done is—
tional K.E. of all its mole-
being triggered in the upward YA YA 2
(A) x (B) x cules
direction, the spring is compres- L L
sed by x. If the mass of a shot is (C) Only translational, vibratio-
YA YA 2
m , on being triggered it will go (C) x (D) x nal and rotational K. E. of all
2L 2L
upto a height of— its molecules
8. A convex mirror of focal length f (D) Translational, vibrational and
kx 2 kx 2
(A) (B) produces an image 1/n of the rotational K.E. plus P.E. cor-
2mg mg
size of the object. The distance responding to molecular
(kx)2 x2 of the object from the mirror is—
(C) (D) forces, of all its molecules
mg kmg (A) f /(1–n) (B) (1 – n)f
(1 – n)f 13. A bi-convex lens made of glass
3. If the density of a small planet is (C) nf – 1 (D)
n (refractive index 1·5) is put in a
the same as that of earth, while liquid of refractive index 1·7. Its
the radius of the planet is 0·2 9. The total energy radiated from a
focal length will—
times that of the earth, the black body source is collected for
one minute and is used to heat a (A) Decrease and change sign
gravitational acceleration on the
quantity of water. The tempera- (B) Increase and change sign
surface of that planet is—
ture of water is found to increase (C) Decrease and remain of the
(A) 0·2 g (B) 0·4 g from 20 °C to 20·5°C. If the abso- same sign
(C) 2 g (D) 4 g lute temperature of the black
body was doubled and the expe- (D) Increase and remain of the
4. A small hollowsphere, which has riment repeated with the same same sign
a small hole in it, is immersed in quantity of water at 20°C, the 14. The concept of temperature to
water to a depth of 0·5 m before temperature of water will be— measure hotness or coldness of
any drop penetrates into it. If the (A) 21°C (B) 22°C a body is a consequence of—
surface tension of water is 0·073 (C) 24°C (D) 28°C (A) Joule’s law
N/m, the radius of the hole is— 10. An ideal monoatomic gas is (B) First law of thermodynamics
(A) 0·03 mm (B) 0·06 mm taken round the cycle ABCD as (C) Newton’s law of cooling
shown in the figure. The work
(C) 0·09 mm (D) 0·15 mm done during the cycle is— (D) Zeroth law of thermodyna-
mics
5. A satellite moves round the earth (2 P, V) (2 P, 2 V)
in a circular orbit of radius R B C
15. The displacement x (in metre) of
a particle in simple harmonic
making one revolution per day. A ↑ motion is related to time t (in
second satellite, also moving in a P seconds) as x = 0·01 cos
circular orbit, moves round the A D π
earth once in 8 days. The radius (P,V) (P, 2 V) ( )
πt +
4
. The frequency of the
of the orbit of the second satellite V→ motion will be—
is— 1
(A) PV (B) PV (A) 0·5 Hz (B) 1·0 Hz
(A) 8R (B) 4R 2 π
(C) 2R (D) R (C) 2 PV (D) Zero (C) Hz (D) π Hz
2

C.S.V. / August / 2009 / 698


16. In the phase diagram shown, the 21. A 4 μ F condenser is charged to (C) Q = – 2 q
point O corresponding to the 400 V and then its plates are (D) Q = 2 2 q
triple point of water. The regions joined through a resistance. The
I, II and III respectively corre- heat produced in the resistance 27. An electron beam in an X-ray
spond to— is— tube is accelerated through a
(A) 0·16 J (B) 0·32 J potential of 40 × 103 volt. These
↑ are then made to fall on a
P I (C) 0·64 J (D) 1·28 J
II Tungsten target. The shortest
………… 22. A charge Q is circulating with
O wavelength of X-rays emitted by
………

4·50 mm III constant speed v in a semi-


of Hg the tube is—
circular loop of wire of radius R. ° °
273·16 K T→ The magnetic moment of this
(A) 0·31 A (B) 3·1 A
loop is— (C) 0·31 cm (D) 3·1 cm
(A) Liquid, Solid, Vapour
Qv R × π 1 28. A conductor of length l is moving
(B) Solid, Liquid, Vapour (A) (B) Q v R
π+2 2 perpendicular to a uniform mag-
(C) Liquid, Vapour, Solid netic field of induction B and per-
(D) Solid, Vapour, Liquid Qv R × π pendicular to its length with a
(C) (D) Qv R
17. If the energy of the photon is 2(π + 2) uniform velocity v . As a result, an
increased by a factor of 4, then e.m.f. e = B l v is induced bet-
23. If F is the force required to keep
its momentum— ween its ends. Under this condi-
a train moving at a constant
(A) Does not change tion—
speed v , the power required is—
(B) Decreases by a factor of 4 1 2 (A) Free-electrons in the con-
(C) Increases by a factor of 4 (A) Fv (B) Fv2 ductor drift from higher
2
(D) Decreases by a factor of 2 potential end to lower poten-
1
(C) Fv (D) Fv tial end
18. The natural frequency of a vibrat- 2
(B) Free-electrons in the con-
ing body depends upon— 24. A potential difference of 220 V is
ductor drift from lower poten-
(1) Its length maintained across a 12,000 Ω tial end to higher potential
(2) Mass distribution rheostat, as shown in the figure. end
(3) Elasticity of the body The voltmeter has a resistance of (C) Free electrons in the con-
(4) Boundary conditions of the 6000 Ω and point C is at one- ductor do not drift along its
body fourth of the distance a to b. length, since there is no
(A) 1 and 2 only Therefore, the reading of the electric field inside the con-
(B) 1, 2 and 3 only voltmeter will be— ductor
(C) 1 and 3 only b (D) Free electrons in the con-
→⎯⎯ ⎯⎯→

(D) All the four ductor do not drift along its


length, since the force on
19. A piece of copper and another of 220 V ←⎯
c each electron due to the
germanium are cooled from room
V electric field is balanced by
temperature to 80 K. The resis-
the force produced on it by
tance of—
a the magnetic field.
(A) Each of them increases
(A) 32 V (B) 36 V 29. In the spectrum of hydrogen
(B) Each of them decreases
(C) 40 V (D) 42 V atom, the ratio of the longest
(C) Copper increases and that wavelength in Lyman series to
of germanium decreases 25. In an elliptic orbit under gravita-
tional force, in general— the longest wavelength in the
(D) Copper decreases and that Balmer series is—
of germanium increases (A) Tangential velocity is cons-
tant (A) 5/27 (B) 1/3
20. A biprism experiment is set up (B) Angular velocity is constant (C) 4/9 (D) 3/2
with edge of biprism vertical (C) Radial velocity is constant 30. Assertion (A) : It is convenient
when half the area on the right to define two specific heats C p
(D) Areal velocity is constant
hand side of the biprism is cove- and Cv in case of gases. How-
red with an opaque material, 26. A charge Q is fixed at each of ever, it is not generally neces-
then we shall see— two opposite corners of a square. sary to define two specific heats
(A) Fringes only in the left half of A charge q is placed at each of in case of solid or liquid.
the field of view the other two corners. If the
Reason (R) : For a given tempe-
resultant electric force on Q is
(B) Fringes only in the right half rature rise, the expansion of a
zero, then Q and q are related
of the field of view solid or liquid is negligible as
as—
(C) Intensity of the bright bands compared to a gas.
will be halved (A) Q = 2 q (A) Only A is true
(D) Uniform illumination (B) Q = – 2 2 q (B) Only R is true

C.S.V. / August / 2009 / 699


(C) Both A and R are true and R thread of the pendulum is g = 9·8 41. A charge Q is placed at the cen-
is the cause of A. m/s2). tre of a cube of side ‘a’. The flux
(D) Both A and R are true, but (A) 3 × 10–4 N (B) 4 × 10–4 N through any one of the sides is—
they are not related to each (C) 5 × 10–4 N (D) 6 × 10–4 N (A) Q/ ε0 (B) Q/6 ε0
other.
36. The stationary wave (C) Q/4πε0 (D) Q/24 ε0
31. In order to prepare a p-type semi-
y = 2a sin kx cos ωt 42. Two point charges of + 3
conductor, pure silicon can be
coulomb and + 9 coulomb repel
doped with— in a closed organ pipe is the
each other with a force of 27
(A) Phosphorous result of superposition of
newton. Charges of –3 coulomb
(B) Aluminium y = a sin (ωt – kx) and— are given to each of these
(C) Antimony (A) y = a sin (ωt + kx) charges, then the force of inte-
(D) Germanium raction is—
(B) y = – a sin (ωt – kx)
(A) 27 N (B) 18 N
32. Plane polarised light is passed
(C) y = – a sin (ωt + kx) (C) 9 N (D) Zero
through a polaroid. On viewing
through the polaroid we find that (D) None of these
43. Consider elastic collision of a
when the polaroid is given one 37. If a long spring is stretched by particle of mass m moving with a
complete rotation about the 0·02 m, its potential energy is U. velocity u with another particle of
direction of light, one of the If the spring is stretched by 0·1 the same mass at rest. After the
following is observed— m, then its potential energy will collision the projectile and the
(A) Intensity of light gradually be— struck particle move in directions
decreases and remains at
U making angles θ1 and θ2 respec-
zero (A) (B) U tively with the initial direction of
5
(B) Intensity of light gradually motion. The sum of the angles,
(C) 5U (D) 25 U
decreases but does not θ1 + θ2, is—
become zero 38. In Young’s experiment mono-
(A) 45° (B) 90°
(C) There is no change in inten- chromatic light is used to illu-
sity minate two slits A and B. Interfe- (C) 135° (D) 180°
(D) The intensity of light is twice rence fringes are observed on a 44. Identify the gate for which the
maximum and twice zero screen placed in front of the slits. truth table is given below—
33. The dimension of e2/4πε0hc , Now a thin foil of mica is place
normally in the path of the beam A B Y
where e, ε0, h and c are electro-
coming from the slit A. Then— 1 1 0
nic charge, electric permittivity,
(A) The fringes will disappear 1 0 1
Planck’s constant and velocity of
light in vacuum respectively, is— (B) The fringe width will 0 1 1
(A) M0L0T0 (B) M1L0T0 decrease 0 0 1
(C) M0L1T0 (D) M0L0T1 (C) The fringe width will (A) NAND gate (B) NOR gate
increase (C) OR gate (D) AND gate
34. A wire of length l, radius r, den-
sity d has a tension T. The esti- (D) The fringe width will remain
45. An ideal spring of force constant
mated graphs for frequency n of unchanged
k is broken into two identical
wire are drawn below. Which 39. A police car horn emits a sound pieces. The force constant of
graph is correct ? at a frequency 240 Hz when the each of the smaller springs is—
car is at rest. If the speed of (A) k /2 (B) k
↑ ↑ sound is 330 m/s, the frequency
n n (C) 2k (D) 4k
heard by an observer who is
approaching the car at a speed 46. With usual notations, the relation
l→ T→
(A) (B)
of 11 m/s is— between current gain α and β
(A) 248 Hz (B) 244 Hz is—
↑ ↑ (C) 240 Hz (D) 230 Hz α
n n (A) β = (B) β = α
40. Air is filled at 60°C in a vessel of 1–α
r → open mouth. Upto what tempera- 1–α α
d → (C) β = (D) β =
(C) (D) ture should the vessel be heated α 1+α
1
35. A pendulum bob of mass 30·7 × so that part the air may 47. A horizontal pipe of cross sectio-
4
10–6 kg and carrying a charge escape ? nal diameter 5 cm carries water
2 × 10–8 C is at rest in a hori- at a velocity of 4 m/s. The pipe is
(A) 273°C (B) 212°C
zontal uniform electric field of connected to a smaller pipe with
20000 V/m. The tension in the (C) 80°C (D) 171°C a cross sectional diameter 4 cm.

C.S.V. / August / 2009 / 700


The velocity of water through the voir at an absolute temperature T (B) Increase the maximum kine-
smaller pipe is— and rejects heat to a sink at a tic energy of the photo-elec-
(A) 6·25 m/s (B) 5·0 m/s temperature of T/3. The amount trons and would increase the
(C) 3·2 m/s (D) 2·56 m/s of heat rejected is— photoelectric current by a
(A) Q/4 (B) Q/3 factor of two
48. Energy of an electron in an exci-
ted hydrogen atom is – 3·4 eV. (C) Q/2 (D) 2Q/3 (C) Increase the maximum kine-
The angular momentum of the tic energy of the photo-elec-
50. The frequency and intensity of a
electron according to Bohr’s trons by a factor of two and
beam of light falling on the sur-
theory is— will have no effect on the
face of a photoelectric material
magnitude of the photoelec-
(A) 1·05 × 10–34 J-s are increased by a factor of 2.
tric current produced
(B) 2·1 × 10–34 J-s This will—
(D) Not produce any effect on
(C) 6·3 × 10–34 J-s (A) Increases the maximum
kinetic energy of the photo- the kinetic energy of the
(D) None of the above emitted electrons and would
electrons as well as the
49. A Carnot engine absorbs an photoelectric current by a increase the photoelectric
amount Q of heat from a reser- factor of two current by a factor of two.

ANSWERS WITH HINTS

C.S.V. / August / 2009 / 701


tion. The angle of prism of P′ will
be—
(A) 2·6° (B) 4°
(C) 5·33° (D) 3°
12. In figure below are given the four
1. When a coil carrying a steady 6. ALGOL resembles— lower energy levels of hydrogen
current is short-circuited, the atom. The probable number of
(A) COBOL transitions is—
current in it decreases n times in
time t0 . The time constant of the (B) BASIC n=4
circuit is— (C) FORTRAN n =3
t0 (D) All of the above n=2
(A) t 0 log n (B)
log n
7. When a hydrogen atom emits a
t0 t0 photon in going from n = 5 to
(C) (D) n=1
n n –1 n = 1, its recoil speed is almost— (A) 3 (B) 4
2. Which of the following is used in (A) 10–4 m /s (C) 5 (D) 6
diesel engine ?
(B) 2 × 10–2 m /s 13. The illuminance of a surface is
(i) Cylinder (ii) Piston (iii) Spark-
(C) 4 m /s 10 lux. If the total area of the
ing plug
surface is 30 cm2, then the
(A) (i) and (ii) only (D) 8 × 102 m /s
luminous flux incident on it will
(B) (i) and (iii) only 8. In an N-P-N transistor circuit, the be—
(C) All (i), (ii) and (iii) collector current is 10 mA. If 90% (A) 3 × 10 –4 lm (B) 3 × 10 –3 lm
(D) None of the above of the electrons emitted reach (C) 3 × 10 –2 lm (D) 3 × 10 –1 lm
the collector—
3. Which of the following state- 14. A ray of light passes through a
ments is correct ? (A) The emitter current will be 9
glass slab of thickness t and
mA
(A) A charged particle can be refractive index μ. If the speed of
accelerated by a magnetic (B) The emitter current will be
light in air be ‘c ’, the time taken
field 11 mA
by the ray to cross through the
(B) A charged particle can not (C) The base current will be 1 plate is—
be accelerated by a mag- mA t tc
netic field (A) (B)
(D) The base current will be –1 μc μ
(C) The speed of a charged mA μt μc
particle can be increased by (C) (D)
9. The emitter current in a transis- c t
a uniform magnetic field
tor circuit is 4 mA. The transistor 15. A beam of unpolarised light is
(D) The speed of a charged
used has α = 0·96. What are the passed first through a tourmaline
particle can be increased by
values of base current and collec- crystal A and then through ano-
a non-uniform magnetic field
tor current ? ther tourmaline crystal B oriented
4. Mirage is formed due to— so that its principal plane is
(A) 160 μA , 3·84 mA
(A) Reflection parallel to that of A. The intensity
(B) Refraction (B) 80 μA, 1·92 mA of final emergent light is I. The
(C) 60 μA, 1·24 mA value of I is—
(C) Total internal reflection
I0 I0
(D) Change in refractive index of (D) 40 μA, 0·96 mA (A)
2
(B)
4
air with change in tempe-
10. In the Bohr model of hydrogen I0
rature (C) (D) None of these
atom, the ratio of the kinetic 8
5. The activity of a sample of energy to the total energy of the 16. The mass of a proton is 1840
radioactive material is A1 at time
electron in n th quantum state times that of an electron. An
t 1 and A 2 at time t2 (t2 > t1). Its
is— electron and a proton, with equal
mean life is T—
(A) – 1 (B) + 1 kinetic energies, enter perpendi-
(A) A1t 1 = A2t 2 cularly uniform magnetic field.
1
A1 – A2 (C) (D) 2 Now—
(B) = constant 2
t2 – t1 (A) The path of proton will be
11. A thin prism P of angle 4° and more curved than that of the
(C) A2 = A1 e
( )
t1 –
t2
T made of glass of refractive index
1·54 is combined with another
electron
(B) The path of proton will be

(D) A2 = A1 e t2
T ( )
t1 thin prism P′ of refractive index
1·72 for dispersion without devia-
less curved than that of
electron

C.S.V. / August / 2009 / 703


(C) The paths of both proton 22. A magnet of magnetic moment effective inductance of 200 μH.
and electron will be equally M is freely suspended in a uni- What must be the range of its
curved form magnetic field of strength B. variable condenser ?
(D) The paths of both will be The work done in rotating the (A) 88–198 pf
straight magnet through an angle θ is
(B) 50 – 100 pf
given by—
17. In an interference experiment (C) 25 – 75 pf
third bright fringe is obtained at a (A) B M
(D) None of these
point on the screen with a light (B) M B sin θ
° (C) M B cos θ 28. The units nanometer, fermi, ang-
source of wavelength 7000 A.
strom and attometer, arranged in
What should be the wavelength (D) M B (1 – cos θ) decreasing order will read as—
of light source in order to obtain
fifth bright fringe at the same 23. When an air column at 15°C and (A) Angstrom, nanometer, fermi,
point ? a tuning fork are sounded attometer
° ° together, 4 beats per second are (B) Fermi, attometer, angstrom,
(A) 5000 A (B) 4200 A produced. The frequency of the nanometer
° ° fork is less than that of the air
(C) 6300 A (D) 7500 A (C) Nanometer, angstrom, fermi,
column. When the temperature
attometer
18. Ice skating can be used to falls to 10°C, the beat frequency
demonstrate that when ice is (D) Attometer, angstrom, fermi,
decreases by one. The fre-
under pressure, its— nanometer
quency of the fork will be—
(A) Melting point is lowered (A) 110 Hz (B) 114 Hz 29. The power radiated by a black
(B) Melting point is raised (C) 113 Hz (D) 106 Hz body is P, and it radiates
(C) Melting point remains un- maximum energy around the
24. When the number of turns in a
changed
coil is doubled without any wavelength λ0. If the tempera-
(D) Coefficient of friction with change in the length of coil, its ture of the black body is now
metal is reduced self-inductance becomes— changed so that it radiates
19. The intensity of light due to a (A) Four times (B) Doubled maximum energy around wave-
source of light in a room full of (C) Halved (D) Squared 3λ0
length , the power radiated by
smoke particles— 4
25. A source of A.C. voltage
(A) Is uniform at all distances it will increase by a factor of—
V = 180 sin ω t is connected in
(B) Obeys inverse square law (A) 4/3 (B) 16/9
series to a resistance and two
(C) Will increase more rapidly 64 256
reactances X L = 10 Ω and XC = (C) (D)
with distance than inverse 27 81
square law 15 Ω. Calculate the active power
if the current in the circuit is 3 A— → ^ ^ ^
(D) Will decrease more rapidly 30. The vector B = 5 i + 2 j – s k is
with distance than inverse (A) 100 watt (B) 200 watt
(C) 380·3 watt (D) 250 watt perpendicular to the vector
square law
→ ^ ^ ^
20. The three primary colours used 26. The system shown in the figure, A = 3 i + j + 2k for s =
in a colour television are— when slightly displaced and
released oscillates with a period (A) 1 (B) 4·7
(A) Red-blue-green
T. If only one spring is used, the (C) 6·3 (D) 8·5
(B) Green-yellow-red period of oscillation will be—
(C) Yellow-blue-black 31. Let ω be the angular velocity of
(D) Yellow-blue-red the earth's rotation about its axis.
Assume that the acceleration
21. The wavelength of H α line in K
due to gravity on the earth’s sur-
hydrogen spectrum was found to
face has the same value at the
°
be 6563 A in laboratory. If the equator and the poles. An object
K
wavelength of same line in the weighed by a spring balance
spectrum of a milky way is gives the same reading at the
° M
equator as at a height h above
observed to be 6586 A, then the
recessional velocity of the milky (A) T (B) T/2 the poles ( h << R). The value of
way will be— h is—
(C) T/ 2 (D) 2 T
(A) 105 m/s ω2 R2 ω2 R2
(B) 1·05 × 106 m /s 27. A radio can tune over the (A) (B)
g 2g
frequency range of a portion of
(C) 10·5 × 106 m /s MW broadcast band (800 kHz to 2ω2 R2 Rg
(C) (D)
(D) 0·105 × 106 m /s 1200 kHz). If its LC circuit has an g ω

C.S.V. / August / 2009 / 704


32. When a given amount of water is (C) 1.06 × 10–34 kg m2/s between the components of the
heated from 2°C to 8°C, its double star, if the total mass
(D) 3·24 × 10–34 kg m2/s
volume varies with temperature equals M and the period of
according to the curve— 39. A coin is placed on a horizontal revolution is T—
platform which undergoes simple GMT2 1/3
(A)
V
(B)
V
harmonic motion about a mean
position O. The angular frequ-
(A) ( )4π2

t ( °C) t
ency of the simple harmonic
motion is ω. The coefficient of
(B) ( )
GMT

1/3

(C)
V
(D)
V friction between the coin and the
platform is μ. The amplitude of
(C) ( )GT
4π2M
1/3

oscillation is gradually increa- MT2


t t sed. The coin will begin to slip on
the platform for the first time—
(D) ( )
4π2G
1/2

33. Two soap bubbles with radii r1 44. An aircraft is going at a speed of
g
and r2 ( r1 > r2) come in contact. (A) For an amplitude of 2 Mach 2. Its speed is nearly
Their common surface has a ω
……… km/hr.
radius of curvature r — (B) At the mean position
(A) 660 (B) 1080
r1 + r2 r1 r2 μg
(A) r = (B) r = (C) For an amplitude of (C) 1440 (D) 2380
2 r1 – r2 ω2
r1 r2 g 45. A uniform wire is bent in the form
(C) r = (D) r = r1r2 (D) For an amplitude of
r1 + r2 of a circle of radius r = 9·8 cm. It
μ ω2
is initially at rest and its diameter
34. The focal length of a convex lens
40. In the reaction OB is horizontal. It is swinging
having a magnifying power of
235 + 0n1 ⎯→ 54X142 + 36Kr89 about O in the vertical plane.
12·5 X is— 92 U
Calculate its angular velocity ω
(A) 8 m (B) 2 cm + 0n1 + α-particle
when its diameter occupies the
(C) 12·5 cm (D) 8 cm The number of α-particles emit- position OB'—
35. A metal wire of length L, area of ted is—
cross-section A and Young (A) Four (B) Three
modulus Y behaves as a spring O G
(C) Two (D) One B
of spring constant k—
YA 2YA 41. A sphere of mass m is suspen-
(A) k = (B) k = ded from a string of length l from
L L G
YA YL the point O. The sphere rotates
(C) k = (D) k = in a circular path in a horizontal
2L A
plane. The string makes an angle
36. The critical angle will be maxi- B
α with the vertical. Find the time
mum when light travels from—
period of rotation— (A) 2 rad/s (B) 15 rad/s
(A) Glass to air
(B) Water to air h (C) 10 rad/s (D) 20 rad/s
(A) T = 2π
(C) Glass to water 3g
46. If the intensity of sound is
(D) Water to glass 1 h doubled, the sound level will
(B) T =
37. The escape velocity for a planet 2π g increease by nearly—
is v e. A particle starts from rest at h (A) A factor of 2
a large distance from the planet, (C) T = 2π (B) 2 db
g
reaches the planet only under
π h (C) 3 db
gravitational attraction, and (D) T =
2 g (D) 4 db
passes through a smooth tunnel
through its centre. Its speed at 42. The fermi energy for a subs- 47. A man swimming at a rate of 5
the centre of the planet will be— tance— km/h wants to cross a 120 m wide
(A) v e (B) 1·5 v e (A) Is directly proportional to T river in a direction perpendicular
(C) 1·5 ve (D) 2 ve (B) Is proportional to T to the stream. If the stream speed
(C) Is independent of T is 4 km/h, find the direction in
38. When an electron jumps from
the second allowed orbit to the (D) Varies as T 2 which the man swims, and the
first allowed orbit in hydrogen, its time he takes to cross the river—
(T is the temperature in kelvin)
angular momentum
by—
changes
43. A double-star is a system of two
stars moving around the centre
(A) cos–1 ( )

4
5
, 144s

(A) 6.6 × 10–34 kg m2/s


(B) 6.6 × 10–27 kg m2/s
of inertia of the system due to
gravitation. Find the distance
(B) sin –1 ( )

4
5
, 100s

C.S.V. / August / 2009 / 705


(C) tan –1 ( )

4
5
, 200s
(B) Ammeter,
voltmeter
galvanometer,
(A)
2
g
(B)
g
2

(C) Ammeter, voltmeter, gal-


(D) cot–1 ( )

4
5
, 50s vanometer (C)
2
g
(D)
1
2g
(D) Voltmeter, ammeter, gal-
48. Which of the following combina- 50. The unit of e.m.f. is—
vanometer
tions is in order of increasing (A) joule
resistance ? 1 2 (B) joule-coulomb
49. If y = x – x is the equation of a
(A) Galvanometer, voltmeter, 2 (C) volt-coulomb
ammeter trajectory, find the time of flight— (D) joule/coulomb

ANSWERS WITH HINTS

C.S.V. / August / 2009 / 706


Acidity of X–H Bond in Non-metal ● The acid strength of conjugate acids is in the order as
Hydrides NH4+ < PH4+ < AsH4+

1. The Polarity of the X–H Bond 3. Charge on Acid or Base


● When all other factors are kept constant, acids become ● The charge on molecule or ion can influence its ability
stronger as the X–H bond becomes more polar. The to act as an acid or a base. This is clearly shown when
second row non-metal hydrides, for example, become the pH of 0·1 M solutions of H3 PO4 and H2 PO4–,
more acidic as the difference between electronegativity HPO 42– and PO43– ions are compared
(ΔEN) of X and H atoms increases. H3PO4 : pH = 1·5
H—F : Ka = 7·2 × 10– 4 ΔEN = 1·8 H2PO4 – : pH = 4·4
H2O : Ka = 1·8 × 10– 16 ΔEN = 1·2 HPO 4 2– : pH = 9·3
NH3 : Ka = 1 × 10– 33 ΔEN = 0·8 PO4 3– : pH = 12·0
CH4 : Ka = 1 × 10– 49 ΔEN = 0·4 ● A compound or ion becomes less acidic and more
● HF is strongest and CH4 is the weakest acid among basic as negative charge increases
above four compounds.
Acidity : H3PO4 > H2PO4– > HPO42–
● 0·1 M solution of HF is moderately acidic, H2O is much
less acidic and the acidity of NH3 is so small that the Basicity : H2PO4– < HPO42– < PO43–
chemistry of aqueous solution of NH3 is dominated by ● Similarly for H2SO4
its ability to act as base.
Acidity : H2SO4 > HSO4–
HF (0·1 M) ; pH = 2·1
Basicity : HSO 4– > H2SO4
H2O (0·1 M) ; pH = 7
NH3 (0·1 M) ; pH = 11·1 4. Oxidation State of Central Atom
● Stronger the acid weaker will be its conjugate base or ● There is no difference in polarity, size or charge when
vice-versa. The increasing order of basic nature of we compare oxyacids of the same element such as
conjugate bases is as : H2SO4 and H 2SO3 or HNO3 and HNO2
F– < OH– < NH2– < CH3– H2SO4 ; Ka = 1 × 103 HNO3; Ka = 28
2. Size of Atom, X H2SO3 ; Ka = 1·7 × 10– 2 HNO2; Ka = 5·1 × 10 – 4
● The Ka data for HF, HCl, HBr and HI reflect the fact Acidity of these oxyacids increases significantly as the
that X—H bond dissociation enthalpy (BDE) becomes oxidation number of central atom becomes larger.
smaller as the X atom becomes larger H2SO4 is much stronger than H2 SO3 and similarly
HF : Ka = 7·2 × 10– 4 BDE = 569 kJ/mole HNO3 is much stronger than HNO 2.
● This trend is easiest to be seen in four oxyacids of
HCl : Ka = 1 × 106 BDE = 431 kJ/mole
chlorine.
HBr : Ka = 1 × 109 BDE = 370 kJ/mole
Oxidation number of
HI : Ka = 3 × 109 BDE = 300 kJ/mole Acid Ka
Cl atom
● The increasing order of acid strength is HOCl 2·9 × 10– 8 +1
HF < HCl < HBr < HI HOClO 1·1 × 10– 2 +3
Similarly H2O < H2S < H2Se HOClO2 5·0 × 102 +5
● The presence of lone pair of electrons on the central HOClO3 1 × 103 +7
atoms of NH3 and PH 3, makes these hydrides Lewis
bases. HOCl is the weakest and HOClO 3 is the strongest acid.
MH3 + H+ ⎯→ MH4+ ● As the oxidation number of Cl atom increases, the
Base Conjugate acid
atom becomes more electronegative. This tends to
● As the size of central atom (N, P) increases, the draw electrons away from oxygen atom which
stability of conjugate acid decreases, i.e., M—H bond surround the Cl atom, thereby making oxygen atom
becomes weaker as size of central atom increases and more electronegative as well. As a result O—H bond
hence basic nature decreases as : becomes more polar and compounds becomes more
NH3 > PH3 > AsH3 ……… acidic.

C.S.V. / August / 2009 / 709


POINTS TO REMEMBER
● O and N atoms are about the same size, yet H 2O is much ● Liquid ammonia is an amphoteric solvent like water because
stronger acid as compared to NH 3. This is only on account of the auto-ionization of NH3 is as
O is more electronegative and O—H bond is more polar. 2 NH3 NH 4+ + NH 2–
● NH 4+ ion is a stronger acid than NH3 molecule because it is It is just like water
easier to remove H + ion from NH4+ than from NH3 molecule. H 2O H+ + OH–
● PH 3 is a stronger acid than NH3 because the compounds ● H2SO4, a stronger acid, gives proton to HNO3, a weaker acid
than H 2SO4.
become more acidic as the size of central atom holding H
hydrogen atom increases and X—H bond becomes weaker. |
● OH – ion is a conjugate base of H2 O and NH2 – ion is the H—OSO 2—OH + H—O–NO2 → H—O—NO2 + –OSO2OH
+
conjugate base of NH3. H2O is a stronger acid than NH3 and Acid Base Acid Base
hence OH– ion is a weaker base than NH2– ion. ↓
● PH3 is a stronger acid than NH3, which means the PH 2– ion H2O + NO2+
Nitronium ion
must be a weaker base than NH2– ion.
HNO3 is acting as a base.
● The relative strength of Bronsted bases can be predicted
● H2SO4 is a weaker acid than HClO4 and acts as a base to
from relative strengths of their conjugate acids, combined with
accept proton from HClO4.
the general rule that the stronger of a pair of acids always has H
weaker conjugate base. |
● A weaker acid is known to be displaced from its salt by a H—OSO 2–OH + H—OClO3 → –OClO3 + H— O—SO 2OH
+
stronger acid, e.g., acetylene displaces NH3 from its salt Base Acid Base Acid
sodamide. ● A strong acid must be stronger than H 3O+ and strong base
HC ≡ CH + NaNH 2 HNH2 + HC ≡ C Na stronger than OH– ion.
Stronger Stronger Weaker Weaker ● Feeble acids and bases are weaker than H 2O. Example
acid base acid base
acetylene (HC ≡ CH)
Water displaces acetylene from its salt. ● Weak acids are weaker than H 3O+ but stronger than H2O.
H–OH + HC ≡ CNa HC ≡ C–H + NaOH Weak bases are weaker than OH– but stronger than H2 O.
Stronger Stronger Weaker Weaker Most of the organic acids and bases are weak.
acid base acid base CH 3COOH + H2O H3O+ + CH3COO–
Hence acetylene is stronger acid than NH3, but weaker acid Acid Base Acid Base
(Weak) (Weak) (Strong) (Strong)
than H 2O.
CH 3NH2 + H2O OH– + CH3NH3+
H2O > HC ≡ CH > NH3 Base Acid Base Acid
● The decreasing order of acidity of hydrocarbons is as (Weak) Weak (Strong) (Strong)

HC ≡ CH > CH2 = CH2 > CH3–CH3 ● The substance which can act both as acid and base is said to
be ampholytic or amphoteric. For example : Liquid NH3, H2O,
Conversely the decreasing order of basicities of anions HF etc.
resulting from these hydrocarbons is as :
NH 3 + NH3 NH 4+ + NH2–
Acid Base Acid Base
: C2H5 > H2C —
— CH : > CH ≡ C :
H 2O + H 2O H3O+ + OH –
● The concentration of an acid solution is determined by how Acid Base Acid Base
many mole of acid is dissolved per litre and its strength is HF + HF H2F+ + F–
determined by how completely it ionizes. Acid Base Acid Base

OBJECTIVE QUESTIONS
1. Which of the following subs- 4. Which of the following acids is 7. Which of the following acids is
tances is not amphoteric ? strongest ? least ionized in 0·1 M solution ?
(A) HCO3– (B) H2O (A) HF (B) HCl (A) HCN (B) HF
(C) NH3 (D) NH4+ (C) HBr (D) HI (C) H2SO3 (D) H2CO3
2. Which is the strongest acid 5. Which is the weakest Bronsted 8. Which is the weakest Bronsted
among the following ? base ? acid among the following ?
(A) Acetylene (B) Water (A) F– (B) Cl– (A) HF (B) H2O
(C) Ammonia (D) Ethylene – (C) NH3 (D) CH4
(C) Br (D) I –
3. Weak acids are— 9. For the reaction
6. Which of the following sub-
(A) Weaker than H2O
stances has the highest pH for Zn 2+ + X– ZnX +
(B) Stronger than H 3O+ 0·1 M solution ? The k eq is greatest when X – is—
(C) Stronger than H 2O (A) NaH2PO4 (B) Na 2HPO 4 (A) NO3– (B) I –
(D) Always amphoteric (C) Na 3PO4 (D) H3PO4 (C) ClO3 – (D) F–

C.S.V. / August / 2009 / 710


10. The correct decreasing order of 19. The conjugate base of ammo- (C) HClO > H3PO4 > H2SO4
basic character is— nium ion is— (D) H3PO4 > HClO > H2SO4
(A) CH3– > OH– > NH2– > F– (A) NH2– (B) NH4+
27. Which can act both as Bronsted
(B) CH3– > – NH2– > OH– > F– (C) NH3 (D) OH– acid and base ?
(C) F– > NH2– > OH– > CH3– 20. Which is the correct order of (A) Cl–
(D) NH2– > CH3– > OH– > F– basic nature ? (B) HCO3–
11. Among HS –, I –, R–NH 2 and NH3, (A) H2O > NH3 > PH3 (C) H3O+
the proton accepting tendency (B) NH3 > PH3 > H2O (D) Both (B) and (C)
will be maximum and lowest (C) NH3 > H2O > PH3
respectively in— 28. Correct increasing order of aci-
(D) PH 3 > NH3 > H2O dity is—
(A) R–NH2 and I –
21. NH3 gas dissolves in H2O to give (A) H3PO4 < HCl < H2CO3 < HI
(B) NH3 and HS –
NH4OH. H2O acts as— (B) H3PO4 < H2CO3 < HCl < HI
(C) I – and HS –
(A) An acid (C) H2CO3 < H3PO4 < HCl < HI
(D) HS – and I –
(B) A base (D) None is correct
12. The electronegativity of chlorine (C) A conjugate base
atom will be highest in— 29. The conjugate acid of azide ion
(D) Amphoteric solvent is—
(A) HOCl (B) HOClO
(C) HOClO2 (D) HOClO3 22. In the reaction (A) NH3 (B) HN3
HClO4 + H2O H3O+ + ClO4– (C) NH2– (D) N2–
13. Which among the following is the
weakest base ? (A) HClO4 is a conjugate acid of 30. Strongest conjugate base results
(A) PO43– H2O from—
(B) H2PO4– (B) H2O is a conjugate acid of (A) Formic acid (B) Acetic acid
(C) HPO 42– H3O+ (C) Ethylene (D) Acetylene
(D) All are equally basic (C) H3O+ is a conjugate base of
31. Liquid ammonia, like water is an
H2O
14. Which of the following factors amphiprotic solvent. Which is the
explains the higher acid strength (D) ClO4– is the conjugate base appropriate auto-ionization equa-
of HI than that of HF ? of HClO4 tion for liquid NH 3 ?
(A) Polarity in H–X bond 23. An aqueous solution of acetic (A) NH3 NH2– + H+
(B) Charge on the molecule acid contains— (B) NH3 + H+ NH4+
(C) Size of atom X (A) CH3COO– and H + (C) 2 NH3 NH4+ + NH2–
(D) Oxidation state (B) CH3COO–, H3O+ and H +
(D) All of these
15. Which of the following is (C) CH3COO–, H3O+ and
CH3COOH 32. Which of the following ions in
incorrect statement ?
aqueous solution gives a neutral
(A) PH 3 is a stronger acid than (D) CH3COOH, CH3COOH and
solution ?
NH3 H+ (A) SO32– (B) NH4+
(B) NO3– is a weaker base than 24. Which is the correct decreasing (C) Na + (D) F–
NO2– order of basic strength ?
33. Which is the correct representa-
(C) PH 2– is a weaker base than (A) CH3–CH2– > NH2– > HC ≡ C– tion of acidic nature of AlCl3 in
NH2–
> OH– water ?
(D) H2O and liquid NH3 are not
amphiprotic solvents (B) HC ≡ C– > CH3–CH2– > NH2– (A) AlCl3 + 3 H2O Al(OH)3
> OH– + 3 HCl
16. The strongest Bronsted base
(C) OH– > NH2– > HC ≡C– (B) [Al (H2O)6] 3+
among the following is—
(A) ClO– (B) ClO2– > CH3–CH2– [Al (H2O)5] 3+ + H2O
(C) ClO3– (D) ClO4– (D) NH2– > HC ≡ C– > OH– (C) [Al (H2O)6] 3+
[Al (H2O)5 OH]2+ + OH–
17. Which is the weakest Lewis > CH3–CH2–
base ? (D) [Al (H2O)6] 3+ + H2O
25. The conjugate base of HPO42–
(A) H– (B) OH– is— [Al (H2O)5 OH]2+ + H3O+
(C) Cl – (D) HCO3– (A) PO43– (B) H2PO4– 34. Amino acid, glycine exists pre-
18. HNO3 in liquid HF behaves as— (C) H3PO4 (D) H4PO3 dominantly in the form of
+
(A) An acid 26. Correct decreasing order of acid NH3CH2COO–. Which is the
(B) A base strength is— conjugate acid of glycine ?
(C) Neither a base nor an acid (A) H2SO4 > HClO > H3PO4 (A) NH2CH2COOH
(D) As a base as well as an acid (B) H2SO4 > H3PO4 > HClO (B) NH2CH2COO–

C.S.V. / August / 2009 / 711 / 5


+
(C) NH3 CH2COOH (C) It is an example of anionic (C) BCl 3 and AlCl3 are both
+ hydrolysis equally strong Lewis acids
(D) NH3 CH2COO–
(D) Bicarbonate ion forms ano- (D) BCl3 and AlCl3 are both
35. Which is the Lewis acid-base ther anion Lewis acids and AlCl3 is
reaction ? stronger than BCl3
(A) Ca + S → Ca2+ + S2– 39. Which of the following reactions
43. The strongest Lewis base among
(B) NH3 + HCl → NH4+ + Cl– will occurs when sodium hydride
the following is—
is dissolved in water ?
(C) NH3 + BF3 → H3N : BF3 (A) CH3– (B) F–
(D) None of these (A) H–(aq) + H2O → H3O– (C) NH2 – (D) OH–
36. According to Lowry and Bronsted (B) H+(aq) + H2O → H3O+ 44. H+ is a—
concept, Cl– ion in aqueous solu- (C) H–(aq) + H2O → OH – + H2 (A) Lewis acid
tion is a— (B) Lewis base
(A) Weak base (B) Strong base (D) None of these
(C) Bronsted base
(C) Weak acid (D) Strong acid
40. Water can act as an acid in pre- (D) None of these
37. Ionic dissociation of CH 3COOH sence of— 45. Cl– is a conjugate base of—
is represented as— (A) HCl (B) NH3 (A) HOCl (B) HOClO
CH3COOH + H2O
(C) C6H6 (D) H2SO4 (C) HCl (D) None of these
H3O+ + CH3COO–
According to Lowry and Brons- 41. Which among the following is the ANSWERS
ted, in this equation we have— weakest base ?
(A) One acid and three bases
(B) One acid and one base (A) C2H5O– (B) F–
(C) Two acids and two bases (C) NO3– (D) CH3COO–
(D) Three acids and one base 42. Which is the correct statement ?
38. An aqueous solution of Na + (A) BCl3 and AlCl 3 are both
HCO3– is alkaline because— Lewis acids and BCl3 is
(A) Bicarbonate ion is alkaline stronger than AlCl3
(B) It is an example of cationic (B) Both BCl3 and AlCl3 are not
hydrolysis Lewis acids ●●●

Exam. Date Exam. Date


6 Sept., 2009 16 Aug., 2009

(Including Previous Years’ Solved Papers)


(Including Previous Years’ Solved Papers)
Main Features : By : Dr. Lal & Jain Code No. 1619 Rs. 270/-
 Test of Reasoning Ability
Highlights
 Test of English Language
● General Awareness ● Educa-
 Test of Numerical Ability tional Awareness ● General English
Comprehension ● Logical and
 Officework Aptitude Analytical Reasoning ● Teaching-
Learning and the School.
Subject Competence
By : Dr. Lal & Jain ● Science ● Mathematics
Code No. 307 Rs. 135/- ● Social Studies ● English

HINDI EDITION Code No. 1129 Rs. 140/- HINDI EDITION Code No. 1119 Rs. 230/-

UPKAR PRAKASHAN, AGRA-2 Upkar Prakashan, AGRA-2


● E-mail : publisher@upkar.in ● Website : www.upkar.in E-mail : publisher@upkar.in Website : www.upkar.in

C.S.V. / August / 2009 / 712


Introduction ● From sulphonic acids—Fusion of sodium salt of sul-
● Aromatic hydroxy compounds are of two kinds (i) phonic acid with NaOH followed by acidification,
gives phenols
Phenols : in which hydroxy group (groups) is attached
Fuse
to aromatic nucleus (benzene) (ii) aromatic alcohols : SO3Na + 2NaOH ⎯→ ONa + Na2SO3
in which hydroxy group is present in side chain e.g.,
benzyl alcohol, C 6H5CH2OH. Sod. sulphonate Sod. phenoxide

● Phenols are of following kinds : H2SO 4


⎯⎯→ OH
(i) Monohydric phenols—
Phenol
OH CH3 CH3 CH 3 ● From aryl halides (Dow’s method)—(i) This method
has limited application in laboratory preparation of
OH simple phenols.
, , , ,
OH
Phenol o-cresol m -cresol Cl + NaOH
OH
p -cresol
Chlorobenzene
OH 300 – 350°C‚ High pressure
⎯⎯⎯⎯⎯⎯⎯⎯⎯⎯→ OH + NaCl
OH Cu. Salt
,
Phenol

α-naphthol β-naphthol (ii) Aryl halides are first converted into their magne-
sium compound (Grignard reagent) in presence of
(ii) Dihydric phenols— ether, which soon changes to phenol by oxidation
OH and subsequent hydrolysis.
OH
Mg [O]
C6H5Br ⎯⎯→ C6H5MgBr ⎯→ C6H5OMgBr
OH Ether
H O
, OH , ⎯⎯→
2
C6H5OH + HOMgBr
Catechol Resorcinol Phenol
( o-dihydroxy benzene) ( m-dihydroxy benzene)
● Higher homologues from lower ones—This is
OH achieved by heating lower homologue with alcohol in
presence of anhydrous zinc chloride.
CH3
anhy. ZnCl
C6H5OH + CH3OH ⎯⎯⎯⎯→
2
C6H4 + H 2O
OH Phenol OH
Hydroquinone or quinol o- and p-cresols
( p-dihydroxy benzene) ● Raschig process—
Cu/Fe
(iii) Trihydric phenols— C6H5H + HCl + O(Air) ⎯⎯→ C6H5Cl + H2O
OH OH OH 425°C
C6H5Cl + H2O (Steam) ⎯⎯→ C6H5OH + HCl
OH , OH , Phenol
OH ● From decarboxylation of salicylic acid with soda
Pyrogallol HO OH lime—
OH Phloroglucinol
Hydroxy quinol OH ONa OH

Methods of Preparation NaOH‚ CaO


COOH ⎯⎯⎯⎯→ HCl
⎯→
360 K
● From diazonium compounds—When diazonium
Salicylic acid Sod. phenoxide Phenol
sulphates are boiled with water or with solution of
CuSO 4 or steam distilled, phenols are formed ● From cumene—Petroleum is recent commercial
source of manufacture of phenol. Benzene and pro-
+ H O‚ H+ pene obtained from petroleum are made to undergo
N2HSO 4– ⎯⎯→
2
OH + N2↑ + H2SO4
Δ Friedel-Crafts reaction to give cumene. This on
Benzene diazonium Phenol oxidation in presence of a metal catalyst followed by
hydrogen sulphate treatment with acid gives phenol.

C.S.V. / August / 2009 / 713


CH3 ● Properties of the nitrophenols
Anhy. AlCl 3 | Name B.P. °C at Solubility in Remark
C6H6 + CH3CH —
— CH2 ⎯⎯⎯⎯→ C6H5 —CH—CH3 70 mm g/100 gm H 2O
Cumene o-nitrophenol 100 0·2 Steam volatile due
CH3 to intramolecular
H-bonding.
|
Oxidation Dil. H 2SO 4 m -nitrophenol 194 1·35 Non-volatile in
⎯⎯⎯→ C6H5 —C— O—OH ⎯⎯⎯→ steam
Aerial | p-nitrophenol High (Dec.) 1·69 Non-volatile in
CH3 steam
Cumene hydroperoxide
CH3COCH3 + C6H5OH (A) Properties in which Phenols Resemble Alco-
Acetone Phenol hols (Reactions due to —OH Group)
Properties of Phenols ● Reaction with Na—Sodium phenoxide is formed.
● Phenols usually have high boiling points due to inter- 2C 6H5OH + 2Na → 2C6H5ONa + H2↑
molecular hydrogen bonding. For example b.p. of Phenol Sod. phenoxide
phenol (C6H5OH), mol. wt. = 94, is 453K while that of
● Esterification—With acids in presence of poly-
toluene (C 6H5CH3), mol. wt. 92, is 384 K.
phosphoric acid or toluene sulphonic acid, phenyl
● o-nitrophenol (intramolecular hydrogen bonding) has
esters are formed. However, the yield of ester is poor
much lower b.p. (Steam volatile) than m -and p-nitro- than that obtained from alcohols.
phenols (intermolecular hydrogen bonding.) TsOH
● Phenol on account of corrosive action, produces ArOH + R—COOH ⎯⎯→ ArOCOR + H2O
blisters when comes in contact with skin. It is poi- Phenyl esters are better prepared by action of acid
sonous and is good antiseptic (0·2%) and disinfectant chlorides or anhydrides on phenol.
(1%). Dilute solution of phenol is used for causteriz- –
:OH
ing wounds caused by mad dog bite. C6H5OH + C6H5COCl ⎯→ C6H5OCOC6H5 + HCl
● Phenol is stronger acid than alcohols since phen- Phenol Benzoyl chloride Phenyl benzoate
oxide ion is stabilised by resonance while alkoxide This is Benzoylation or Schotten-Baumann reac-
ion is not. Electron withdrawing groups increase the tion.
acidity while electron releasing groups decrease the –
:
OH
acidity of phenol. The increasing order of acidic C6H5OH + (CH3CO)2O ⎯→
nature is as— Phenol Acetic anhydride
OH OH OH C6H5OCOCH3 + CH3COOH
Phenyl acetate
< < This is acylation reaction.

CH3 NO2 Key Point


p-cresol Phenol p-nitrophenol ● Phenyl esters, when heated with anhy. AlCl 3‚ undergo
OH Fries rearrangement. The acyl group migrates to o- or p-
OH
NO2 position to form phenolic ketones.
O2N NO2
OCOCH OH
< < 3
Anhy. AlCl 3
⎯⎯⎯⎯→ COCH 3
NO2 NO2
2, 4-dinitrophenol Picric acid
Phenyl acetate o-hydroxyaceto
● Phenol itself is weaker acid than carbonic acid phenone
(H2CO3) and hence does not decompose NaHCO 3 to OH
give CO2. Instead phenols are recovered from
aqueous solution of phenoxides by bubbling CO2.
+
C6H5ONa + CO2 + H2O ⎯→ C6H5OH + NaHCO3
● Acid ionization constants (Ka) for some phenols are
COCH 3
listed below. Higher the value of K a higher is the p-hydroxyaceto
acidic nature. phenone

Name Ka Name Ka ● Formation of Ethers (Alkylation)


Phenol 1·1 × 10–10 o-aminophenol 2·0 × 10–10 Δ
o-cresol 0·63 × 10–10 m-aminophenol 69 × 10–10 C6H5ONa + CH3—I ⎯→ C6H5OCH3 + NaI
m-cresol 0·98 × 10–10 Methoxy benzene
o-nitrophenol 600 × 10–10
p-cresol 0·67 × 10–10 (Anisole)
m-nitrophenol 50 × 10–10
o-fluorophenol 15 × 10–10 Δ
m-fluorophenol 5·2 × 10–10 p-nitrophenol 690 × 10–10 C6H5ONa + C2H5 —I ⎯→ C6H5OC2H5
p-fluorophenol 1·1 × 10–10 Ethoxy benzene
o-chlorophenol 77 × 10–10 { 2‚ 4-dinitrophenol
2‚ 4‚ 6-trinitrophenol
1000‚000 × 10–10
Very large (Phenetole)
m-chlorophenol 16 × 10–10 This reaction is known as Williamson's synthesis. It
p-chlorophenol 6·3 × 10–10
is a nucleophilic substitution reaction.

C.S.V. / August / 2009 / 714


The reduction in presence of molybdenum oxide
Key Points
gives benzene.
● Aryl halides (ArX) are too inert to react with phenol to give MoO
C6H5OH + H2 ⎯⎯→ C6H6 + H2O
esters. However, at high temperature and in presence of
finely divided copper, the reaction is achieved. ● Reaction with phenyl isocyanate—Phenyl urethane
High temperature
C6H5ONa + BrC6H5 ⎯⎯⎯⎯⎯⎯→ is obtained as a colourless crystalline compound
Cu
which is used for characterisation of phenols.
C6H5OC6H5 + NaBr (Ullmann reaction)
Diphenyl ether C6H5OH + C6H5N — — C—— O → C6H5NH·COOC6H5
● The reaction of halogen substituted acids and halogen
Phenol Phenyl isocyanate Phenyl urethane
substituted phenol is important because the products are
Weedicides used in agriculture. ● Reactions of aromatic ring—The —OH group in
Cl phenol is o - and p -directing because it increases
electron density at o - and p -positions due to reso-
Cl OH + ClCH2COOH
nance. Thus phenol undergoes electrophilic substitu-
2, 4-dichlorophenol Chloroacetic acid tion reactions as—
Cl ➠ Halogenation—Like —NH 2 group, —OH group is so
⎯→ Cl O CH2CO OH much activating that it is rather difficult to prevent poly
substitution.
2, 4-dichlorophenoxy
acetic acid (2, 4-D), a weedicide OH OH
3Br ‚ H O Br Br
● Bucherer reaction—Phenolic group is replaced by 2 2
⎯⎯⎯→
amino group by heating phenol with ammonium
sulphite or bisulphite. Phenol
(NH4) 2SO 3/NH3 Br
C6H5OH ⎯⎯⎯⎯⎯→ C6H5NH2 2, 4, 6-tribromo phenol
150°C
Replacement of phenolic group by amino group can OH OH
also be achieved by heating phenol with double com-
pound of zinc chloride and ammonia at 300°C. 3Br water
2 Br Br
⎯⎯⎯⎯→
ZnCl2/NH3 OH
C6H5OH ⎯⎯⎯⎯→ C6H5NH2 + H2O OH
300°C
Note : This reaction is not used for preparation of ani- Resorcinol Br
2, 4, 6-tribromo resorcinol
line but finds extensive use in commercial pre-
paration of naphthyl amines from corresponding Note : If it is required to arrest the reaction at mono
naphthols. substitution stage, the reaction should be carried
● Reaction with PCl5 —Replacement of —OH group of out in non-polar solvents like CCl4 and CS 2 and
phenols by halogen atom is rather difficult. When at lower temperature.
treated with PCl 5 or PBr5 halo benzenes are obtained
in small amount and main product being triphenyl OH OH OH
phosphate. Br /CCl
2 4 Br
⎯⎯⎯→ +
(B) Reactions in which Phenols Differ from Alco- or CS2
hols Phenol
● Reaction with ferric chloride—Aqueous solution of Br
phenol on reacting with aqueous solution of FeCl 3 ➠ Sulphonation—This reaction is temperature depen-
gives violet coloured solution probably due to forma- dent. Low temperature favours o-sulphonation while
tion of hexa-coordinated complex. high temperature p-sulphonation.
6C 6H5OH + FeCl3 → [Fe(OC6H5)6] –3 + 3HCl + 3H+ OH
This reaction is used as qualitative test of phenolic
group. SO3 H
( o-phenol sulphonic
● Reaction with zinc—Phenol on distillation with zinc OH acid)
°
C

dust gives benzene.


15

C6H5OH + Zn → ZnO + C6H6 H2SO 4


● Reduction of phenol—When reduced with H2 in pre-
10

OH
°
0

sence of Ni (200 – 250°C) cyclohexanol is formed


C

which is used as a solvent in rubber industry. ( p-phenol sulphonic


OH HO H acid)
H H
Ni H SO3 H
+ 3H2 ⎯⎯⎯⎯→ H
200 – 250°C H H ➠ Nitration—(a) With dil. HNO 3 at 293 K, a mixture of
H H
Phenol 2-nitrophenol (major) and 4-nitrophenol (minor) is
H H
Cyclohexanol formed.

C.S.V. / August / 2009 / 715


OH
Key Point
(10%) ● Salicylic acid obtained by Kolbe’s-Schmidt reaction is a
very useful starting chemical for preparation of a number
OH of medicinal compounds.
Dil. HNO 3 NO 2 O CO CH3

20 °C COO H
OH (CH3CO)2O
Conc. H2SO 4 2-Acetoxy benzoic
N O2 acid (aspirin) analgesic
(50%) and steam volatile OH and antipyretic
OH
(b) Nitration of phenol with mixture of conc. HNO 3 CO O H C 6H 5 O H C O O C 6H 5
and H2SO4 gives a poor yield of 2, 4, 6-trinitro- POCl3
phenol (Picric acid). Poor yield is due to the Salicylic Phenol salicylate (Salol)
excessive oxidation side reaction. acid intestinal antiseptic

OH OH OH
CH 3OH COOCH3
HNO3-H2SO 4 NO2
⎯⎯⎯⎯→ NO2 Conc. H2SO4
Methyl salicylate (oil of
Phenol wintergreen)
NO2 Use in perfumery and
2, 4, 6-trinitrophenol flavouring agent in food,
(Picric acid) drinks and cosmetics.
Analgesic in rheumatic
OH OH and sciatica pains
HNO3-H2SO 4 NO2
⎯⎯⎯⎯→ NO2 ➠ Reimer-Tiemann reaction—(a) The treatment of
OH OH phenol with chloroform (CHCl 3) in presence of NaOH
Resorcinol NO2 at 340K followed by hydrolysis gives o-hydroxy ben-
Styphinic acid zaldehyde (salicylaldehyde) with small amount of p-
(c) The picric acid in good yield can be prepared by hydroxy benzaldehyde.
following reaction : OH OH OH
OH OH OH
CHCl /NaOH
3 CHO
⎯⎯⎯⎯→ +
Conc. H2SO 4 SO3 H 70°C/HCl
⎯⎯⎯⎯→ +
373 K Salicylaldehyde CHO
Phenol (Main) p-hydro-
SO3 H xybenzaldehyde
OH (Minor)

NO2 NO2 Note : o -isomer, due to chelation (intramolecular H-


Conc. HNO3
⎯⎯⎯⎯→ bonding) is more volatile than p -isomer (inter-
molecular H-bonding) and can be separated by
NO2 steam distillation.
Picric acid (b) By using carbon tetrachloride in place of chloro-
➠ Friedel-Crafts alkylation reaction—When phenols form a mixture of o-hydroxy and p-hydroxy
are treated with alkyl halides in presence of anhy. benzoic acids is formed.
AlCl3, alkyl phenols are formed. OH ONa OH
OH OH OH CCl4/NaOH COO Na Dil. HCl COOH
Anhy. AlCl3 ⎯⎯⎯→ ⎯⎯→
CH3 + 70°C —NaCl
+ CH3Cl ⎯⎯⎯⎯→
Phenol Salicylic acid
o-cresol ➠ Reaction with formaldehyde—Under different con-
CH3 ditions, different products are formed.
p-cresol
➠ Kolbe-Schmidt reaction— (i) Phenol when treated with formalin in presence of
a dil. acid or alkali at low temperature, p -
ONa OH
hydroxy benzyl alcohol along with small quantity
150°C COONa of o-isomer is formed. This is known as Lederer
+ CO2 ⎯⎯→ Manasse reaction.
5-7 atm
Sod. phenoxide Sod. salicylate OH OH OH
OH NaOH C H 2OH
+ HCHO ⎯⎯→ +
5 days
Dil. HCl COOH
⎯⎯⎯→
—NaCl C H 2OH
Salicylic acid (Main product)

C.S.V. / August / 2009 / 716


(ii) Phenol on condensation with insufficient NaNO /H SO
2 2 4
amount of HCHO in presence of an acid catalyst, HO ⎯⎯⎯⎯⎯→
gives a linear polymer called novolak.
Tautomerises
OH OH HO N—
—O O—
— —
— NOH
HCHO CH 2OH
⎯⎯→ p-nitrosophenol Quinonemonoxime
Conc.
OH OH H 2SO 4 C6H5OH

C6H5OH CH2 ⎡⎢ ⎤
⎯⎯→
⎢⎣ HO N—
— — OH+ ⎥

⎥ HSO 4

OH OH Phenol indophenol hydrogen sulphate
(Deep blue)
HCHO CH2 C H 2OH
⎯⎯→ H 2O

(iii) In presence of excess of formaldehyde and using


a basic catalyst like ammonia or strong aqueous HO N—
— —
—O
alkali at higher temperature, a different kind of
Phenol indophenol
condensation occurs. Two kinds of products are (Red)
first formed
NaOH
OH OH
⎡⎢ ⎤
— O⎥
C H 2OH
(a) 2 + 2HCHO ⎯→ + ⎢⎣ –O N—
— —
⎥ Na+

Sodium salt of phenol indophenol
C H 2OH (Deep blue)
OH ➠ Phthalein reaction—When phenols are heated with
HOCH2 CH2OH phthalic anhydride in presence of conc. H2SO4 or
anhydrous zinc chloride, phthaleins, with characteri-
stic colours, depending on the pH, are formed.
Bis (hydroxymethyl) phenols HO OH
OH

(b) 2 + 2HCHO Phenol Phenol


H H
O
Anhy. ZnCl
2
⎯→ HO CH2 OH ⎯⎯⎯⎯→
C –H O 2

p, p′-dihydroxydiphenyl methane O
These products undergo slow polymerisation to give
C
a resin called Bakelite, a three dimensional polymer.
O
Phthalic anhydride
CH2
OH HO OH
CH2 CH2 OH

CH2 C
CH2
O
HO CH2
C
CH2
O
Phenolphthalein
➠ Liebermann’s reaction—When a crystal of phenol is Phthalein reaction is a diagnostic test for phenols.
added to a mixture of conc. H 2SO4 and sodium nitrite Phenolphthalein is used an important indicator in
and reaction mixture is warmed, a characteristic blue acid-base titrations.
or green colour is obtained. On dilution with water When treated with alkali, it undergoes an interesting
colour becomes red but again turns deep blue on colour change. It turns red first but on addition of
adding excess of caustic soda. The chemistry of this excess of alkali it turns colourless again. These
reaction is of diagnostic value— changes are shown as—

C.S.V. / August / 2009 / 717


HO OH products is uncertain. However, the following
sequence of reactions is predicted.
OH O
||
C O
NaOH ⎯→
2
+ H 2O
O ⎯⎯→
||
C O
Quinone
O Quinone forms brilliant red addition product with
Phenolphthalein phenol.
(Colourless)
– + C6H5OH + O —
— —
—O
HO O Na

⎯→ OH… O —
— —
— O… HO

C Phenoquinone (Red)
(b) With potassium permanganate :
O
CH(OH)COOH
KMnO 4
C C6H5OH ⎯⎯⎯→ | + 2CO2
[O] CH(OH)·COOH
Phenol
O Mesotartaric
Colourless acid
(c) With a mixture of KClO3 and conc. HCl.
OH O
HO O
KClO Cl Cl
⎯⎯→ 3
HCl Cl Cl
C Phenol
O
– + Chloranil
O Na or Tetrachloroquinone
(d) With alkaline solution of potassium persulphate.
C
OH OH
O
K S O
NaOH ⎯⎯⎯→
2 2 8
Alkaline

+ – OH
NaO O Hydroquinone or quinol
➠ Gattermann’s aldehyde synthesis :
OH
C
– Anhy. AlCl
: OH + HCN + HCl ⎯⎯⎯⎯→
3
Gas
– +
COONa OH OH
(Red colour) H 2O
⎯→ + NH3
NaOH CH —
— NH CHO
o-hydroxy benzaldehyde
+ – – +
NaO ONa Small quantity of p-hydroxy benzaldehyde is also for-
med.

Points to Remember
C
● Phenol is also known as carbolic acid. It is used as a disin-
fectant in carbolic soaps and lotions and as a preservative
OH in inks.
– +
COONa ● Bakelite is a phenol-formaldehyde thermosetting polymer
which is generally used as an insulator for electrical work.
Trisodium salt
(Colourless) ● 2, 4, 6-Trinitrophenol (Picric acid) is used in dyeing silk and
wool and also used in preparation of explosives. It is effective
Similarly resorcinol and phthalic anhydride give fluo- in treatment of burns.
rescein, which dissolves in dilute alkali to produce ● Phenol finds extensive use in preparation of drugs like,
intense green fluorescence. salicylic acid, aspirin, salol, phenacetin etc.
● The reaction of phenol and phthalic anhydride leads to the
➠ Oxidation—(a) Phenols are much susceptible to formation of phenolphthalein which is used as an indicator
atmospheric oxidation than alcohols, but the nature of in volumetric analysis and as purgative in medicines.

C.S.V. / August / 2009 / 718


● Phenyl salicylate (salol) is used in toothpastes because it is ● Hydroxy toluenes (cresols) are used in preservation of timber
under the name of creosote oil. Lysol is an emulsion of
a good antiseptic. crude cresols in soap solution.
● Methyl salicylate is main constituent of oil of wintergreen. ● Ehrlich, the founder of chemotherapy discovered Salvarsan
It is obtained by direct esterification of salicylic acid with or 606. It is an effective drug for treatment of trypano-
CH 3OH (Fischer-Speter’s method). It is used in preparation of soniasis. Hydrochloride of salvarsan is used in treatment of
iodex. syphilis and it is also an effective antimalarial. It is prepared
from phenol.
● Certain phenols like eugenol (clove oil), isoeugenol (nutmeg
oil), thymol (thyme and mint oil), vanillin (vanila beans) are NH 2 NH2
found in nature.
HO OH
● Decreasing order of acidic nature of some derivatives of
phenol is as : As = As
(i) 2, 4-dinitrophenol > p -nitrophenol > p-cresol > m-amino- Salvarsan (606)
phenol. ● Neosalvarsan has been found more useful as it is readily
water soluble. It can be used for injection purpose.
(ii) Phenol > p-cresol > m-cresol > o-cresol
● 3, 4, 5-trihydroxy benzoic acid (gallic acid) is present in tea NH2 NHCH2 OSO 2 Na
and roots of pomegranate. In combined state it is present in
gall nuts, oak bark and acacia bark. At m.p. it gives pyro- HO OH
gallol. As = As
OH OH Neosalvarsan
● 1, 2, 3-trihydroxy benzene (Pyrogallol) is used in hair dye to
HO Δ HO convert grey hair to black. In gas analysis it is also used for
⎯→ + CO2
HO COOH HO absorbing unpleasant gases.
● Levodopa, 3-(3, 4-dihydroxy phenyl)-L-alanine is a drug
Gallic acid Pyrogallol
used in treatment of perkinsonism.
Gallic acid on exposure to air turns brown to black, thus used
in the manufacture of blue-black ink. Bismuth gallate, under NH2
the name of dermatol is used in treatment of skin infection. |
HO CH2— CH—COO H
● o -hydroxy benzoic acid (salicylic acid) is used in the treat- Levodopa
ment of eczema and other skin diseases and rheumatic
HO
pains.
● Paracetamidophenol (para-acetamol) is actually hydroxy
● Claisen-rearrangement—When allylphenyl ether is heated derivative of acetanilide and is a main antipyretic.
at 573 K, it undergoes rearrangement as
O–CH2—CH = C H2 OH HO NHCOCH3

573 K CH2—CH = C H2
⎯⎯→ Paracetamol
● Phenacetin (acetophenetidine) is ethyl ether of paracetamol.
Allylphenyl ether o-allylphenol
● Elbs-persulphate oxidation : C2 H5 O NHCOCH3

OH OH OH Phenacetin
K 2S2O8‚ KOH OH It is widely used as an analgesic and antipyretic, usually in
⎯⎯⎯⎯→ + combination with aspirin, caffeine and codeine.
● Acid strength of phenol (C6 H 5 OH) with respect to other
Phenol Catechol (Minor) compounds is in following decreasing order :
OH
Hydroquinone or quinol R—COOH > H 2CO3 > C6H5OH > H2O > R—OH
(Major)

OBJECTIVE QUESTIONS
1. Phenol is acidic in nature and it 3. The fusion of sodium benzene 5. Salicylic acid is—
can react with— sulphonate with NaOH, followed (A) m-hydroxybenzoic acid
(A) Sodium bicarbonate by acid hydrolysis, gives— (B) p-hydroxybenzoic acid
(B) Potassium carbonate (A) Benzene (C) o-hydroxy benzoic acid
(C) Sodium hydroxide (B) Benzoic acid (D) All of these
(D) All of these (C) Phenol
6. Intramolecular hydrogen bonding
(D) Sodium salicylate
2. Which of the following com- is present in—
pounds does not contain a 4. Which does not react with phenol (A) Phenol
carboxylic group ? (C6H5OH) ? (B) p-nitrophenol
(A) Anisole (A) Sodium (C) o-nitrophenol
(B) Picric acid (B) Caustic soda (D) m-nitrophenol
(C) Carbolic acid (C) Caustic potash 7. Which of the following com-
(D) All of these (D) Washing soda pounds is the most acidic ?

C.S.V. / August / 2009 / 719


(A) p-nitrophenol (B) Phenyl- n-propane (B) Benzene, dinitrobenzene, m -
(B) p-chlorophenol (C) Isopropyl benzene nitroaniline
(C) p-aminophenol (D) 2, 4-dimethylbenzene (C) Benzene, nitrobenzene,
(D) o-cresol hydrazobenzene
15. Which of the following com-
(D) Toluene, m -nitrobenzene,
8. Which of the following com- pounds is attacked by an electro-
m-toluedine
pounds is formed when phenol phile most easily ?
reacts with PCl5 ? (A) Toluene 22. Phenol is less acidic than—
(A) Chlorophenol (B) Phenol (A) p-methoxyphenol
(B) Chlorobenzene (C) Benzene (B) p-nitrophenol
(C) Triphenyl phosphate (D) Chlorobenzene (C) Carbonic acid
(D) Both (B) and (C) (D) Both (B) and (C)
16. When phenol is distilled with zinc
9. Which of the following reactions dust, the product is— 23. Reimer-Tiemann reaction invol-
does not give phenol or sodium (A) Toluene ves—
phenoxide ? (A) Carbonium ion intermediate
NaOH (B) Benzene
(A) C6H5COCl ⎯⎯→ (B) Carbanion intermediate
(C) Benzoic acid
Alc. KOH
(B) C6H5N2Cl ⎯⎯→ (D) Zinc phenoxide (C) Carbene intermediate
Aqu. KOH
(C) C6H5N2Cl ⎯⎯→ 17. Which compound is formed when
(D) Free radical intermediate
H O 24. C6H5OH + ClCOCH3
(D) C6H5NNCl ⎯⎯→
2 sodium phenoxide is heated with
Δ aq. NaOH
ethyl iodide ? ⎯⎯⎯→ C6H5OCOCH3
10. Which of the following reactions (A) Ethyl phenyl alcohol is called—
converts phenol into salicylic
(B) Phenetole (A) Kolbe reaction
acid ?
(A) Kolbe’s reaction (C) Phenol (B) Reimer-Tiemann reaction
(B) Etard’s reaction (D) None of these (C) Schotten-Baumann reaction
(C) Reimer-Tiemann reaction 18. Which of the following com- (D) Dow’s reaction
(D) Dow’s reaction pounds undergoes nitration most
readily ? 25. Sodium phenoxide when heated
11. A compound which is useful in with CO 2 under pressure, gives—
(A) Benzoic acid
treatment of burns is— (A) Sodium benzoate
(B) Toluene
(A) Creosote oil (B) Benzoic acid
(C) Nitrobenzene
(B) Picric acid (C) Phenol
(D) Phenol
(C) 2, 4, 6-trinitro resorcinol (D) o-hydroxy benzoic acid
(D) Acetyl salicylic acid 19. Cyclohexanol is a—
(A) Primary alcohol 26. Which product is formed when
12. In the reaction sequence phenol reacts with benzene
CCl HCl (B) Secondary alcohol diazonium chloride ?
Phenol⎯⎯→
4
(A) ⎯⎯→ (B)
NaOH (C) Phenol (A) Phenyl hydroxylamine
Sodalime
⎯⎯⎯→ (C) (D) Tertiary alcohol (B) Para aminoazobenzene
Product (C) is— 20. When phenol is treated with (C) Phenyl hydrazine
(A) Sodium salicylate excess of bromine water, which
(D) Para hydroxyazobenzene
(B) Benzoic acid of the following compounds is
formed ? 27. Phenol on reacting with dil.
(C) Phenol HNO3 at room temperature
(A) m-bromophenol
(D) Salicylic acid gives—
(B) o- and p-bromophenol
13. Aspirin is an acetylation product (C) 2, 4, 6-tribromophenol (A) Picric acid
of— (D) All of these (B) m-nitrophenol
(A) m-hydroxybenzoic acid (C) o - and p-nitrophenol
21. In the reaction sequence
(B) p-hydroxybenzoic acid (D) All of these
Zn
(C) o-hydroxybenzoic acid Phenol ⎯⎯⎯→ (X)
Distillation 28. Phenol and ethanol can be dis-
(D) Phenol Conc. H2SO 4 Zn
⎯⎯⎯⎯⎯→ (Y) ⎯⎯⎯→ (Z) tinguished by using—
14. Cumene on oxidation in pre- conc. HNO3‚ 60°C NaOH
(A) Caustic soda
sence of metal catalyst followed The product (X), (Y) and (Z)
(B) Anhydrous AlCl3
by treatment with acid gives respectively are—
phenol. The cumene is— (C) FeCl3
(A) Benzene, nitrobenzene, ani-
(A) o-cresol line (D) All of these

C.S.V. / August / 2009 / 720


29. A diazonium salt reacts with (C) Salicylic acid 43. Correct increasing order of acidic
phenol to give an azo dye. This (D) Cyclohexanol nature is—
reaction is termed as— (A) H2O < H2C2 < H2CO3
37. Which statement regarding
(A) Diazotization C6H5OH is correct ? < C6H5OH
(B) Coupling (B) C2H2 < H2O < C6H5OH
(A) It is a carboxylic acid
(C) Both (A) and (B) < H2CO3
(B) It is insoluble in water
(D) None of these
(C) It has higher b.p. than (C) C6H5OH < C2H2 < H 2CO3
30. The Dow process is used for toluene < H2O
conversion of chlorobenzene (D) None of these (D) H2C2 < H 2O < H 2CO3
into—
38. Phenol is a— < C6H5OH
(A) Benzene
(A) Carboxylic acid
(B) Nitrophenol 44. Which of the following will not
(B) Base form phenol or phenoxide ?
(C) Phenol
(C) Alcohol (A) C6H5SO3Na
(D) Chlorophenol (D) None of these
(B) C6H5N2Cl
31. Phenol on reacting with HNO 3 or 39. Which compound gives purple
H2SO4— (C) C6H5COOH
colour with neutral ferric chloride
(D) C6H5Cl
(A) Forms ester solution ?
(B) Gives ether (A) Boric acid 45. Phenol dissolves in aqueous
(C) Does not give ester (B) Phenol NaOH to give sodium phenate,
but on passing CO2 into solution,
(D) Forms toluene (C) Aniline
phenol is thrown out. This
32. Carbolic acid is— (D) Benzoic acid
indicates that—
(A) C6H5OH 40. Phenol is most easily soluble (A) Phenol is weakly acidic, but
(B) H2CO3 in— stronger than H2CO3
(C) HCOOH (A) NaHCO3 solution (B) Phenol is basic
(D) None of these (B) NaOH solution (C) Phenol is amphoteric
(C) Dil. HCl (D) Phenol is weakly acidic
33. Main component of oil of winter
(D) None of these weaker than H2CO3
green is—
(A) Salicylic acid 41. The reaction 46. If salicylic acid is heated with
(B) Methyl salicylate zinc dust, the product formed will
OH
be—
(C) Phenyl salicylate CHO
(A) Phenol
(D) Ethyl salicylate
H2O2 / OH– OH (B) Benzene
34. Phenol on heating at 60°C with a ⎯⎯⎯→
OH (C) Benzoic acid
mixture of chloroform and excess
is known as— (D) Zinc benzoate
of NaOH and on subsequent
acidification gives— (A) Reimer-Tiemann reaction 47. When salicylic acid is treated
(A) 2-hydroxybenzoic acid (B) Liebermann’s nitroso reac- with bromine water, the product
tion formed is—
(B) 2-hydroxy benzaldehyde
(C) Dakin reaction (A) 2, 3, 4-tribromophenol
(C) 3-hydroxybenzoic acid
(D) Lederer-Manasse reaction (B) 3, 4, 5-tribromosalicylic acid
(D) 3-hydroxy benzaldehyde
(C) 3, 6-dibromosalicylic acid
35. Which of the following com- 42. Which will undergo Friedel-Crafts
alkylation reaction ? (D) 2, 4, 6-tribromophenol
pounds gives violet colour with
neutral solution of FeCl 3 ? OH CH3 COOH 48. Chemically salol is—
(A) Benzoic acid (A) Sodium salicylate
(B) Formic acid (B) Acetyl salicylic acid
(C) Salicylic acid (C) Phenyl salicylate
N O2
(D) Acetic acid (D) Methyl salicylate
(I) (II) (III)
36. When phenol reacts with CHCl 3 C H 2C H 3 49. Which of the following properties
in presence of NaOH, salicyl- of benzoic acid and phenol are
aldehyde is formed, if we use similar ?
pyrene in place of CHCl3, the (A) Both are weak acids and do
product is— (IV) not react with NaHCO 3
(A) Salicylaldehyde (A) I and IV (B) I, II, III (B) Both are equally acidic in
(B) Phenolphthalein (C) III and II (D) II and IV nature

C.S.V. / August / 2009 / 721


(C) Both liberate hydrogen on (C) Sodium phenate + CH3I (Continued from Page 708 )
reacting with metallic sodium ⎯→ Anisol
–HI
(D) Both form esters (D) None of these
50. The electrophilic substitution in 56. Phenol on reacting with bromine
phenol takes place at— dissolved in carbon disulphide
(A) Orthoposition (CS2), gives—
(B) Metaposition (A) m-bromophenol only
(C) Paraposition (B) o-bromophenol
(D) Both ortho and para posi- (C) Both o- and p-bromophenol
tions (D) 2, 4, 6-tribromophenol

51. Anisole is the reaction product of 57. When phenetole reacts with HI,
phenol and dimethyl sulphate. which is formed ?
This reaction is termed as— (A) C6H5OH and C2H5I
(A) Coupling (B) C2H5OH and C6H5OH
(B) Esterification (C) C2H5OH and iodobenzene

(C) Etherification (D) Ethane and benzene

(D) None of these 58. Which is not true statement ?


(A) Paracetamol is parahydroxy
52. Which of the following reactions
derivative of acetanilide
is given by phenol ?
(B) Salol is phenyl salicylate
(A) Williamson's reaction
(C) Phenyl esters on heating
(B) Coupling reaction with AlCl3 give cresols
(C) Lederer-Manasse reaction (D) Benzoylation of phenol gives
(D) All of these phenyl benzoate

53. The reaction 59. Phenol gives novolak on react-


OH ing with—
(A) CH3CHO in presence of acid
OH– or
+ HCHO ⎯⎯→
+
at low temp.
H (B) HCHO in presence of acid at
OH OH low temp.
(C) HCOOH in presence of
C H 2O H + NaOH at low temp.
(D) Formalin in presence of
C H 2O H alkali at high temp.
is called— 60. The amount of bromine required
(A) Reimer-Tiemann reaction to convert 1·0 g of phenol into
2, 4, 6-tribromophenol, is—
(B) Sand Meyer reaction
(A) 2·5 g (B) 5·1 g
(C) Lederer-Manasse reaction (C) 10·22 g (D) 15·3 g
(D) Kolbe reaction
ANSWERS
54. A compound ‘A’ when treated
with CH 3OH and a few drops of
H2SO4, gave a smell of winter
green. The compound ‘A’ is—
(A) Succinic acid
(B) Phenol
(C) Salicylic acid
(D) Salicylaldehyde
55. Schotten-Baumann reaction is—
NaOH
(A) Phenol + CHCl3 ⎯⎯→
Salicylaldehyde
(B) Phenol + Benzoyl chloride
NaOH
⎯⎯→ Phenyl benzoate ●●● ●●●

C.S.V. / August / 2009 / 722


16. At 25°C, the dissociation constant
of a base, BOH, is 1 × 10–12. The
concentration of hydroxyl ions in
0·01 M aqueous solution of the
base would be—
(A) 1·0 × 10– 5 mol L– 1
1. 60 gm of a compound on ana- 10. The number of moles of KMnO 4
(B) 1·0 × 10– 6 mol L– 1
lysis produce 24 gm carbon, 4 gm reduced by one mole of KI in
hydrogen and 32 gm oxygen. The alkaline medium is— (C) 2·0 × 10– 6 mol L– 1
empirical formula of the com- (A) One (B) Two (D) 1·0 × 10– 7 mol L– 1
pound is—
(C) Five (D) One fifth
(A) CH2O2 (B) CH2O 17. The function of moderator in a
(C) H2 (D) CH4 11. 2 Ag + 2 H2SO4 → Ag2SO4 nuclear reactor is—
+ 2 H2O + SO2 (A) To produce more neutrons
2. The energy of second Bohr orbit
of the hydrogen atom is – 328 kJ In the above reaction H2SO4 acts (B) To stop nuclear reaction
mol– 1; hence the energy of fourth as a— (C) To increase speed of neu-
Bohr orbit would be— (A) Catalyst tron
(A) – 41 kJ mol– 1 (B) Oxidising agent (D) To slow down speed of neu-
(B) – 82 kJ mol– 1 (C) Reducing agent tron
(C) – 164 kJ mol– 1
(D) Acid as well as oxidant 18. For a first order reaction A → B
(D) – 1312 kJ mol– 1 the reaction rate at reactant con-
12. A solution has a 1 : 4 mole ratio
3. The number of neutrons in the centration of 0·01 M is found to
element 4Be9 is— of pentane to hexane. The vapour
be 2·0 × 10 – 5 mol L– 1 s– 1. The
(A) 3 (B) 5 pressures of the pure hydro-
half -life period of the reaction is—
(C) 7 (D) 9 carbons at 20°C are 440 mm Hg
for pentane and 120 mm Hg for (A) 30 s (B) 220 s
4. Which of the following is the (C) 300 s (D) 347 s
hexane. The mole fraction of
electron deficient molecule ?
(A) C2H6 (B) B2H6 pentane in the vapour phase 19. A colloidal solution is subjected
(C) SiH4 (D) PH 3 would be— to an electric field. The colloidal
(A) 0·200 (B) 0·549 particles move towards anode.
5. 8·2 litre of an ideal gas weight
9·0 gm at 300 K and 1 atm pres- (C) 0·786 (D) 0·478 The coagulation of the same
sure. The molecular mass of the solution studied using NaCl,
13. N2(g) + 3H2(g) → 2 NH3(g) BaCl 2 and AlCl 3 solutions. The
gas is—
(A) 9 (B) 18 For the above reaction the coagulation power is in the order
correct statement is— of—
(C) 25 (D) 36
(A) ΔH < ΔE (B) ΔH = ΔE (A) BaCl 2 > NaCl > AlCl3
6. The correct order in which the
O–O bond length increases in the (C) ΔH > ΔE (D) None of these (B) NaCl > BaCl2 > AlCl3
following is— 14. Equilibrium constants K1 and K2 (C) AlCl3 > BaCl2 > NaCl
(A) O2 < H2O2 < O3 for the following equilibria : (D) NaCl > AlCl3 > BaCl2
(B) O3 < H2O2 < O2 1 K1
NO(g) + O 2 NO2(g) 20. A reaction occurs spontaneously
(C) H2O2 < O2 < O3 2 if—
(D) O2 < O3 < H2O2 K2
and 2NO 2(g) 2NO(g) + O2(g) (A) TΔS < ΔH and both ΔH and
7. Both ionic and covalent bond is ΔS are + ve
are related as—
present in— (B) TΔS > ΔH and ΔH is + ve and
(A) KCN (B) KCl 1 ΔS is – ve
(A) K2 = 2 (B) K2 = K12
(C) H2 (D) CH4 K1 (C) TΔS > ΔH and both ΔH and
1 K1 ΔS are + ve
8. Which of the following molecules (C) K2 = (D) K2 =
has trigonal planar geometry ? K1 2 (D) TΔS = ΔH and both ΔH and
ΔS are + ve
(A) BF 3 (B) NH3 15. Variation of heat of reaction with
(C) PCl3 (D) IF3 temperature is known as— 21. The pH of the solution obtained
by mixing 40 ml of 0·10 M HCl
(A) Kirchhoff’s equation
9. The pH of pure water at 80° will with 10 ml of 0·45 M of NaOH
be— (B) van’t Hoff’s isotherm is—
(A) > 7 (B) < 7 (C) van’t Hoff’s isochore (A) 4 (B) 8
(C) = 7 (D) None of these (D) None of the above (C) 12 (D) 14

C.S.V. / August / 2009 / 723


22. The correct order of acid strength (C) CH3—CH2—C⎯ CH—CH3 (C) Phosphate group
is— || | (D) Deoxyribose group
(A) HClO4 < HClO3 < HClO2 CH2 CH3
2-ethyl-3-methyl-but-1-ene 36. The best method for the separa-
< HClO tion of naphthalene and benzoic
(D) CH3—C ≡ C—CH—(CH3)2
(B) HClO < HClO2 < HClO3 acid from their mixture is—
4-methyl-2-pentyne
< HClO4 (A) Distillation
29. C3H9N cannot represent—
(C) HClO4 < HClO < HClO2 (B) Sublimation
< HClO3 (A) 1° amine
(C) Chromatography
(D) HClO2 < HClO3 < HClO4 (B) 2° amine (D) Crystallisation
< HClO (C) 3° amine 37. The first ionisation potential of
23. 50 ml of H2 SO4 require 10 gm (D) Quaternary ammonium salt Na, Mg, Al and Si are in the
CaCO3 for complete decomposi- order—
30. Which one of the following pairs
tion. The normality of acid is— represents stereoisomerism ? (A) Na < Mg < Al < Si
(A) 2 (B) 0·30 (B) Na < Al < Mg < Si
(A) Structural isomerism and
(C) 4 (D) 0·20 (C) Na > Mg > Al > Si
geometrical isomerism
24. A solution of urea (mol. mass 56 (B) Optical isomerism and geo- (D) Na < Si < Al < Mg
g mol– 1 ) boils at 100·18 °C at the metrical isomerism 38. The major organic product formed
atmospheric pressure. If Kf and (C) Chain isomerism and rota- from the following reaction :
Kb for water are 1·86 and 0·512 K tional isomerism (i) CH3NH2
kg mol– 1 respectively, the above O (ii) LiAlH
solution will freeze at—
(D) Linkage isomerism and geo- ⎯⎯⎯→
4

metrical isomerism (iii) H 2O
(A) 0·654°C (B) – 0·654°C is—
31. The main product obtained from
(C) 6·54°C (D) – 6·54°C H
phenol with PCl 5 is— NCH3
25. The isomerism represented by (A) BHC (A)
OH
ethyl acetoacetate is— (B) Hexachlorobenzene H
(A) Keto-enol isomerism NCH3
(C) Chlorobenzene (B)
(B) Geometrical isomerism
(D) Triphenyl phosphate OH
(C) Enantiomerism
O—NHCH3
(D) Diastereoisomerism 32. Which amongst the following is (C)
26. Which one of the following forms the most stable carbocation ?
+ +
micelles in aqueous solution (A) CH3 (B) CH3CH2 (D)
above certain concentration ? NCH 3
CH3 H
(A) Dodecyl trimethyl ammo- + | 39. Which is not present in chloro-
nium chloride (C) CH3—CH (D) CH3—C+
phyll ?
(B) Glucose | |
CH3 CH3 (A) Carbon (B) Calcium
(C) Urea
(C) Magnesium (D) Hydrogen
(D) Pyridinium chloride 33. Hydrolysis of sucrose is—
40. Which functional group partici-
27. The compound known as oil of (A) Inversion pates in disulphide bond forma-
wintergreen is— (B) Hydration tion in proteins ?
(A) Phenyl acetate (C) Saponification (A) Thioester (B) Thioether
(B) Phenyl salicylate (D) Inhibition (C) Thiol (D) Thioacetone
(C) Methyl salicylate 41. The molecular formula of cryolite
(D) Methyl acetate 34. Products of the following reac-
is—
tion—
28. Names of some compounds are (A) Na 2F.AlF6 (B) Na 3AlF6
(i) O 3
given. Which one is not the CH3–C ≡ C–CH 2–CH3 ⎯⎯⎯⎯⎯→ (C) Na 3AlF5 (D) Na 2AlF3
correct IUPAC name ? (ii) Hydrolysis
42. The monomer of the polymer;
(A) CH3—CH2—CH2 are— CH3
CH3 (A) CH3COOH + CO2 | CH3
| (B) CH3COOH + HOOC–CH2CH3 CH2—C—CH2—C⊕
—CH—CH—CH2CH3 | CH3
(C) CH3CHO + CH3CH2CHO
| CH3
CH2CH3 (D) CH3COOH + CH3COCH3
is—
3-methyl-4-ethyl heptane
35. The pairs of base in DNA are CH3
(B) CH3—CH —CH —CH3 held together by— (A) H2C = C
| |
(A) Ionic bonds CH3
OH CH3
3-methyl-2-butanol (B) Hydrogen bond (B) CH3CH = CHCH3

C.S.V. / August / 2009 / 724


(C) CH3CH = CH2 by the actinoids than the corres- (C) [Co(NH3)6] 3+
(D) (CH3)2C = C(CH3)2 ponding lanthanoids, is—
(D) [Ni (NH3)6] 2+
(A) More energy difference bet-
43. Aluminium appears like gold ween 5f and 6d orbitals than 49. The most acidic oxide is—
when mixed with— between 4 f and 5d orbitals (A) P2O5 (B) N2O5
(A) 90% Cu (B) 75% Ni (B) Lesser energy difference bet- (C) Sb2O5 (D) As 2O5
(C) 80% Sn (D) 80% Co ween 5f and 6d orbitals than
between 4 f and 5d orbitals 50. H2S gas when passed through a
44. The aqueous solution containing (C) Larger atomic size of acti- solution of cations containing HCl
which one of the following ions noids than the lanthanoids precipitates the cations of second
will be colourless ? (D) Greater reactive nature of the group of qualitative analysis but
(Atomic No. : Sc = 21, Fe = 26, actinoids than the lantha- not those belonging to the fourth
Ti = 22, Mn = 25) noids group. It is because—
(A) Sc 3+ (B) Fe 2+ 47. By passing air over red hot coke (A) Presence of HCl decreases
(C) Ti 3+ (D) Mn2+ the gas obtained is— the sulphide ion concentra-
(A) Coal gas (B) Water gas tion
45. Lithophone is a mixture of— (C) Oil gas (D) Producer gas (B) Solubility product of group II
(A) CuSO 4 + ZnS sulphides is more than that of
48. Which one of the following is an
(B) CaSO 4 + ZnS inner orbital complex as well as group IV sulphides
(C) BaSO4 + CaSO4 diamagnetic in behaviour ? (C) Presence of HCl increases
(Atomic number : Zn = 30, the sulphide ion concentra-
(D) BaSO4 + ZnS Cr = 24, Co = 27, Ni = 28) tion
46. The main reason for larger num- (A) [Zn (NH3)6] 2+ (D) Sulphides of group IV cations
ber of oxidation states exhibited (B) [Cr (NH3)6] 3+ are unstable in HCl

ANSWERS WITH HINTS

C.S.V. / August / 2009 / 725


(C) A decrease in enthalpy
(D) A decrease in internal
energy
12. Fog is a colloidal system in
which—
(A) Liquid particles dispersed in
1. Two solutions (a) containing (C) m
Z
A → m–4
Z–2
B + 24He gas
FeCl3 (aq) and (b) containing
A and B are isodiapheres (B) Gaseous particles disper-
K4[Fe (CN)6] are separated by
(D) All correct sed in liquid
semipermeable membrane as
shown below. If FeCl3 on reaction (C) Solid particles dispersed in
7. Which curve represents zero
with K4 [Fe (CN) 6] produces blue liquid
order reaction ?
colour of— (D) Solid particles dispersed in
gas
…………

(a) (b) ↑
……………………………… (A) [A] 13. Which arrangement of electrons
FeCl 3 K 4 [Fe(CN) 6] lead to anti-ferromagnetism ?
t →
5 pm (A) ↑↑↑↑
Fe 4[Fe(CN)6], the blue colour will ↑ (B) ↑↓↑↓
be noticed in— (B) [A] (C) Both (A) and (B)
(A) (a) t → (D) None of these
(B) (b)
14. A particular reaction at 27°C for
(C) In both (A) and (B) ↑
which ΔH > 0 and ΔS > 0 is found
(D) Neither in (A) nor in (B) (C) [A]
to be nonspontaneous. The reac-
2. Volume of a mixture of 6·02 × 1023 tion may proceed spontaneously
t →
oxygen atoms and 3·01 × 10 23 if—
hydrogen molecules at STP is— ↑ (A) Temperature is decreased
(A) 28·0 litre (B) 33·6 litre (D) [A] (B) Temperature is kept constant
(C) 11·2 litre (D) 22·4 litre (C) Temperature is raised
t →
3. The molal freezing point constant (D) It is carried out in open
of water is 1·86 K molality– 1. If 8. If one mole of monoatomic gas vessel at 27°C
342 g of cane-sugar (C 12H22O11)
are dissolved in 1000 g of water,
( )γ=
5
3
is mixed with one mole 15. The correct order of relative sta-
bility of half filled and completely
the solution will freeze at—
(A) – 1·86°C (B) 1·86°C
of diatomic gas γ=( )
7
5
the filled shells is—
(A) p 3 < d 5 < d10 < p6
(C) – 3·92°C (D) 2·42°C value of γ for the mixture is—
(B) d 5 < p3 < d10 < p6
4. The speed of electron in the first (A) 1·40 (B) 1·50
(C) d 5 < p3 < d10 < p6
orbit of hydrogen atom in the (C) 1·53 (D) 3·07
(D) p3 < d10 < d 5 < p6
ground state is— 9. A certain weak acid has a disso-
[ c = velocity of light] 16. For the reaction A + B → C + D
ciation constant 1·0 × 10 – 4 . The
c c the variation of the concentration
(A) (B) equilibrium constant for its reac-
1·37 1370 of product is given by the curve—
c c tion with a strong base is—
(C) (D) Y
13·7 137 (A) 1·0 × 10– 4 (B) 1·0 × 10–10
↑ Z
Concentration

5. ZSM –5 is used to convert— (C) 1 × 1010 (D) 1·0 × 10–14


(A) Alcohol to petrol W
10. If glycerol and methanol were
(B) Benzene to toluene
sold at same price in the market,
(C) Toluene to benzene
which would be cheaper for pre-
(D) Heptane to toluene X
paring an artifreezer solution for
6. Which among the following is the radiator of an automobile ? Time →
wrong about isodiapheres ? (A) Glycerol (A) X (B) Y
(A) They have the same diffe- (B) Methanol (C) Z (D) W
rence of neutrons and
(C) Both equal 17. Water glass is—
protons or same isotopic
number (D) None of these (A) Another name for sodium
(B) Nucleide and its decay pro- 11. When a solid melts, there is— silicate
duct after α emission are (A) An increase in enthalpy (B) A special form of glass to
isodiapheres (B) No change in enthalpy store water only

C.S.V. / August / 2009 / 727


(C) Hydrated form of glass 23. 0·2 g of an organic compound 30. An orange coloured when solu-
(D) Hydrated silica containing C, H and O, on com- tion acidified with H2SO4 treated
bustion yielded 0·147 g CO 2 and with a substance X gives a blue
18. Which one is correct representa- 0·12 g water. The percentage of coloured solution of CrO5. The
tion for oxygen in it is— substance X is—
2 SO 2 (g) + O2 (g) 2 SO3 (g) ? (A) 73·3% (B) 78·45% (A) H2O (B) dil. HCl
[P SO3]2 (C) 83·23% (D) 89·50% (C) H2O2 (D) Conc. HCl
(A) Kp =
[P SO2]2 [PO2]
24. At 25°C, the solubility of iodine in 31. In the reaction, C 6H5CH3
[SO 3] 2 Oxidation NaOH
(B) Kc = water is 0·35 g/l. If distribution ⎯⎯⎯⎯→ A ⎯⎯→ B
[SO 2] 2 [O2] coefficient of I 2 between CS 2 and Soda lime
[ nSO3]2
⎯⎯⎯⎯→ C, the product C is—
water is 600, the solubility of I2 in Δ
(C) Kp =
[ nSO2]2 [nO2] CS 2 in g/l will be about— (A) C6H5OH (B) C6H6
(A) 1714 (B) 210
[ ]
P –1 (C) C6H5COONa (D) C6H5ONa
×
Total moles (C) 569·6 (D) 857
32. In the purification of bauxite by
(D) All correct 25. The reaction described below is— Bayer’s process, the chemical
19. Super halogen is— CH 3 (CH 2)5 used is—
OH– (A) Na 2CO3
(A) F2 (B) Cl2 H3C C—Br ⎯→
(C) Br2 (D) I 2 (B) Cryolite
H
(C) NaOH
20. In the following reaction (CH 2)5 CH 3
(D) A mixture of NaOH and
[Cu (H2O)3 (OH)]+ + [Al (H2O)6] 3+ OH—C CH 3 Na 2CO3
(a) (b) H 33. Toilet soap is—
→ [Cu(H2O)4] 2+ + [Al (H2O)5 (OH)]2+ (A) SE 1 (B) SN2
(c) (d)
(A) A mixture of calcium and
(C) SN1 (D) SE 2 sodium salts of higher fatty
(A) (a) is an acid and (b) is a
26. Chloride ion and potassium ion acids
base
are isoelectronic— (B) A mixture of potassium
(B) (a) is a base and (b) is an
(A) Their sizes are same stearate and glycerol
acid
(B) Cl– ion is bigger than K + ion (C) A mixture of sodium salts of
(C) (c) is conjugate acid of (a)
higher fatty acids
and (d) is conjugate base of (C) K+ ion is relatively bigger
(b) (D) A mixture of potassium salts
(D) Their sizes depend on other
of higher fatty acids
(D) (c) is conjugate base of (a) cation and anion
and (d) is conjugate acid of 34. When concentrated nitric acid is
O
(b) heated it decomposes to give—
27. The polymer H2C CH 2 (A) Oxygen and nitrogen
21. A complex shown below can
exhibit— (B) Nitric oxide
O O (C) Oxygen
CH 2 (D) Nitrogen dioxide and oxygen

is obtained when HCHO is 35. Which of the following is an


allowed to stand. It is a white example of zwitter ion ?
solid. The polymer is— (A) Urea
(A) Trioxane (B) Glycine hydrochloride
(B) Formose (C) Ammonium acetate
(C) Para formaldehyde (D) α-alanine
(A) Optical isomerism only (D) Metaldehyde
(B) Geometrical isomerism only 36. When SO2 is passed through
28. The function of flux during the cupric chloride solution—
(C) Both optical and geometrical smelting of the ore is—
isomerism (A) The solution becomes
(A) To make the ore porous colourless and a white pre-
(D) None of these (B) To remove gangue cipitate of Cu 2Cl2 is obtained
22. In the chemical reaction (C) To facilitate reduction
(B) A white precipitate is ob-
Ag2O + H2O + 2e → 2Ag + 2OH– (D) To facilitate oxidation tained
(A) Water is oxidised 29. RMgX on heating with cyanogen (C) The solution becomes colour-
(B) Electrons are reduced chloride gives— less
(C) Silver is reduced (A) R—NC (B) R—Cl (D) No visible change takes
(D) Silver is oxidised (C) R—CN (D) None of these place

C.S.V. / August / 2009 / 728


37. Indigo belongs to the class of— respectively. The gas which can (C) CHCl3
(A) Mordant dye most easily be liquefied is—
(A) O2 (B) N2 (D)
(B) Vat dye
(C) NH3 (D) CH4
(C) Direct dye
(D) Disperse dye 42. In presence of peroxide, hydro- 47. Grignard’s reagents add to—
gen chloride and hydrogen iodide (A) C —
—O (B) —C — —
—N
38. 0·5 g of an organic compound do not give anti-Markownikov’s
was Kjeldahlised and the NH 3 (C) C —
—S (D) All of these
addition to alkenes because—
evolved was absorbed in certain (A) Both are highly ionic 48. Which of the following is most
volume of 1N H 2SO4. The resi- (B) One is oxidising and the acidic ?
dual acid required 60 cm3 of N/2 other is reducing (A) Phenol
NaOH. If the percentage of nitro- (C) One of the steps in both the
gen is 56 then the volume of 1N (B) m-chlorophenol
reactions is endothermic
H2SO4 taken was— (D) All the steps are exothermic (C) Benzyl alcohol
(A) 30 ml (B) 40 ml in both the cases (D) Cyclohexanol
(C) 50 ml (D) 60 ml 43. The number of P—O—P bonds 49. Number of σ and π-bonds in
in cyclic metaphosphoric acid is—
39. Equal weights of methane and benzaldehyde is—
(A) Zero (B) Two
oxygen are mixed in an empty (A) 4π-bonds and 13 σ-bonds
(C) Three (D) Four
container at 25°C. The fraction of
44. Which among MeX, RCH2X, (B) 4π-bonds and 8 σ-bonds
the total pressure exerted by oxy-
gen is— R2CHX, R3 CX is most reactive (C) 4π-bonds and 14 σ-bonds
towards SN2 reaction ?
(A) 1/3 (B) 1/2
(A) MeX (B) RCH2X (D) 8π-bonds and 10 σ-bonds
1 273 (C) R2CHX (D) R3CX
(C) 2/3 (D) × 50. An organic compound X, on
3 298
45. Which one has the pyramidal treatment with acidified K2Cr2O7
40. Which of the following is the least shape ? gives compound Y which reacts
stable carbanion ? (A) PF 3 (B) CO32 – with I2 and sodium carbonate to
(A) HC ≡ C– (B) (C6H5)3C– (C) SO3 (D) BF 3 form triiodomethane. The com-
(C) (CH3)3C – (D) CH3– pound X can be—
46. Which is not a moth repellent ? (A) CH3OH
41. The values of vander Waals
constant ‘ a’ for the gases O2, N 2, Cl Cl (B) CH3COCH3
(A)
NH3 and CH4 are 1·360, 1·390, (C) CH3CHO
4·170 and 2·253 L2 atm mol– 2 (B) Perchloroethane (D) CH3CHOHCH3

ANSWERS WITH HINTS

C.S.V. / August / 2009 / 729


Introduction (b) Metamorphosis of spermatids into sperm, the
● The formation of gametes usually takes place in spermiogenesis or spermatoleosis.
specially restricted parts of the body which are known
as gonads (i.e., Ovary in female and testis in male). Formation of Spermatids
● The lining of sex organs (gonads) is termed as germi- ● The formation of spermatids starts from the primary
nal epithelium which possesses power of division and germ cells or primordial germ cells of the testis and is
produces the cells which undergo reduction to form
differentiated into three phases :
the gametes.
● The process by which gametes are produced in the (i) Multiplication phase—The testes are com-
gonads is known as gametogenesis. The process of posed of seminiferous tubules which are lined by
formation of male sex cells or male gametes is called germinal epithelium. Some of the cells of ger-
spermatogenesis and the products are known as minal epithelium get differentiated as primary
spermatids. The maturing of ova from the cells of germ cells or primordial germ cells. These be-
ovary is termed oogenesis and the products are come comparatively bigger in size and possess a
known as ova or ootids. prominent nucleus. During multiplication phase,
the primordial germ cells multiply by mitosis and
Spermatogenesis the ultimate products are known as sperma-
● The process of maturation of reproductive cells in the togonia. Each spermatogonium represents a
testis of male so as to form male gametes or sperm is diploid cell consisting of 2n-number of chromo-
known as spermatogenesis. somes.

Blood vessels, lymphatic


vessels and nerves pass
alongside the vas deferens Seminiferous
to supply the testis. tubules

Vas deferens
Epididymis
Interstitial cells
produce testosterone

Tubule network

Compartment Lumen of
seminiferous
tubule

Sperm
Spermatid
Secondary spermatocyte
Primary spermatocyte

Spermatogonium
A tubule network connects Sertoli cells nourish and
seminiferous tubules in protect sperm during
compartments to their development
the epididymis.

Fig. : Sectioned view of testis showing seminiferous tubules and epididymis.


● The process is much complicated and can be studied (ii) Growth phase—The spermatogonia thus formed
under two heads : after repeated divisions from primary germ cells
(a) Formation of spermatids from the cells of ger- do not undergo further division by mitosis but
minal epithellium produced as a result of meiotic prepare themselves for maturation division.
division. These increase in size by the accumulation of

C.S.V. / August / 2009 / 733


nourishing material obtained from germinal cells. sperm cell. The spermatid contains mitochondria,
The enlarged cells are known as primary sper- Golgi body and centriole, but with one haploid set of
matocytes. The nucleus of each primary sper- chromosomes. In this form it is not capable of func-
matocyte is of ordinary size but soon enlarges tioning as a male gamete because the male gamete
considerably so that in a mature primary sperma- is motile and has to reach the ovum.
tocyte the nucleus is much larger than that of the ● During spermiogenesis, the nucleus shrinks by losing
spermatogonial cell.
water from the nuclear sap and the chromosomes
(iii) Maturation phase—Each diploid primary sper- become closely packed.
matocyte goes through the first meiotic division
(reduction division) with its long drawn out ● The nucleus changes from its usual spherical form
prophase. The pairing and splitting of homolo- and becomes elongated and narrow.
gous chromosomes takes place and leads to the ● The centrosome of the spermatid consists of two
formation of tetrad. Crossing over exchange of centrioles. During spermiogenesis, these move and
homologous. Chromosomes also takes place. As come to lie behind the nucleus. One of them enters
a result, halving of the chromosome number
the depression developed in the posterior part of the
takes place in the first maturation division and the
nucleus and is called the proximal centriole.
two cells formed are haploid having n-chromo-
somes. These are known as secondary sper- ● The other one is called the distal centriole. It occupies
matocytes. a position behind the proximal one with its axis
coinciding with the longitudinal axis of the spermato-
zoon. This distal centriole gives rise to the axial fila-
ment of the flagellum and act as basal granule.
● Mitochondria from different parts of the spermatid
accumulate around the proximal part of axial filament
and distal centriole in the middle piece of the sper-
matozoon.
● The Golgi complex of the spermatid forms the acro-
some of the spermatozoon and forms a cap around
the nucleus. It helps the sperm in penetration into the
ovum.

Structure of Spermatozoon
● A typical spermatozoon consists of head, middle
piece and tail.
● Head is the anterior most part of the spermatozoon. It
is often conical but its shape varies considerably. The
anterior tip of the head is formed of acrosome which
enables the spermatozoon to penetrate through the
egg membrane.
● Acrosome contains hyaluronidase enzymes and
hydrolases, released when the sperm cell-membrane
fuses at several points with the acrosome during
acrosome reaction, dissolving the jelly around the
egg so that sperm can penetrate it.
Fig. : The stages of spermatogenesis ● The remaining portion of the head is occupied by the
● The secondary spermatocytes undergo 2nd matura- nucleus, whose posterior margin is depressed to
tion division which is simple mitotic division. As a accommodate the proximal centriole, which initiates
result, each secondary spermatocyte is divided into cell-division in the fertilized egg.
equal cells which are known as spermatids. ● Middle piece of the spermatozoon encloses distal
● Each spermatid consists only one set of homologous centriole and mitochondria, which carry oxidative
pairs and is, therefore, haploid. It undergoes meta- enzymes which provides energy for the propulsion of
morphosis to form the sperm. As a result of these two the spermatozoon.
consecutive divisions, four haploid spermatids are
formed from each spermatocyte. ● Tail or flagellum is the longest but most narrow part
of the spermatozoon and its movements propel the
Spermiogenesis or Spermatoleosis spermatozoon. It consists of an axial filament, a thin
● Spermiogenesis is the gradual differentiation of layer of cytoplasm and an outer sheath of plasma-
stationary rounded spermatid into an active motile lemma.

C.S.V. / August / 2009 / 734


C.S.V. / August / 2009 / 735
Oogenesis
The oocyte completes
Immature ovum meiosis while the follicle Secondary
(primary oocyte) enlarges and matures. follicle Mature
in primary follicle graafian follicle

Immature ovum
(secondary oocyte)

Germinal
epithelium
Fluid-filled cavity

Rupturing of follicle
during ovulation

Ovulated immature ovum


Degenerating (secondary oocyte)
corpus luteum
Mature Developing
Blood vessels, nerves corpus luteum corpus luteum
and lymphatic vessels
service the ovary.

Fig. : Sectioned view showing primary secondary and graafian follicles in which oocytes form through meiosis.

● Oogenesis is the process of development and matu- ducts are known as oocytes. These are diploid cells
ration of ova from the primary germ cell produced by containing the same number of chromosomes as in
the division of cells of the germinal epithelium in the the parent body. The oocytes cease to divide and
ovary of female organism. The process includes three enter the growth phase.
steps or phases—Multiplication phase, Growth phase ● Growth phase of female gametes is much prolonged.
and Maturation phase. The growth in size of the oocyte is not only due to
● Multiplication phase is the first phase and similar to increase in the amount of its cytoplasm but mostly
that found in spermatogenesis. The cells of germinal due to accumulation of nutritive material. The most
epithelium divide and produce oogonia. These common form of food storage consists of granules of
undergo repeated mitotic divisions and the final pro- yolk. Yolk consists of proteins, phospholipids and
Stages Cells Nuclear and cell divisions Chromosomes neural fat. The follicle cells assist the growth of
in each cell
oocyte by secreting substances which are taken up
Oogonium 46
by the oocyte.
● In the primary oocyte, large amount of fat and proteins
become accumulated in the form of yolk and due to
its heavy weight, it is usually concentrated towards
Proliferation of
cells by mito- the lower portion of egg, forming the vegetal pole.
sis before birth
The portion of the cytoplasm containing the egg
Primary pronucleus remains separated from the yolk and
oocytes
in primary
46 occurs towards the upper side of egg forming the
follicles animal pole. During the growth phase, tremendous
After puberty, one primary
oocyte matures and is changes occur in the nucleus of the primary oocyte.
Enlarged ovulated about once a
primary
oocyte month until menopause.
46
Diploid Nuclear Changes
Meiosis I Haploid
● Simultaneously with the growth of oocyte, its nucleus
The secondary
Polar
Secondary oocyte oocyte is released at 23 enters into the prophase of meiosis. The homologous
in secondary and ovulation and does
body graafian follicles not complete meio- chromosomes pair but further steps are postponed
sis II unless it is ferti-
lized by a sperm. until the end of growth period.
Meiosis II
● The nucleus increases in size by the formation of
Polar body 23 nuclear sap. It is called the germinal vesicle. The
Polar bodies
Mature nucleoli are greatly increased to compensate the
ovum
disintegrate. large amount of RNA required for the increased
Fig. : The stages of oogenesis metabolism of growing oocyte.

C.S.V. / August / 2009 / 736


itself. This large cell is called secondary oocyte. It
Differences between
receives almost whole of the cytoplasm of the primary
Spermatogenesis and Oogenesis
oocyte. The small cell is known as the first polar
Spermatogenesis Oogenesis body. It consists almost a bare nucleus. But both the
cells have haploid (n) number of chromosomes.
1. It occurs in the testes. 1. It occurs in the ovaries. ● The haploid secondary oocyte and first polar body
2. Growth phase is short so 2. Growth phase is very long pass through the second maturation division. Due to
that spermatocytes are so that oocytes are much second maturation division, the secondary oocyte
only twice the size of larger than oogonia. forms a mature egg and a second polar body. By the
spermatogonia. second maturation division, the first polar body also
3. Spermatocytes have cyto- 3 . Oocytes have cytoplasm divides into two secondary polar bodies. These polar
plasm and nucleus with rich in RNA, ATP, enzy- bodies ooze out from the egg and degenerate, while
normal contents. mes and yolk; and nucleus the haploid egg cell or secondary oocyte is released
with giant chromosomes from the ovary for fertilization.
and large nucleoli. ● When it is released from the ovary, the secondary
4. A primary spermatocyte 4. A primary oocyte divides oocyte is surrounded by a protective sphere of cells
divides equally to form unequally to form one called the corona radiata which is derived from the
two similar secondary large secondary oocyte follicle in the ovary. The size of the corona radiata
spermatocytes. and one minute polar and its irregular surface makes it easier for the entire
body. assembly to be trapped by the uterine tube.
5. A secondary spermato- 5. A secondary oocyte also
cyte also divides equally, divides unequally, forming Key Points
forming two similar sper- one large ootid and one
● Amount of yolk in the ooplasm varies from species to
matids. minute polar body.
species.
6. Upto the formation of 6. Oogonia are separate and ● The eggs with very little amount of yolk are known as the
spermatids, the cells often surrounded by follicle microlecithal eggs, e.g. , Amphioxus, Eutherian mammals.
remain interconnected. cells. ● The eggs containing moderate amount of yolk are called
7. A spermatogonium pro- 7. An oogonium produces mesolecithal eggs, e.g., Petromyzon, Dipnoi amphibia.
duces four functional sper- one functional ovum and ● The eggs with large amount of the yolk are known as
matozoa. three non-functional polar macrolecithal eggs, e.g. , Myxine, Cartilaginous and
bodies. bonyfishes, reptiles, birds and monotremata.
● The eggs with evenly distributed yolk contents in the
8. Spermatozoa are minute, 8. Ova are much larger,
ooplasm are known as Homolecithal eggs, e.g., eggs of
streamlined, yolkless and rounded, often with yolk Echinoderms.
motile. and non-motile. ● The eggs in which the yolk is not evenly distributed in the
9. Spermatogenesis is often 9. Oogenesis is not necessa- ooplasm are known as Heterolecithal eggs.
a continuous process. rily a continuous process. ● When in heterolecithal eggs, the amount of yolk is con-
10. Spermatogenesis is 10. Oogenesis is often comp- centrated in the one half of the egg to form the vegetative
usually completed in the leted in the reproductive pole of the egg; then this condition is known as teloleci-
thal. Telolecithal eggs may be moderately telolecithal
testes so that mature tract of the female or even
( e.g., Amphibians) or highly telolecithal (e.g., Hen).
sperms are released. in water as oocytes leave
● In macrolecithal and highly telolecithal eggs, the amount
the ovaries.
of yolk is very large and it occupies the largest portion
● Maturation phase—A full grown primary oocyte except a small disc-shaped portion of the cytoplasm. The
undergoes first meiotic division and produces two cytoplasm contains the zygote nucleus and is known as
the germinal disc, e.g., eggs of fishes, reptiles and birds.
daughter cells. The division of its cytoplasm is un-
● In the centrolecithal eggs, the yolk accumulates in the
equal. One daughter cell is extremely small, whereas
centre of the ooplasm, e.g. , Insects.
the other is almost as large as the primary oocyte

OBJECTIVE QUESTIONS
1. In spermiogenesis, the sperma- (B) Cells of Sertoli (C) Polar bodies
tozoa are produced from— (C) Lymph spaces (D) None of these
(A) Spermatids (D) Capillaries 4. Spermatogenesis requires the
(B) Primary spermatocytes presence of—
3. The minute cells which are sepa-
(C) Spermatogonia rated from the developing over (A) Fructose
(D) Secondary spermatocytes during their maturation phase, are (B) Progesterone
called— (C) Testosterone
2. Between the spermatogonia are
the— (A) Primary oogonia (D) Thyroxine
(A) Epithelial cells (B) Secondary oogonia (Continued on Page 765 )

C.S.V. / August / 2009 / 737


Introduction Pharyngeal gill slits Dorsal hollow nerve cord

● Chordates are bilateral and deuterostomial eucoelo- Notochord


mate eumetazoa, basically possessing, in the
embryo or throughout life a flexible but firm support-
ing skeletal rod, called notochord.
● Phylum Chordate is the largest of the deuterostome
Mouth Anus Tail
phyla. It is the highest and the most important phylum
Pharynx Trunk Hepatic caecum Digestive tube
comprising a vast majority of living and extinct ani-
mals including humans. Fig. : Diagrammatic side view of a chordate showing the
● Most of the living chordates are well known familiar three fundamental chordate characters.
vertebrate animals such as the fishes, amphibians, 2. Notochord or chorda dorsalis—The notochord is
reptiles, birds and mammals. Besides, they include a an enlarged rod-like flexible structure extending the
number of marine forms such as the tunicates and length of the body. It is present immediately beneath
lancelets(amphioxus). the nerve cord and just above the alimentary canal.
● The chordates are probably the most conspicuous ● Notochord originates from the endodermal roof of the
and the best-known group in the entire animal embryonic archenteron.
kingdom, partly because of their large size and partly
● Structurally, it is composed of large vacuolated noto-
because of the important role they perform in their
chordal cells containing a gelatinous matrix and sur-
ecosystems. They are of primary interest because
rounded by an outer fibrous and an inner elastic
humans himself are member of the group.
sheath.
● From a purely biological viewpoint, chordates are
● The notochord is the prime diagnostic feature of the
interesting because they illustrate well the broad
phylum chordata which derives its name from it.
biological principles of evolution, development and
relationship. ● Notochord is present at some stage in all chordates.
In most vertebrates, it occurs complete only in
The Fundamental Chordate Characters embryo, but remnants may persist between the verte-
● All the chordates possess three outstanding unique brae, which obliterate it.
characteristics at some stage in their life cycle. These ● The notochord may persist even in adult cephalo-
three fundamental morphological features include: chordates.
1. A dorsal hollow or tubular nerve cord. 3. Pharyngeal gill slits—In all the chordates, at some
2. A longitudinal supporting rod-like notochord. stage of their life history, a series of paired lateral gill
slits perforate through the pharyngeal wall of the gut
3. A series of pharyngeal gill slits.
behind the mouth. These are also termed as pharyn-
1. Dorsal hollow nerve cord—The central nervous geal pouches.
system of the chordates is present dorsally in the
● They are seen only during embryonic development in
body. It is in the form of a longitudinal, hollow or
most vertebrates. In the lower chordates, fishes and
tubular nerve cord lying just above the notochord and
amphibian larvae, the pharyngeal pouches become
extending lengthwise in the body.
functional gills.
● The nerve cord or neural tube is derived from the
dorsal ectodermal neural plate of the embryo and ● In terrestrial vertebrates, the pouches are modified for
encloses a cavity or a canal, called neurocoel. various purposes.
● In humans, the first pair of pouches become the
● Nerve cord persists throughout life in the lower chor-
dates, but in the higher chordates, it is surrounded or eustachian tubes. The second pair become the
replaced partly or fully, in adults by a jointed vertebral tonsils, while the third and fourth pairs become the
column. thymus gland and parathyroid.
● The above common features appear during early
● In vertebrates, the anterior region of nerve cord is
specialized to form a cerebral vesicle or brain which embryonic life of all the chordates, but all the features
is enclosed by a protective bony or cartilaginous rarely persist in the adult. Often they are modified or
cranium. The posterior part of nerve cord becomes even lost in adult stages of higher chordates.
the spinal cord and protected within the vertebral ● The notochord disappears during development in
column. most vertebrates, the nerve cord remain in adults.

C.S.V. / August / 2009 / 738


Advancement of Chordates over other Phyla ● The vertebrates are commonly called higher chor-
● Phylum chordata has some advantages over other
dates.
phyla due to certain characters : ● The protochordates lack a head and a cranium, so
1. Living endoskeleton—Only chordates possess a that they are known as Acraniata.
living endoskeleton. It grows in size with the rest of ● Vertebrates have a distinct head and cranium, there-
the body. This living endoskeleton permits greater fore, called Craniata.
freedom of movement. ● The vertebrates are further subdivided into Agnatha
2. Efficient respiration—The gills in aquatic chordates and Gnathostomata.
and the lungs in terrestrial forms, form efficient ● Agnathans lack true jaws and paired appendages,
organs of respiration. include a small number of primitive but highly specia-
3. Efficient circulation—The circulatory system of the lized fish-like forms, the extinct Ostracoderms and
chordates is well developed and the blood flows the modern Cyclostomes.
freely in the respiratory organs ensuring rapid ex- ● All other vertebrates have true jaws and paired
change of gases. appendages, called Gnathostomata.
4. Centralized nervous system—A growing tendency ● A basic division of Gnathostomata recognises two
of centralization of nervous system is found in chor- super classes Pisces and Tetrapoda. The super-
dates and the sensory system is modified accord- class Pisces includes all the fishes which are strictly
ingly. The advancement of nervous and sensory aquatic forms with fins. The superclass Tetrapoda is
organs explains the great power of the chordates for formed by four-legged vertebrates including amphi-
adapting themselves most successfully to a variety of bians, reptiles, birds and mammals.
environments.
General Characteristic of Phylum Chordata
Major Subdivisions of Chordata 1. Aquatic, aerial or terrestial.
● Chordata is divided into three subphyla : Urochor- 2. Body bilaterally symmetrical and metamerically seg-
data, Cephalochordata and Vertebrata. mented.
● The first two subphyla are groups of primitive chor- 3. A postanal tail usually project beyond the anus at
dates without vertebral column and are commonly some stage and may or may not persist in the adults.
called lower chordates. They are grouped together as 4. Exoskeleton often present in some forms.
Protochordates.
5. A cartilaginous or bony living jointed endoskeleton
Group A : Acraniata Group B : Craniata present in the majority of vertebrates.
(Protochordata) or Lower (Euchordata) or Higher 6. Body triploblastic with three germinal layers.
Chordata Chordata
7. Coelomates, having a true coelom.
1. Exclusively marine, small- 1. Aquatic or terrestrial,
sized chordates. mostly large sized
8. A skeletal rod, the notochord present at some stage
vertebrates. in life cycle.
2. No appendages, cephali- 2. Usually 2 pairs of 9. Pharyngeal gill slits present at some stage.
zation and exoskeleton. appendages, well- 10. Digestive system complete with digestive glands.
developed head and
exoskeleton present. Classification of Chordates
3. Coelom enterocoelic, 3. Coelom schizocoelic, ● Phylum Chordata is separated into three primary
budding off from embryo- arising by splitting of subdivisions, called subphyla, based on the character
nic archenteron. mesoderm. of notochord. These are :
4. Notochord persistent. No 4. Notochord covered or
1. Subphylum—Urochordata
skull, cranium and verte- replaced by a vertebral
bral column. column. Skull and cra- 2. Subphylum—Cephalochordata
nium well developed. 3. Subphylum—Vertebrata
5. Pharynx with permanent 5. Pharyngeal gill clefts
gill clefts. Endostyle pre- persist or disappear Subphylum—Urochordata
sent. endostyle absent. ● Characters—The urochordates are marine, mostly
6. Heart chamberless when 6. Heart made of 2, 3 or 4 sessile, filter-feeders. They have the following
present. No red blood chambers. Blood con- characters :
corpuscles in blood. tains R.B.C. 1. The notochord occurs only in the tail of the larva
7. Kidneys protonephridia. 7. Kidneys meso- or and disappears in the adult.
metanephridia. 2. The nerve cord is present in the larva, but is
8. Sexes separate or united 8. Sexes separate. Only replaced by a single dorsal ganglion in the adult.
reproduction asexual as sexual reproducing,
well as sexual. Gono- Gonoducts always pre-
3. The gill slits are numerous, persist in the adult and
ducts usually absent. sent. open into an ectoderm-lined cavity, the atrium,
9. Development indirect 9. Development indirect
instead of to the exterior. There are no gills.
with a free-swimming or direct, with or with- 4. The tail does not persist throughout life.
larval stage. out a larval stage. Examples—Herdmania, Doliolum, Salpa.

C.S.V. / August / 2009 / 739


Subphylum—Cephalochordata 8. Endoskeleton—An internal framework of carti-
● Characters—The cephalochordates are also marine lage or bone or both is always present. It grows
and filter-feeders. They have the following charac- with the body. The notochord is replaced partly
ters : or fully by a jointed vertebral column in the adult,
1. The notochord extends upto the tip of snout and hence called vertebrata.
persists throughout life. 9. Circulatory system—Heart is ventral and con-
2. The nerve cord persists throughout life, but no sists of 2 to 4 chambers. Blood contains red
brain is formed. corpuscles that have haemoglobin. There is
3. The gill slits are numerous and persist in the adult. closed circulatory system consisting of blood
They open in the atrium. There are no gills. vascular and lymphatic system. Hepatic portal
4. The body wall consists of myotomes. system is present. In many vertebrates, renal
5. Tail persists throughout life. portal system is also present.

Subphylum Vertebrata 10. Respiratory organs—Respiratory organs may


The term vertebrate was introduced by Lamarck. be gills, skin in aquatic vertebrates. Higher terre-
Vertebrates show the following characters : strial vertebrates have lungs for respiration.
1. Body regions—Vertebrate body typically con- 11. Nervous system—Nerve cord is differentiated
sists of 4 regions—head, neck, trunk and tail. into anterior brain and posterior spinal cord. A
The neck and tail may be lacking in some cases. skull develops to provide cranium for protecting
Head is prominent and sense organs are well the brain.
developed.
12. Sense organ—Sense organs are eyes, ears,
2. Segmentation—There is some degree of seg- tongue, nasal chambers and skin.
mentation. It is indicated by muscles, vertebrae,
ribs, paired blood vesssels and nerves. 13. Cranial nerves—Cranial nerves may be 8, 10 or
3. Appendages—There are as a rule 2 pairs of 12 pairs.
appendages, which may be fins or limbs. One or 14. Digestive tract—Digestive tract is complete and
both pairs are absent in certain forms. is ventral to the central nervous system.

Outline Classification of Phylum Chordata

Group Classes
Acrania or 1. Ascidiacea
Subphylum 2. Thaliacea
Protochordata Urochordata
(Lower chordata) 3. Larvacea
Subphylum 1. Leptocardii
Cephalochordata

Phylum
Chordata

Division 1. Ostracodermi
Agnatha
Group 2. Cyclostomata
Craniata Anamniota
3. Placodermi
or Euchordata
(Higher chordata) {
Subphylum
Vertebrata
Superclass
Pisces { 4. Chondrichthyes
5. Ostelchthyes
(Lower
vertebrata)
Division
Gnathostomata ⎧⎪6. Amphibia Amniota
Superclass ⎨⎪7. Reptilia
8. Aves (Higher
Tetrapoda ⎩9. Mammalia vertebrata)

4. Integument—Skin consists of epidermis of many 15. Endocrine system—All vertebrates have duct-
layers of cells and dermis of connective tissue. less endocrine glands that secrete hormones for
metabolic regulation.
5. Exoskeleton—Epidermis often produces an
exoskeleton of keratinized cells. It may consist of 16. Excretory system—Vertebrates have a pair of
scales, feather, hair. kidneys that require a high blood pressure for
6. Muscles—Three types of muscles–striped, un- working.
striped and cardiac are present. 17. Reproductive system—The sexes are sepa-
7. Coelom—There is a spacious true coelom that rate. They have gonoducts. Fertilization may
contains viscera loosely suspended in it by internal or external. Asexual reproduction is lack-
mesenteries. ing.

C.S.V. / August / 2009 / 740


Geological Representation Examples—Lampreys (Petromyzon) and Hagfishes
● Cambrian and Ordovician periods—The first fossils (Myxine).
of vertebrates were found in the rocks of the Ordovi- ● Gnathostomata—These are jawed vertebrates, hav-
cian period in the form of Ostracoderms. These were ing true jaws and paired limbs. Embryonic notochord
small jawless bony fishlike forms related to cyclo- is usually replaced in adult by a vertebral column.
stomes. This shows that their chordate ancestors Mouth has jaws hence called gnathostomata. Paired
must have existed much before in the late Cambrian. nostril are present.
● Silurian and Devonian periods—Some fossil fishes ● Gnathostomata is divided into two super classes :
were found in the Silurian period and far more were Pisces and Tetrapoda.
present in the succeeding Devonian period which is ● Super class Pisces—It includes true fishes. All are
known as 'Age of Fishes'. aquatic. The body bears paired as well as median
● Ostracoderms were jawless fishes, but during Devo- fins. Respiration occurs typically by gills. They are
nian, the first jawed fish—the Placoderm arose. The cold-blooded. Each eye has a well developed nictita-
placoderms became extinct without leaving any living ting membrane and possess scaly skin.
representative. It is likely that early placoderms were Super class Pisces is divided into following three
ancestors of cartilaginous and bony fishes both, classes :
which had true jaws.
1. Class Placodermi—It includes the earliest (Paleo-
● Carboniferous periods—In late Devonian to early
zoic) fossil fishes which lived in fresh water. Body had
Carboniferous period, the lobed-finned fishes arrived.
an external protective armour of bony scales or
They were the first vertebrates to walk on land and
plates. Primitive jaws with teeth were present. Skele-
became the primitive stem to amphibia.
ton was bony and fins were mostly formed of large
● The amphibians became abundant and mutated in spines.
many directions during carboniferous, which is Examples—Climatius, Dinichthys.
usually known as the ‘Age of Amphibians’.
2. Class Chondrichthyes—Mostly marine fishes hav-
● Mesozoic era—In the early carboniferous, the very ing cartilaginous endoskeleton and skin with placoid
primitive amphibians also gave origin to primitive scales. Gill slits not covered by operculum. Male
reptiles. Reptiles reached their peak during Mesozoic fishes have pelvic claspers.
era, which is aptly known as the ‘Age of Reptiles’. Examples—Scoliodon (Dogfish) and Chimera (Rat-
They included dianosaurs. fish).
● The ancestral mammals were derived from the 3. Class Osteichthyes—Fresh water as well as marine
primitive reptiles during Triassic period. The first bird fishes. Endoskeleton mostly bony. Skin having
also appeared in the late Jurassic period and one of various types of scales (cycloid, clenoid) other than
the fossil Archaeopteryx had both reptilian as well placoid. Gill slits are covered by operculum. Male
as avian characters. fishes without claspers.
● Cenozoic era—Following the decline of the reptiles Examples—Labeo, Hippocampus, Protopterus.
during the late Mesozoic era, both birds and mam- ● Super class Tetrapoda—Land vertebrates with two
mals started flourishing. pair of pentadactyle limbs. They have cornified skin
● The mammals became more diversified of all the ani- and lungs. Sensory organs are adapted for vision,
mals during Cenozoic era, which is called the ‘Age of hearing and smelling etc. Tetrapoda is divided into 4
Mammals’. classes :
1. Class amphibia—Adults are amphibious and respire
Classification of Subphylum Vertebrata by lungs. Larval stages usually aquatic and breathe
● Subphylum vertebrata is divided into two sections : by gills. Skin moist, glandular and with no external
Agnatha and Gnathostomata. scales. Heart 3-chambered. Cold blooded.
● Agnatha—These are jawless vertebrates and are Example—Rana (frog), Bufo (toad).
most primitive. The mouth does not possess jaws 2. Class Reptilia—Terrestrial, skin dry and covered by
hence called Agnatha. They do not have exoskeleton ectodermal horny scales or bony plates. Heart incom-
and paired appendages, and have single nostril. pletely 4-chambered. Cold blooded.
● Section Agnatha is further divided into following Example—Hemidactylus (lizard), snakes, crocodiles,
classes : turtles.
1. Class Ostracodermi—These were group of 3. Class Aves—Body covered with feathers. Forelimbs
extinct agnatha, and were primitive, heavily modified into wings. No teeth in beak. Heart 4-
armoured. World's first vertebrates collectively chambered. Warm blooded.
called Ostracoderms. Example—Columba (Pigeon).
2. Class Cyclostomata—These have body without 4. Class Mammalia—Body covered by hairs. Skin glan-
scales, jaws and lateral fins. Mouth rounded and dular. Mammary glands present. Heart 4-chambered.
suctorial, gills 5–16 pairs. They are parasites and Warm-blooded.
scavengers. Example—Homo (humans), whale, elephant.

C.S.V. / August / 2009 / 741


Review at a Glance
● Phylum chordata name was established by Balfour.
● Early chordates evolved during Ordovician period of Paleozoic era.
● According to Garstrong, chordates have evolved from echinoderm larva by paedogenesis.
● All vertebrates are chordates but all chordates are not vertebrates.
● Presence of notochord, pharyngeal gill slits and dorsal tubular nerve cord are three characteristic features of chordates.
● In vertebrates notochord is replaced by vertebral column.
● Notochord is derived from mesoderm.
● In urochordata, notochord is found only in tail region of larval stage.
● In cepalochordata, notochord is present throughout the life.
● In mammals, remanent of notochord is present among invertebral disks as nucleus pulposus.
● Dorsal tubular nerve cord is ectodermal in origin.
● All chordates are deuterostomes, i.e., blastopore forms anus.
● All chordates are enterocoelic.
● Members of urochordata show retrogressive metamorphosis.
● The first fossile of vertebrates were found in rocks of the Ordovician period in the form of Ostracoderms.
● Devonian period is known as age of fishes.
● Carboniferous period usually known as age of amphibians.
● Mesozoic era is known as age of reptiles.
● Cenozoic era is known as age of mammals.
● World's first vertebrates collectively called Ostracoderms.

OBJECTIVE QUESTIONS
1. Vertebrates skin is covered by— 7. A common characteristic among (Continued from Page 702 )
(A) Scales all mammals is—
(B) Feathers (A) They are carnivorous
(C) Hairs (B) They have ventral nerve
cord
(D) All the above
(C) They do not moult
2. The vertebrates are the mem- (D) They have seven cervical
bers of— vertebrae
(A) Cephalochordata
8. Which of the following is not
(B) Agnatha found in the amphibian skin ?
(C) Gnathostomata (A) Epidermis
(D) Urochordata (B) Mucous glands
3. The study of fishes is called— (C) Scales
(A) Ichthyology (D) Chromatophores
(B) Herpatology 9. The presence of true placenta is
(C) Saurology characteristic feature of sub-
class—
(D) Ornithology
(A) Eutheria
4. Cartilaginous fishes do not (B) Metatheria
have— (C) Prototheria
(A) Gill slits (B) Operculum (D) All the above
(C) Scales (D) Pelvic fin
10. Which of the following era is
5. Urinary bladder is absent in— known as age of mammals ?
(A) Reptiles (B) Aves (A) Devonian
(C) Mammals (D) Amphibians (B) Carboniferous
(C) Mesozoic
6. Which of the following bird has
(D) Cenozoic
teeth in beak ?
(A) Ostrich ANSWERS
(B) Kiwi
(C) Pelican
(D) Archaeopteryx ●●● ●●●

C.S.V. / August / 2009 / 742


Introduction the lower leg. Others are very small, such as the
muscle of the eyelid.
● Muscles are in use during every waking and sleeping
● Muscle shape vary. Some muscles are triangular
moment. Some muscles are under conscious voluntary
(deltoid) while others are rectangular (rectus abdo-
control, while others operate without conscious control.
minis) or trapezoidal (trapezius).
● All muscle tissues have three characteristics in
● Many are spindle-shaped (biceps brachii).
common :
● Each skeletal muscle is composed of parallel bundles
1. Muscles can contract and shorten in length.
of individual muscle cells. Each muscle cell is sur-
2. After contraction, muscles relax and return to their rounded by delicate connective tissue. Bundles of
former length. these cells, called fascicles are wrapped in another
3. Muscles are excitable, responding to electrical or thin layer of connective tissue.
chemical signals (stimuli) from the nervous and ● The connective tissue wrappings are actually conti-
endocrine systems. nuous with each other at the end of the muscle, where
● Muscle is the only tissue in the body that has all these they unite to form an extremely tough fibrous connec-
properties. tive tissue material called a tendon. Tendons attach
muscle to bone.
● A muscle is composed of many muscle cells that are
bundled into fascicles. ● Individual skeletal muscle cells have a tubular shape,
contain many nuclei per cell and appear striated or
Antagonistic Muscles striped. These muscle cells are relatively large com-
● A muscle can pull a part of the body by its contraction pared with other human cells.
(shortening). It cannot push that part by relaxation ● Muscle cells contain the same cellular components or
(elongation). Hence, the muscles are typically arran- organelles found in most other human cells, but some
ged in antagonistic (opposing) pairs, one of which components have specialized functions or are unique.
moves a body part in one direction and the other
● The smooth endoplasmic reticulum forms into parallel
moves that part in the opposite direction by its contrac-
saclike compartments called the sarcoplasmic reticu-
tion. Of course, when one muscle contracts, its oppo-
lum or SR.
sing muscle relaxes.
Muscle
● On the outer surface of the muscle cell’s plasma mem-
Blood vessels brane are the openings of tiny tubes called transverse
Fascia
surrounding Connective tissue tubules or T-tubules. These tubules are extensions of
muscle surrounding fascicle the plasma membrane that run deep into the cell and
lie close to the SR. They relay the electrical signals
Fascicle
that trigger contraction.
Tendon
● Within each muscle cell and running its entire length
Connective tissue
surrounding each are numerous protein fibres called myofibrils. Myofi-
muscle cell brils are the structures responsible for contraction.
Muscle cell
● Each myofibril consists of a series of units called
Bone
sarcomeres. The orderly arrangement of sarcomeres
gives a striated appearance to skeletal muscle cells.

Fig. : Relationship of a skeletal muscle with its muscle cells


● Each sarcomere is made of a rather complex arrange-
and connective tissue wrappings. ment of several kinds of protein. Two of these proteins
are actin and myosin.
Skeletal Muscles Structure ● Thousands of actin molecules link together to form a
● Skeletal muscles attach to bones and move the skele- long threadlike macromolecules, called actin fila-
ton. They also attach to other muscles and to the facial ments. Similarly, thousands of myosin molecules form
skin. the somewhat thicker myosin filaments. The mole-
● Some skeletal muscles are very large, such as the cular interaction between actin and myosin filaments
gastrocnemius, the main muscle forming the calf in contributes directly to muscle contraction.

C.S.V. / August / 2009 / 743


C.S.V. / August / 2009 / 744
Skeletal Muscle Contraction ● This reaction requires energy, which comes from the
ATP attached to each myosin head.
● For muscle contraction, a stimulus is needed. Before a
muscle cell can contract, it must be stimulated. Stimu- ● Myosin acts as an enzyme (when Ca2+ is present) and
lation is provided by nerve impulses that pass along breaks the ATP into ADP + P + energy. This energy is
nerve cells. used to change the shape of the myosin head, which
causes the actin filaments to slide a little towards the
● A nerve cell stimulates a muscle cell by secreting a
centre of sarcomere.
chemical substance called acetylcholine (ACh). Acetyl-
● After sliding, a new ATP attaches to the myosin head,
choline permits a nerve cell to communicate chemi-
cally with a muscle cell at a site called the neuromus- breaking the myosin-actin bond. When this bond is
cular junction. broken, the myosin head is released and returns to its
original shape. Immediately, the myosin head bonds to
● Upon its release, acetylcholine diffuses across the
another site on the actin filament. Then it changes
junction and binds to specific receptors on the plasma shape again and the filament slide a little past one
membrane of the muscle cell. This binding initiates an another.
electrical impulse which travels down the T-tubules
● The sequence of myosin-actin bonding, sliding and
into the interior of the cell.
release is repeated rapidly over and over until the
● When the impulse reaches the SR, Ca 2+ stored in the
muscle has shortened sufficiently. This is the sliding
SR is released. The released Ca 2+ forms bond with the filament mechanisms for muscle contraction, and it
troponin-tropomyosin protein complex which is consumes a great deal of ATP.
associated with the actin filaments.
● When an individual muscle is stimulated, the actin fila-
● When Ca2+ forms this bond, the troponin-tropomyosin ment in every sarcomere unit slide toward the middle
complex shifts position, exposing sites on the actin of the sarcomere. As a result, the myofibrils shorten. A
filaments for attachment to the head portions of the muscle cell shortens (contracts) then all its myofibrils
myosin filaments. shorten. An entire muscle contract when many of its
● After exposing sites for attachment, a series of events muscle cells contract simultaneously.
cause the shortening or contraction of a muscle cell. ● Contraction ends when the muscle cell ceases to be
Each myosin head projecting from a myosin filament stimulated by the nerve cell. If an impulse no longer
bonds with an actin filament. After forming these travels down the T-tubules to the cell’s interior, Ca2+ is
bonds, the myosin heads change shape and pull the actively transported back into the SR. Without Ca2+,
actin filaments toward the centre of sarcomere. the troponin-tropomyosin protein complex shifts back
into its original position, masking the sites on the actin
Spinal cord filaments and preventing further bonding by the myosin
heads. When this happens, the muscle relaxes and
Nerve cell
return to its original shape.
● In muscle contraction, oxygen is used, carbon dioxide
is produced, glycogen is consumed and heat is gene-
rated. Only a part of energy released in the process is
Nerve cells carry used. In muscle contraction, the rest is converted into
impulses from
spinal cord to heat.
muscle cells. ● Muscle Tonus (Tone)—All apparently relaxed skeletal
muscles always remain in partial contraction as long
as their nerves are intact. This state of sustained
partial contraction is called muscle tonus or tone. It is
Muscle a sort of mild tetanus.
cells
● The tone is maintained by a constant flow of nerve
impulses to muscle fibres. A muscle under slight ten-
sion can react more rapidly and can contract more
Neuromuscular
junction
strongly than one which is completely relaxed. Almost
End of
nerve cell all human daily activities are carried out by tetanic
contractions of muscles. Tetanus is necessary to
maintain posture and form of body.
● Muscle Fatigue—The duration for which a muscle can
remain contracted depends on its ability to supply
Plasma membrane ATPs to the contractile proteins (myosin and actin).
of muscle cell Acetylcholine ACh released from The reduction in force of contraction of a muscle after
(ACh) nerve cell attaches to prolonged stimulation is called muscle fatigue. A
ACh receptors receptors on muscle
cell plasma membrane. muscle can contract in the absence of oxygen, but it
gets fatigued sooner. This is due to the fact that lactic
Fig. : Pathway of an impulse from the spinal cord to the acid is not disposed off without oxygen and collects in
neuromuscular junction. the muscle fibres.

C.S.V. / August / 2009 / 745


● Fatigue is thus primarily caused by an excess of lactic ● A muscle contracts only when the stimulus it receives
acid. A completely fatigued muscle refuses to respond is of sufficient strength. The smallest strength of sti-
to nervous stimuli. mulus that causes contraction is called the threshold
stimulus.
Types of Muscles According to Type of ● When a muscle cell receives a threshold stimulus, it
Motion contracts with maximum strength. Even if a muscle cell
receives a stimulus greater than the threshold, it will
1. Flexors—The flexors bend one part of a limb on
another at a joint.
contract with the same maximum contraction strength.
Example— Biceps . It brings the forearm towards the
This is called the all-or-none law, and it states that a
upper arm. muscle cell will contract maximally or not at all.
2. Extensors—The extensors extend or straighten a limb. ● The all-or-none law applies to individual muscle cells,
Example—Triceps. It extends the forearm. but not to an entire muscle. This explains that muscle
3. Abductors—The abductors pull a limb away from the cells in a muscle do not all have the same threshold
middle of the body. value.
Example—Deltoideus. It draws the entire forelimb to ● Isotonic and isometric muscle contractions are used
the side. in certain exercise activities.
4. Adductors—The adductors bring a limb toward the ● Isotonic means ‘same strength’ (constant tension, and
midline of the body. in an isotonic contraction, the strength of the contrac-
Example— Latissimus dorsi. It presses the entire tion remains the same but the muscle shortens. Exa-
forelimb against the side.
mple—Weight lifting.
5. Depressors—The depressors lower a part.
● Isometric means ‘same length’. In an isometric con-
Example— Depressor mandibulae. It lowers down the
lower jaw to open the mouth.
traction, the strength of the contraction increases, but
6. Elevators—The elevators raise a part.
there is practically no shortening of the muscle. This
Example—Masseter. It lifts up the lower jaw to close
type of contraction does not result in body movement.
the mouth. Sitting upright in a chair while contracting abdominal
7. Rotators—The rotators rotate a part. muscles is an example of an isometric contraction.
Example— Pyriformis . It raises and rotates the ● Tonic (Slow, Red) Muscle Fibres—These are thin,
femur. dark red and slow contracting muscle fibres. They
8. Pronators—These rotate the forearms to turn the contain a high content of haem-protein pigment called
palms downward or backward. myoglobin, abundant mitochondria, low glycogen con-
9. Supinators—These rotate the forearm to turn the tent and poorly formed sarcoplasmic reticulum (SR).
palms upward or forward. ● Myoglobin imparts them dark colour and stores oxygen
10. Sphincters—These decrease the size of the aper- as oxyhaemoglobin. Its oxygen by anaerobic oxidation
tures to close them. in mitochondria provides energy for muscle contrac-
11. Dilators—These widen the apertures. tion. Less lactic acid accumulates in this process. This
The adductor and abductor; elevator and depressor; enables the red muscle fibres to carry on slow and
pronator and supinator; and sphincters and dilators are all sustained contractions for long periods without fatigue.
antagonistic muscles.
● The tonic muscle fibres are innervated by thin, slow
● The single isolated contraction of a muscle fibre conducting nerve fibres.
caused by a single nerve impulse or artificial stimulus ● The body muscles meant for sustained work at a slow
is called muscle twitch. Immediately after a twitch, the rate for a prolonged duration are composed mostly of
muscle fibre relaxes. red muscle fibres. The extenser muscles of the back in
● Fundamentally, skeletal muscles function by twitching, humans remain in sustained contraction to maintain
and most graceful movements of body are the product errect posture against gravity, and are rich in red
of twitches. muscle fibres.
● The contraction period is the time during which the Twitch (Fast, White) Muscle Fibres—These are
much thicker, lighter in colour and fast-contracting
muscles actually shortens. The relaxation period is the
muscle fibres. They have a low content of myoglobin,
time it takes the muscle to relax and return to its
few mitochondria, abundant glycogen granules and
original length.
well formed SR.
● A muscle twitch can be altered by increasing the fre-
● They derive energy for their fast contraction mainly by
quency of stimulation (the number of impulses per
anaerobic oxidation, accumulate lactic acid during stre-
second). Muscle contractions are known as summa-
nuous work and soon get fatigued.
tion, and are produced when the frequency of stimula-
tion is so rapid that a muscle does not have time to ● These muscle fibres are innervated by thick fast con-
relax completely after each impulse. Summation pro- ducting nerve fibres.
duces a greater strength of contraction by adding ● The body muscles, which are meant for fast and stre-
together many separate contractions. nuous work for short durations, are composed mostly
● If the frequency of stimulation is further increased so of white muscle fibres.
that there is no relaxation, the muscle will remain in a ● The muscles that move eyeballs are very rich in white
completely contracted state called tetanus. muscle fibres.

C.S.V. / August / 2009 / 746


Differences between Red and White Skeletal Muscle Fibres
Red (Slow) Muscle Fibres White (Fast) Muscle Fibres

1. They are thin. 1. They are much thicker.


2. They contain abundant mitochondria, low glycogen content 2. They are poor in mitochondria, and have abundant glycogen
and poorly formed sarcoplasmic reticulum. granules and well formed sarcoplasmic reticulum.
3. They are dark red as they contain abundant pigment 3. They are light in colour as they have very little myoglobin.
myoglobin.
4. Their myoglobin stores O2 as oxymyoglobin that relea- 4. They have little or no store of oxygen.
ses O2 for oxidation during muscle contraction.
5. They get energy for contraction by aerobic respiration. 5. They get energy for contraction mainly by anaerobic
respiration.
6. They accumulate little lactic acid. 6. They accumulate lactic acid during strenuous work.
7. They undergo slow sustained contractions for long periods. 7. They undergo fast contractions for short periods.
8. They are not fatigued with work. 8. They soon get fatigued with work.
9. They are innervated by thin, slow-conducting nerve fibres. 9. They are innervated by thick fast-conducting nerve fibres.
Example : extensor muscles of the back in man. Example : eyeball muscles.

OBJECTIVE QUESTIONS
1. Myoglobin is found in— 7. The contraction and relaxation
(A) Slow muscle fibres phases of a muscle constitute—
(B) Fast muscle fibres (A) Beat (B) Twitch
(C) Blood (C) Condition (D) Stimulus
(D) Lymph 8. The sprain is caused due to
2. Which of these plays a role in excessive pulling of—
muscle contraction ? (A) Muscles (B) Nerves
(A) K+ (B) Na + (C) Tendons (D) Ligaments
(C) Mg2+ (D) Ca 2+ 9. Sarcoplasmic reticulum plays a
3. In skeletal muscle, the T- major role during—
tubules— (A) Muscle contraction
(A) Secrete acetylcholine (B) Muscle excitement
(B) Give mechanical support
(C) Muscle relaxation
(C) Relay electrical signals
(D) All the above
(D) Store calcium
4. As compare to the slow muscle, 10. When a muscle bends one part
fast muscle has— upon other, it is called—
(A) More myoglobin (A) Extensor (B) Flexor
(B) More mitochondria (C) Abductor (D) Regulator
(C) More sarcoplasmic reticulum
(D) All the above ANSWERS
5. Which of the following is the con-
tractile protein of a muscle ?
(A) Tubulin ●●●
(B) Myosin (Continued from Page 687 )
(C) Tropomyosin
(D) All the above
6. In muscle, the contraction occurs
due to—
(A) Myosin
(B) Actin
(C) Both (A) and (B)
(D) None of these ●●●

C.S.V. / August / 2009 / 747


15. What is the name of cycle which
produces CO2, ATP, NADH and
FADH2?
(A) Oxidative carboxylation
(B) Oxidative phosphorylation
(C) Citric acid cycle
1. A type of dentition in which the 8. Sensory cells that occur in the (D) Glycolysis
teeth are replaced throughout the pits or canals of the lateral line
16. The cartilage making the lower
lifetime, is called— system of fishes ?
jaw of cartilaginous fishes, is
(A) Monophyodont (A) Neuromast called—
(B) Polyphyodont (B) Neurochrome (A) Dentary
(C) Diastemic (C) Neuroglia (B) Mentomeckelian
(D) None of the above (D) None of the above (C) Meckel’s cartilage
09. Adipocytes are— (D) Angulosplenial
2. Which of the following lipids do
not have fatty acids ? (A) Bones 17. In a DNA molecule, the—
(A) Steroids (B) Cartilage (A) Bases are covalently bonded
(C) Specialized connective to the sugars
(B) Cholesterol
tissues (B) Sugars are covalently bon-
(C) Phospholipids ded to the phosphates
(D) Nerves
(D) Both (A) and (B) (C) Bases are hydrogen bonded
10. Formation m -RNA from DNA is to one another
3. Which of the following hormone called—
(D) All of these are correct
initiates the preparation of the (A) Translocation
uterus for implantation of the 18. Ecotone is characterised by—
(B) Transcription
ovum ? (A) Transitional zone between
(C) Transduction two diverse communities
(A) Oxytocin
(D) Duplication (B) Terrestrial ecosystem
(B) Vasopressin
11. Diapedesis is— (C) Zone of transition between
(C) Progesterone water and land
(A) Formation of WBC
(D) HCG. (D) Forest ecosystem
(B) Bursting of WBC
4. Which brain wave pattern is 19. DNA nucleotide differences bet-
(C) Formation of Pus
ween organisms—
common in young children and (D) Passage of WBC (A) Indicate how closely related
sleeping adults ?
12. The chief function of utriculus in organisms are
(A) Alpha waves (B) Indicate that evolution occurs
humans is—
(B) Beta waves (C) Explain why there are phe-
(A) To perceive sound vibrations
(C) Theta waves notypic differences
(B) To maintain body equilibrium
(D) Delta waves (D) All of these are correct
(C) To perceive pressure
20. Process of urea formation in
5. In birds, a comb like structure is (D) To act as shock absorber humans is also known as—
present projecting into the cavity
of eye, is called— 13. Housefly has— (A) Krebs cycle
(A) Keel (B) Pecten (A) Chewing mouth parts (B) Hans Krebs cycle
(B) Piercing-sucking mouth parts (C) Nitrogen cycle
(C) Baleen (D) Ossicles
(D) Transamination
6. German measles is caused by— (C) Sponging mouth parts
(D) Siphoning mouth parts 21. Ends of long bones are covered
(A) Rubella virus with—
(B) Varicella virus 14. Which of the following conditions (A) Cartilage (B) Muscles
(C) Papilloma virus meet the Hardy-Weinberg equili- (C) Ligaments (D) Blood cells
(D) None of the above brium ?
22. In some animals, allantois is also
7. Quill feathers present on wings (A) The homozygous recessive related with—
are also called— phenotype is non-viable
(A) Storage of nitrogenous
(A) Down feathers (B) The dominant allele mutates wastes
(B) Barbules to a recessive allele (B) Blood formation
(C) Coverts (C) The alleles are sex-linked (C) Digestion
(D) Remiges (D) Mating is random (D) All the above

C.S.V. / August / 2009 / 748


23. Territoriality occurs as a result of 31. The protein which polymerizes to 40. Waste product of adenine and
predation— form microtubules is— guanine metabolism is—
(A) Predation (A) Actin (B) Chitin (A) Ammonia (B) Uric acid
(B) Parasitism (C) Tubulin (D) Ferritin (C) Urea (D) All these
(C) Competition 32. The centrum of a typical vertebra 41. The skin of certain sharks and
(D) Cooperation of a frog is— rays containing small knobs, is
24. During respiration, failure of (A) Procoelus called—
ventilation leads to— (A) Shagreen (B) Pulp
(B) Heterocoelus
(A) Decreased oxygen tension (C) Dentine (D) Ganoin
(C) Opisthocoelus
(B) Decreased carbon dioxide 42. Sympathetic nerves in mammals
tension (D) Acoelus
arise from—
(C) Carbonate tension 33. Which vitamin does not act as a (A) Sacral region
(D) Bicarbonate tension coenzyme ?
(B) Cervical region
25. Forest destruction leads to den- (A) Riboflavin (C) Thoraco-lumbar region
trimental effects on— (B) Pantothenic acid (D) None of the above
(A) Loss of a CO 2 sink
(C) Biotin
(B) Loss of biodiversity 43. Which one of the following is
(D) Folic acid correct about malaria ?
(C) Loss of possible medicinal
plants 34. Fovea centralis in eye, lacks— (A) Gametocytes fuse in mos-
(D) All of the above quito's stomach to form egg
(A) Rod cells
cells.
26. The association of sea anemone (B) Blood vessels
(B) Mosquito pick up gametocy-
and hermit crab is an example (C) Nerve fibres tes when sucking blood from
of—
(D) All the above an infected person
(A) Mutualism
35. Balbiani rings are the charac- (C) Merozoites invade red blood
(B) Commensalism
teristics of— cells and become tropho-
(C) Parasitism
zoites
(D) None of the above (A) Polytene chromosomes
(D) All the above are correct
27. Simple two neuron reflex arc (B) Lampbrush chromosomes
involves— 44. Osteoarthritis disease usually
(C) Sex chromosomes
(A) Sensory neuron associated with aging and is
(D) Ring chromosomes called—
(B) Spinal cord
(C) Effector neuron 36. The stage of animal embryonic (A) Communicable disease
development at which the gut (B) Degenerative disease
(D) All of the above
cavity and germ layers first
28. Tube feet are locomotory organs appear, is called— (C) Deficiency disease
of— (A) Involution (D) Allergy
(A) Jelly fish (B) Carp fish (B) Epiboly 45. Medullary cavity of which follo-
(C) Silver fish (D) Star fish (C) Gastrulation wing bone has yellow bone
29. Down syndrome— (D) Formative movement marrow ?
(A) Is always caused by non- (A) Long bones
37. Pebrine is a disease of—
disjunction of chromosome (B) Short bones
(A) Honey bee (B) Fish
21 (C) Spongy bones
(C) Silk worm (D) Lac insect
(B) Shows no overt abnormali- (D) All of the above
ties 38. Most primitive living mammals
which provide an evidence of 46. Stomach pain is felt through—
(C) Is more often seem in
children of elderly mothers organic evolution from geogra- (A) Interoceptors
after the age of thirty five phical distribution are found in— (B) Proprioceptors
and above (A) China (B) India (C) Teloceptors
(D) Both (A) and (C) are correct (C) Africa (D) Australia (D) None of the above
30. In the development of the human 39. Which one of the following pro- 47. Camel's hump conserve water
embryo, the ectoderm is respon- cesses make direct use of for longer time because it is
sible for the development of— oxygen ? composed of—
(A) Nervous system (A) Glycolysis (A) Muscular tissue
(B) Lens of eyes (B) Fermentation (B) Skeletal tissue
(C) Sweat glands (C) Krebs cycle (C) Areolar tissue
(D) All the above (D) Electron transport (D) Adipose tissue

C.S.V. / August / 2009 / 749


48. Black-water fever is caused by—
(A) Plasmodium falciparum
(B) Leishmania donovani
(C) Plasmodium ovale
(D) Plasmodium malariae
49. The transfer of genetic material
of one bacterium to another by
virus is called—
(A) Transduction
(B) Translation
(C) Transcription
(D) Replication
50. Charcot-Leydon crystals are
found in faeces of man during
infection of—
(A) Entamoeba histolytica
(B) Trypanosoma gambiense
(C) Ascaris
(D) All the above

ANSWERS WITH HINTS

(Continued on Page 790 )

C.S.V. / August / 2009 / 750


16. Ticks and mites are—
(A) Myriapods
(B) Arachnids
(C) Insects
(D) None of these
17. The malleus, incus and stapes of
1. Which of these provides phos- 8. The wishbone of a bird, which
ear ossicles of mammals are res-
pholipids dual properties ? support the flight muscles, formed
pectively modified bones of—
(A) Fatty acid non-polar tails from—
(A) Articular, hyomandibular and
(A) Fusion of the collar bones
(B) Charged phosphate head quadrate
(B) Ventral extensions of the
(C) Fatty acid polar tails vertebrae (B) Quadrate, articular and hyo-
(C) Anterior ribs mandibular
(D) Both (A) and (B)
(D) Posterior extensions of the (C) Articular, quadrate and hyo-
2. Which of the following is present larynx mandibular
in microsomes of liver cells and (D) Quadrate, hyomandibular
9. Synthesis of m -RNA and t-RNA
plays a part in detoxification ? and articular
occurs in—
(A) Cytochrome P-450 (A) G1 phase (B) G2 phase 18. Hashimoto disease is caused
(C) S-phase (D) All these when—
(B) Adenylate kinase
(A) Adrenal gland is destroyed
(C) Cytochrome C-reductase 10. Diapedesis is associated with— by autoimmunity
(D) Nucleoside diphosphokinase (A) Leucocytes
(B) Thyroid gland is destroyed by
(B) Urinary system autoimmunity
3. When atoms share one or more (C) Nervous system
electron pairs, the bond is— (C) Kidney is destroyed
(D) Peristalsis (D) Pancreas is destroyed
(A) Covalent bond
11. Absence of lack of ‘intrinsic fac- 19. Long chain molecules of fatty
(B) Ionic bond tor’ in diet causes— acids are formed by—
(C) Hydrogen bond (A) Pernicious anaemia (A) Polymerisation of 2 carbon
(D) All the above (B) Cooley’s anaemia compounds
(C) Aplastic anaemia (B) Decomposition of fats
4. Gooseflesh caused by erection
(D) None of the above (C) Polymerisation of glycogen
of skin papillae, as from cold,
shock, fright or fear, is called— 12. Which of the following is dinu- (D) None of the above
(A) Cutis anserina cleotide ? 20. Volkmann's canals are found in—
(B) Cutis aurantiasis (A) FAD (A) Bones of frog
(B) NAD (B) Bones of fowl
(C) Cutis hyperelastica
(C) Nucleic acid (C) Bones of rabbit
(D) Cutis marmorata
(D) Both (A) and (B) (D) Cartilage of rabbit
5. Which of these viruses become
13. Food energy passed at each 21. Bedbugs can survive during long
life-long residents of host cells ?
trophic level is approximately— time of starvation because—
(A) Herpes virus
(A) 10% (B) 30% (A) It stores glycogen
(B) Varicella virus (C) 50% (D) 90% (B) It convert uric acid to amino
(C) Rubella virus acid and thus it can use its
14. Which of the following is viral
(D) Both (A) and (B) excretory materials
disease ?
(C) Its life span is very long
6. Cuboid bone is associated with— (A) Impetigo
(D) It can minimise its require-
(A) Carpels (B) Warts
ments
(B) Metacarpels (C) Both (A) and (B)
(D) None of these 22. Which of the following is involved
(C) Metatarsals in DNA synthesis and cell divi-
(D) None of these 15. A non-essential hereditary factor sion ?
that may exist either free within a (A) Vitamin K (B) Folic acid
7. Which of these are a group of cell or in a state in which it is
viruses important in investigating (C) Vitamin E (D) Vitamin D
integrated in a chromosome, is
viral carcinogenesis ? known as— 23. Oxygen toxicity is related with—
(A) Papova viruses (A) Episome (A) Failure of ventilation of lungs
(B) Parvo viruses (B) Cosmid (B) Collapse of alveolar walls
(C) Adeno viruses (C) Both (A) and (B) (C) Blood poisoning
(D) Pox viruses (D) None of the above (D) None of the above

C.S.V. / August / 2009 / 751


24. For most people, dreaming 35. Which one of the following is not 44. Exotoxins are related with—
occurs during which stage of a Ketone body ? (A) Tetanus
sleep ? (A) Acetoacetic acid (B) Diphtheria
(A) Deep sleep (B) Succinic acid (C) Both (A) and (B)
(B) REM sleep (C) β-hydroxybutyrate (D) None of these
(C) Alpha sleep (D) Acetone 45. Degradative reactions—
(D) Slow wave sleep
36. German measles is caused by— (A) Are oxidation reactions
25. Latissimus dorsi muscles in
(A) Rubella virus (B) Are synthetic reactions
humans—
(B) Varicella virus (C) Requires a supply of NADPH
(A) Draws arm downward and
(C) Papiloma virus molecules
backward
(B) Draws legs forward (D) None of these (D) Both (A) and (C)
(C) Moves head 37. Antihistamine drugs relieve— 46. Which of the following is an
(D) Moves ankles example of bile acids ?
(A) Allergy
(A) Glycocholic acid
26. Trochanter is associated with— (B) Angina pectoris
(B) Taurocholic acid
(A) Femur (B) Humerus (C) Stroke
(C) Both (A) and (B)
(C) Ulna (D) Radius (D) Nephrites (D) None of these
27. Heparin is produced by— 38. When diminished blood volume 47. Which one of the following is not
(A) Liver cells is present in human body, it is related with the path of sound
(B) Nervous cells termed— vibrations ?
(C) Kidney cells (A) Hypovolemia (A) Auditory canal
(D) Spleen (B) Oligemia (B) Tympanic membrane
28. Podocytes are found in— (C) Oligohaemia (C) Semicircular canals
(A) Spleen (B) Pancreas (D) All the above (D) Cochlea
(C) Kidneys (D) Heart
39. Middle part of human sperm con- 48. Wolffian body is known as—
29. Which of the following stimulates tains—
erythropoiesis ? (A) Pronephros
(A) Nucleus
(A) Fe ++ (B) Mg++ (B) Mesonephros
(B) Mitochondria
(C) Ca ++ (D) Cu ++ (C) Metanephros
(C) Centriole
(D) Nucleus and mitochondria (D) None of these
30. Fovea centralis in eye lacks—
(A) Rod cells 40. Another term for adaptive evo- 49. Chaga’s disease is caused by—
(B) Blood vessels lution is— (A) Trypanosoma gambiense
(C) Nerve fibres (A) Clinal change (B) Trypanosoma cruzi
(D) All the above (B) Microevolution (C) Trypanosoma bruci
(C) Macroevolution (D) None of the above
31. Ichthyophis is a member of— (D) All the above
(A) Mollusca (B) Pisces 50. Inheritance of which of the follow-
41. Eggs having yolk in the centre ing is not Mendelian ?
(C) Amphibia (D) Annelida and cytoplasm in the peripheral (A) Plasmagene
32. Waste product of adenine and area, are called—
(B) Oncogene
guanine metabolism is— (A) Isolecithal
(C) Polygene
(A) Uric acid (B) Ammonia (B) Microlecithal
(D) All the above
(C) Urea (D) All these (C) Centrolecithal
33. Cis-trans test is related with— (D) Telolecithal
ANSWERS WITH HINTS
(A) Crossing over 42. Which of the following belongs to
(B) Heredity phylum Arthropoda ?
(C) Mutation (A) Gold fish (B) Silver fish
(D) All the above (C) Cuttle fish (D) Star fish

34. Parathion, malathion and femi- 43. Parathion, Malathion and Femi-
trothion insecticides belong the trothion insecticides belong to
group of— the group of—
(A) Carbamates (A) Carbamates
(B) Pyretheroids (B) Pyretheroids
(C) Organophosphates (C) Organophosphates
(D) Triazines (D) Triazines

C.S.V. / August / 2009 / 752


17. Which of the following is an
inactive enzyme precursor ?
(A) Polyglycoids
(B) Cholenzymes
(C) Activases
(D) Zymogens
1. Which of the following prevents (C) NADPH2
18. Restriction fragment length poly-
the mother’s immune response to (D) Solar energy morphisms (RFLPs)—
the developing embryo ?
9. What is the function of porphyrin (A) Identify individuals geneti-
(A) Trophoblast
in earthworm ? cally
(B) Blastocyst (A) To protect against harmful (B) Are the basis for DNA
(C) Umblical vein germs fingerprints
(D) Umblical artery (B) To help in respiration
(C) Can be subjected to gel
2. Auer’s bodies are present in the (C) To help in excretion
electrophoresis
cytoplasm of— (D) To protect from harmful ultra-
violet rays (D) All the above
(A) Myeloblasts
10. Which of the following is not a 19. Notochord is usually regarded
(B) Myelocytes
water soluble vitamin ? as—
(C) Monoblasts
(A) Tocopherol (A) Ectodermal
(D) All the above
(B) Niacin (B) Mesodermal
3. Milkman’s syndrome is charac- (C) Folic acid (C) Endodermal
terized by— (D) Ascorbic acid
(A) Failure of reabsorption of (D) Blastodermal
11. Which division of PNS is often
phosphates by renal tubules 20. In cladistics—
called ‘voluntary nervous sys-
(B) Failure of absorption of lac- tem’ ? (A) A clad must contain the
tose (A) Somatic common ancestor plus all its
(C) Failure of absorption of (B) Autonomic descendents
amino acids (C) Both (A) and (B) (B) Derived characters help
(D) None of these construct cladograms
(D) None of these
4. Certain aquatic mammals, such (C) Data for the cladogram is
12. FAD is derived from— presented
as seals, sealions, and walruses
(A) Thiamin (B) Riboflavin (D) All the above
are included in the order—
(C) Pyridoxine (D) Folic acid
(A) Pinnipedia (B) Fissipedia 21. Bones enlarge by—
(C) Hyracoidea (D) Cetacea 13. The final hormonal stimulus lead- (A) Auxentic growth
ing to ovulation in human female
5. Failure of an organ or part to (B) Multiplicable growth
is provided by—
develop or grow, is called— (C) Accretionary growth
(A) Estrogen
(A) Agenesia (B) Agenitalism (D) Appositional growth
(B) Progesterone
(C) Ageism (D) Agerasia 22. Free-living Planaria is a—
(C) Follicle stimulating hormone
6. Group of related species forming (D) Luteinizing hormone (A) Roundworm
fertile hybrids are called— (B) Flatworm
14. Autoimmune thyroiditis is asso-
(A) Coenospecies (C) Tapeworm
ciated with—
(B) Subspecies (D) Fluke
(A) Hashimoto’s disease
(C) Sibling species
(B) Grave’s disease 23. Bilharziasis is a parasitic disease,
(D) None of these caused by—
(C) Goitre
7. Olfactory organ of a snake is— (D) Acromegaly (A) Schistosoma
(A) Jacobson’s organ (B) Taenia solium
15. Convulsions in infants is caused
(B) Johnston’s organ due to deficiency of— (C) Fasciola hepatica
(C) Organ of Bojanus (A) Iodine (B) Vitamin B 6 (D) Enterobius vermicularis
(D) None of these (C) Vitamin D (D) Vitamin C 24. Guillain-Barre syndrome is rela-
8. Which of the following is source ted with—
16. A cell-coded protein, that is
of energy to an ecosystem ? formed in response to viral infec- (A) Acute polyneuritis
(A) Heat liberated during respi- tion, is— (B) Infectious polyneuritis
ration (A) Antigen (B) Interferon (C) Landry’s paralysis
(B) ATP (C) Histone (D) Antibody (D) All the above

C.S.V. / August / 2009 / 754


25. Which of the following is the only 33. Sum of constructive processes in 42. Which part of human brain is
amino acid metabolized by the body cells is called— associated with integration of
brain ? (A) Catabolism sympathetic and parasympathe-
(A) Alanine (B) Anabolism tic activities ?
(B) Glutamic acid (C) BMR (A) Hypothalamus
(C) Histidine (D) All the above (B) Cerebrum
(D) Glutanine (C) Medulla oblongata
34. Which of these is present in
(D) Neopallium
26. Electron micrographs following human buccal cavity ?
freeze-fracture of the plasma (A) Trypsin (B) Ptyalin 43. Iter or Aqueduct of sylvius is
membrane indicate that— (C) Lipase (D) Pepsin located between—
(A) The membrane is a phos- (A) Lateral ventricles
pholipid bilayer 35. Which layer of the heart wall
(B) Optocoels
consists cardiac muscles ?
(B) Some proteins span the (C) III and IV ventricles
membrane (A) Endocardium
(D) None of these
(C) Protein is found only on the (B) Myocardium
surface of the membrane (C) Epicardium 44. Autologous blood tranfusion is—
(D) Glycolipids and glucoproteins (D) All of these (A) Use of patient’s own blood
are antigenic (B) Use of patient’s parental
36. Human nerve cells originate from
blood
27. Enzymes which catalyse reac- the embryonic—
(C) Use of patient’s children
tions involving electron transfer, (A) Ectoderm
blood
called— (B) Ectoderm and mesoderm
(D) All the above
(A) Transferases (C) Endoderm
(B) Hydrolases (D) Mesoderm 45. Which type of tissue forms the
(C) Ligases thin surface for gas exchange in
37. Which of the following is called lungs ?
(D) Oxidoreductase incomplete proteins ? (A) Epithelial (B) Connective
28. Arytenoid cartilages are found (A) Most animal proteins (C) Nervous (D) Muscular
in— (B) Most plant proteins
46. Unmyelinated axons in spinal
(A) Sternum (B) Hyoid (C) Both (A) and (B)
cord are present in—
(C) Nose (D) Larynx (D) None of these
(A) White matter
29. Which of the following animals are 38. Which of the following syndrome (B) Gray matter
devoid of respiratory, excretory is associated with easily dislo- (C) Both (A) and (B)
and circulatory organs ? cation of body joints ?
(D) None of these
(A) Tapeworms (A) Ehlers-Danlos syndrome
47. Carcinoid syndrome is associated
(B) Sponges (B) Tay-Sachs syndrome with—
(C) Liver flukes (C) Marfan’s syndrome (A) Serotonin
(D) None of these (D) None of these (B) Bradykinin
39. Reticulocytes are— (C) Prostaglandins
30. Active transport—
(A) WBCs (D) All of these
(A) Requires a carrier
(B) Moves a molecule against (B) Immature RBCs 48. Which of the mitosis stage provi-
its concentration gradient (C) Blood platelets des best opportunity for prepar-
(D) Lymphocytes ing human karyotype ?
(C) Requires supply of energy
(A) Anaphase (B) Metaphase
(D) All the above are correct 40. Which of the following division of
(C) Prophase (D) Telophase
peripheral nervous system is
31. Parasympathetic effect— often called ‘voluntary nervous 49. Castle’s intrinsio factor helps in—
(A) Lowers blood pressure system ? (A) Absorption of cyanoco-
(B) Slows heart rate (A) Somatic balamin
(C) Promotes digestion (B) Autonomic (B) Nervous stimulation
(D) All the above (C) Both (A) and (B) (C) Cardiac reflex
32. Enzyme enterokinase converts— (D) None of these (D) All the above
(A) Proteins into peptides 41. Meroblastic cleavage refers to 50. Bright’s disease is associated
(B) Caseinogen into casein which type of division of eggs ? with—
(C) Trypsinogen into trypsin (A) Incomplete (B) Spiral (A) Kidneys (B) Liver
(D) Pepsinogen into pepsin (C) Complete (D) Horizontal (C) Lung (D) Bone

C.S.V. / August / 2009 / 755


Introduction (iv) Soil water or solution—Soil water is held by
capillary and absorptive forces between and around the
Edaphic factors take in account the structure and
surface of soil particles. Actually soil water is a dilute solu-
composition of soil with its physical and chemical charac-
tion of various organic and inorganic compounds, which
teristics. The study of soil (Latin = Solum) is called pedo-
become available to plants as mineral nutrients. Water in
logy or edaphology, which is important for geology,
the soil comes mainly through infiltration of precipitated
mineralogy, paleobotany, paleozoology and petrology.
water (rain, sleet, snow and hail) and irrigation, whereas it
Soil is a natural habitat for plants, animals and micro-
is lost from the soil chiefly through evaporation, percola-
organisms. It may be defined as “any part of earth's
tion and transpiration.
crust in which plants root grow.” However, soil is not
merely mineral matter, but a complex of several other Soil Formation (Pedogenesis)
types of components. Thus biologically the soil may be
defined as “the weathered superficial layer of the C. F. Marbut states that “a mature soil is one that has
earth's crust in which the living organisms grow and assumed the profile features characteristic of predominant
also release the products of their activities, death and soils on the smooth uplands within the general climatic
decay.” and botanic regions in which it is found.
Soil is a stratified mixture of organic and inorganic
Composition of Soil materials, both of which are decomposition products. The
From the above classical definition of soil it is evident soil forming rocks are the parent material from which
that soil is, thus, not merely a mineral particles, but it has mineral constituents of soil are derived by fragmentation
also a biological system of living organisms as well as or weathering. Organic components of soil are formed
some other components. Therefore, it is preferred to call it either by decomposition (or transformation) of dead
a soil complex, which includes the following categories of remains of plants or animals or through metabolic acti-
components— vities of living organisms present in the soil.
(i) Mineral matter—The matrix of mineral particles Soil-forming rocks—There are two basic kinds of
derived by varying degrees of break down of the parent soil-forming rocks as given below—
material–rock occur in the soil. The minerals represent (i) Sedimentary rocks—These are formed by
about 90% of the total weight of the soil. deposition of weathered minerals which are derived from
(ii) Soil atmosphere—Soil atmosphere occupies igneous rocks.
the pore space between soil particles, which is not water– (ii) Igneous rocks—These are formed due to cooling
filled. The soil atmosphere differs from the above ground of molten magma or lava.
atmosphere as it is normally lower in oxygen and higher in
carbon dioxide content. Chemical Nature of Minerals of Soil-forming
(iii) Soil organic matter or humus—Soil organic Rocks
matter or humus are partially decayed organic compo- Rocks are the chemical mixture of various kinds of
nents derived from long and short-term addition of mate- minerals. The chemical nature of most common and
rial from organisms growing above and below the ground abundant minerals of soil-forming rocks are given in the
that is microorganisms, plants and animals. following table—

Table : Chemical Composition of Some Common Soil


Dokuchayev’s View
Minerals
● According to Dokuchayev, a famous Russian pedologist,
“the soil is a result of the actions and reciprocal influences Minerals Chemical Constituents
of parent rocks, climate, topography, plants, animals and
A. Sand and Silt minerals
age of the land.
1. Quartz or silica SiO2
● Hence, soil can be represented by the following formula :
2. Feldspars
S = (g.e.b) Δt
(a) Orthoclase K2Al2Si6O16
where, S = Soil
(b) Calcium feldspar CaAl2Si2O8
g = Geology
(c) Plagioclase NaAlSi3O8
e = Environment
3. Amphibole (Mg, Fe)7 (Si4O11) 2 (OH)2
b = Biological influences
4. Calcite; magnesite; CaCO3, MgCO3 and
t = Time and dolomite (CaCO3, MgCO3)

C.S.V. / August / 2009 / 757


5. Micas bacteria, cyanobacteria and lichens dissolve the mineral
(a) Biotite K, Mg, Fe, Al silicate components of the rocks. These metals dissolved with
(b) Muscovite K (OH)2 Al2 (AlSi3) O10 organic products of microbial activities are known as
6. Iron oxides
chelates.
(a) Limonite FeO (OH), X H2O (iv) Hydration—Reversible changes occur due to
(b) Haematite Fe2O3
water absorption by haematite to limonite, (FeO3
(c) Magnetite Fe2O4 Fe 2O3 . 3 H2O), as a result the rocks swell up and causes
7. Pyroxene (Mg, Fe) SiO4
disruption.
8. Olivine and serpentine (Mg, Fe)2 SiO4 (v) Carbonation—Various chemicals produced in
B. Clay minerals
atmosphere and by metabolism of microorganisms bring
about carbonation. For example, reversible change of
1. Montmorillonite (Ca, MgO) Al 2O3,
CaCO3 to Ca (HCO3)2 leads to solution loss of limestone
5 SiO3, 5 H2O
or disruption of calcium carbonate cemented rocks as
2. Kaolin Al2O3, 2 SiO2, 2 H 2O hydrogen carbonate is more soluble than the carbonate.
(vi) Oxidation-reduction—Certain chemicals bring
Process of Soil Formation about oxidation-reduction reactions, such as reversible
(1) Weathering of Soil Forming Rocks—Soils are change of Fe3+ to Fe2+, cause disruption of rocks,
derived from parent rocks by the process called weather- because Fe3+ is less soluble than Fe 2+.
ing. Formation of soil is initiated by disintegration or (C) Biological weathering—According to Jacks
weathering of parent rocks by certain physical, chemical (1965-66), a number of microorganisms ( e.g., bacteria,
and biological agents as a result, soil-forming rocks are fungi, nematodes, protozoans, etc.) lichens and mosses
broken down into small particles called regoliths, which are early colonizers which transform the rock into a
finally develop into mature soil by pedogenesis weathering dynamic system, storing energy and synthesizing organic
is accomplished by three types of forces—physical, material. Their activities alter the chemical composition
chemical and biological. and physical structure of the rock. For example, lichens
(A) Physical weathering—It takes place due to the are present in the initial stages of biological succession
following processes— and their growth may cause cracking or flaking, exposing
(i) Wetting-drying—Wetting-drying causes the dis- greater area of rock to further weathering. Lichens as well
ruption of layer or lattice structure of the rock, which as mosses extract mineral nutrients such as Ca, Mg, S, P,
swells upon wetting and contract on drying leading to Al and Si from the rock. These elements are combined
disintegration of rock, which provides more aeration. with organic complexes and eventually return to develop-
(ii) Heating-cooling—It causes the disruption of ing soil when vegetation decomposes. Joffe states that
heterogenous crystalline rocks in which inclusions have there is no biogeochemical weathering. According to him,
differential coefficients of thermal expansion. it is either chemical or physical weathering by biological
agencies.
(iii) Sand blast—In arid and desert conditions the
rocks are disrupted by physical action of wind and sand.
Products of Weathering
(iv) Freezing—It causes the disruption of porous,
lamellar or vesicular rocks by frost shatter due to expan- ● The surface rocks of the earth are weathered and as a
result of weathering, small particles of parental materials
sion of water.
are formed.
(v) Glaciation—At mountain tops, ice formation takes ● The soil which is formed by weathering of rocks is called
place mostly in the winter season. When the summer embryonic or primary soil.
approaches, ice starts melting and glaciers (huge sliding ● The soil which develops in situ above parent bedrocks is
masses of ice) move downwardly on the slops. In the known as sedimentary and residual soil.
glacier movement, the rocks are corroded and finally ● In a few places soil material comes from accumulated
broken into sand particles. organic matter as peat.
(B) Chemical weathering—Chemical weathering, ● Soil material transported from one area to another is called
which brings about disappearance of original rock mine- eolian or loess.
rals either completely or partly, includes the following pro- ● Transported or secondary soils are those which are
cesses— carried to other places by carriers like gravity, glacier,
water and wind.
(i) Solution—Certain mobile components of rocks,
such as calcium sulphates and chlorides are simply remo- (2) Mineralization and Humification—Break down
ved by agents like water (solution) making the rock porous of organic debris into humus is accompanied by decom-
and hence liable to further disintegration. position and finally by mineralization. Higher organisms in
(ii) Hydrolysis—During hydrolysis, components like the soil consume fresh material and leave partially
alumino silicates of rocks break down and potassium or decomposed products in their excreta. This is further
silicon are washed out which give rise to simpler mineral decomposed by bacteria and fungi into various com-
matter like clay alumino-silicates. pounds of carbohydrates, proteins, fats, lignin and resins.
(iii) Chelation—Certain chemical exudates, produced These compounds are broken down into simpler products
through biochemical activities of microorganisms like such as carbohydrate, water salts and minerals. This

C.S.V. / August / 2009 / 758


process is called mineralization. The residual amorphous (B) Devoid of plant nutrients
incompletely decomposed black coloured organic matter (C) Fit for agriculture
which undergoes mineralization is called humus. The (D) None of the above
process of humus formation is called humification. The
soils of deserts in which vegetation is scanty and new 2. Soil is composed of—
soils freshly formed from rocks, apparently contain very (A) Air + water + minerals
little humus, while mature soil with vegetation, the most. (B) Water + organic matter + minerals
(3) Organo-mineral Complex Formation—In the (C) Air + water + organic matter + minerals
final stage of pedogenesis, colloidal particles, which are
formed as a result of weathering, humification and mine- (D) Water + organic matter
ralization, accumulate and may aggregate into crumbs 3. The chemical weathering involves the—
or concretions. About 60–70% colloidal humus particles
become associated with mineral particles to form organo- (A) Solution (B) Hydrolysis
mineral complexes. The crumbs of organo-mineral com- (C) Oxidation (D) All of these
plexes, according to Wallwork (1970), are formed by two
mechanisms—electrochemical bonding and cement- 4. Humus is an example of—
ing. (A) Fertilizer
Aggregation of negatively charged colloidal clay (B) Crystalloid
and/or humus particles of water molecules and metallic (C) Component of soil structure
ions particularly calcium takes place in electrochemical (D) Organic colloids
bonding method of crumb formation. The cementing
mechanism involves the action of substances adsorbed on 5. Sequence of humification and mineralization is—
the surface of soil particles which effectively make them (A) Dead organic matter → litter → duff → humus
compact. The crumbs increase the total pore space in the (B) Minerals → humus → litter → duff
soil, providing good aeration and drainage. Some soluble (C) Dead organic matter → duff → litter → minerals
organic compounds are removed from the top soil by
water which percolates downwardly through humus mixed
→ humus
soil. (D) Humus → minerals → litter → duff

OBJECTIVE QUESTIONS ANSWERS


1. Eroded soils are—
(A) Richer in plant nutrients ●●●

Just Released

By : Dr. Vijay Agarwal Translated by : Nidhi Sahni


Code No. 1647 Price : Rs. 90/-
This book contains :
☞ Complete coaching in the form
of a book ☞ Practical approach to
crack the CSE ☞ How to prepare
G.S., Optional subjects, Essays &
General Hindi ☞ How to face the
interview and its competitive envi-
ronment ☞ Clear and complete
information about the traditions of
competitive exams ☞ Last minute
preparation/ revision mantras.
HINDI EDITION Code No. 215 Rs. 110/-
● E-mail : publisher@upkar.in
UPKAR PRAKASHAN, AGRA-2 ● Website : www.upkar.in

C.S.V. / August / 2009 / 759


Introduction materials consist of compounds that are immediately
Biochemistry has traditionally been considered usable by the crops to which they are applied. Others are
‘organic’ and in fact much of the biochemistry involves quickly converted within the soil to forms that can be
organic or carbon chemistry. It has been known for a long assimilated. Some fertilizer chemicals are specifically
time, however, that in plants many other elements in the designed to dissolve slowly or to delay reaction within the
form of macro or micronutrients are also present. Mate- soil and, therefore, prolong the release of easily absorbed
rials are added to the soil, or applied directly to crop compounds to provide sustained feeding over the growth
foliage, to supply elements needed for plant nutrition. cycle of the plants.
These materials may be in the form of solids, semisolids, Aqueous solutions of urea, ammonia and ammonium
slurry suspensions, pure liquids, aqueous solutions or nitrate (UAN solutions) are used directly by the farmers
gases. as well as in the preparation of granular N-P-K products
by mixing with other materials, such as normal superphos-
Fertilizers phate and triple superphosphate. UAN solutions are also
Crop requirements of fertilizer components could be spread directly by field application or used to prepare
satisfied by the spreading of individual materials for each complete N-P-K fertilizer solutions or suspensions.
element deficient in the soil. However, economy favours
the single application of a balanced mixture that satisfies Fertilizing
all nutritional needs of a crop. Many commercial fertilizers,
● Addition of elements or other materials to the soil to
therefore, contain more than one of the primary fertilizer
increase or maintain plant yields is known as fertilizing.
elements. The chemical elements, such as nitrogen (N),
phosphorus (P) and potassium (K) are the macronutrients, ● Fertilizers may be organic or inorganic.
or primary fertilizer elements, which are required in ● Organic fertilizers are usually manures and waste mate-
greatest quantity. rials which in addition to providing small amounts of growth
elements also serve as conditioners for the soil.
Sulphur (S), calcium (Ca) and magnesium (Mg),
called secondary elements, are also necessary to the ● Methods of applying fertilizers vary widely and depend on
health and growth of vegetation, but they are required in such factors as kind of crop and stage of growth, applica-
tion rates, physical and chemical properties of the ferti-
lesser amounts compared to the macronutrients. The
lizers and soil type.
other elements of agronomic importance, called macro-
nutrients and provided for plant ingestion in small (trace) ● Two basic application methods of fertilizers used are bulk
spreading and precision placement.
amounts, include boron, zinc, molybdenum, cobalt,
copper and iron. All these fertilizer elements along with ● Liquid fertilizer of the high pressure type ( e.g., anhydrous
other chemical elements, occur naturally in agricultural ammonia) is usually regulated by valves or positive dis-
placement pumps.
soils in varying concentrations and mineral compositions
which may or may not be in forms readily accessible to ● Non-volatile fertilizer solutions are often pumped into the
root systems of plants. supply lines of irrigation systems to allow simultaneous
fertilization and irrigation.
Many commercial fertilizers contain more than one of
the primary fertilizer elements. The composition of ferti-
lizer mixtures, in terms of the primary fertilizer elements,
Biofertilizers
are identified by an N-P-K code : N denotes elemental Biofertilizers are the organisms which enrich the soil
nitrogen; P denotes the anhydride of phosphoric acid in nutrients due to their biological activity. To combat the
(P2O5); K denotes the oxide of potassium (K2O). All are ill effects of chemical fertilizers, biofertilizers have now
expressed numerically in percentage composition, or units been introduced for the pollution-free and better growth of
of 20 lb each per short ton (10 kg per metric ton) of the plants of useful aspects. The chief sources of biofer-
finished fertilizer as packaged. Formula 8-32-16 thus tilizers are blue-green algae (cyanobacteria), fungi and
contains a mixture aggregating 8 wt% N in some form of bacteria.
nitrogen compounds, 32 wt% P 2O5 in some form of phos-
phate and 16 wt% K2O in some form of potassium com- 1. Algae as Biofertilizers
pounds, to give a product with a total of 56 fertilizer units. Early in the history of agriculture in coastal Asia the
The commercial N-P-K formulas totals 100% plant value of sea weeds in fertilizing the soil was discovered.
nutrients because the formulas indicate only the nutrient Long before the recognition of the potash content, sea
portions of the primary-element compounds. weeds were employed as fertilizers by the farmers. Not
Growing plants can assimilate only fertilizer elements only as the fertilizers, but also the water-holding capacity
in the combined state of inorganic compounds that are of fragments of the algae in the soil proved effective.
amenable to osmotic absorption. Many modern fertilizer Furthermore, the yield of paddy is increased substantially

C.S.V. / August / 2009 / 760


when paddy field is inoculated with nitrogen-fixing blue- 4. Micorrhiza as Biofertilizer
green algae, such as Anabaena oryzae , Tolypothrix Mycorrhiza is the symbiotic relationship between a
tenius, Calothrix, Cylindrospermum bengalense, and fungus and the roots of higher plant. Mycorrhiza performs
Nostoc commune. the following functions :
Blue-green algae (cyanobacteria) secrete growth pro- ● It enhances phosphate nutrition in plant.
moting substances like IBA, NAA, IAA and various pro- ● It enhances water and various other nutrient up-
teins and vitamins. They add sufficient amounts of organic take.
matter in the soil. They can grow and multiply under wide ● It enhances greater plant vigour (heterosis),
pH range of 6·5–8·5. Therefore, they can be used as the
growth and yield.
possible tool to reclaim saline or alkaline soil because of
Mycorrhiza is of two types—ectomycorrhiza and
their ameliorating effect on the physio-chemical properties
endomycorrhiza. Ectomycorrhiza increases water and
of soil.
nutrient intake by plants. It occurs in plants like Eucalyp-
2. Bacteria as Biofertilizers tus, Ficus, Oak, Pine, etc. It absorbs and stores nitrogen,
potassium, phosphorus and calcium in fungus. It also
Rhizobium is one of the best fertilizers of leguminous
converts complex organic molecules into simpler and
plants. Rhizobium is a bacterium living in the root nodules
easily absorbable form. Endomycorrhiza is important in
of leguminous plants and making symbiotic association
phosphate nutrition of plants. It is also called Vesicular
with them. The root cells of leguminous plants possess a
Arbascular Mycorrhiza (VAM). VAM has capacity to
purple coloured pigment known as leghaemoglobin in
penetrate even the cortical cells.
which Rhizobium floats to fix atmospheric nitrogen.
Clostridium, Azotobacter, Aerobacter and Methano-
Other Benefits of Mycorrhiza to Plants
bacterium are free-living bacteria which fix atmospheric
nitrogen. Rhizobial biofertilizer can fix 50–150 kg/Na/ ● VAM fungi enhance water uptake in plants.
ha/yr. ● They play a key role for selective absorption of immobile
(Cu, Zn and P) and mobile (Fe, Mn, Cl, N, Br, S, Ca and
3. Pteriodophytes as Biofertilizers K) elements to plants. These are available to plants in less
amount.
In the cavities of leaves of certain aquatic pterido- ● They increase resistance in plants and with their presence
phytes, such as Azolla , a large number of plants of the effects of pathogens and pests on plant health is
Anabaena ( Anabaena azollae, a blue-green alga) are pre- reduced.
sent which have the capacity to fix atmospheric nitrogen ● Some of the trees like pines cannot grow in new areas
and make it available to Azolla (an aquatic pteridophyte). unless soil has mycorrhizal inocula because of limited or
Azolla supplies nitrogen, increases organic matter and coarse root hairs.
fertility in soil and shows tolerance against heavy metals. ● They increase the longevity of feeder roots, surface areas
Dr. P. K. Singh has done an outstanding work on mass of roots by forming mantle and spreading mycelia into soil
cultivation of Azolla and its uses as biofertilizer in rice and and, in turn, the rate of absorption of major and minor
nutrients from soil resulting in enhanced plant growth.
other crop fields.

OBJECTIVE QUESTIONS
1. An aquatic fern used as bioferti- (C) Manures (C) Supplies organic matter
lizer is— (D) All of the above (D) None of the above
(A) Azolla 4. A legume having symbiotic asso- 7. Leaves of Azolla possess the
(B) Marsilea ciation with Rhizobium and Aero- colonies of—
(C) Salvia rhizobium is— (A) Anabaena
(D) None of the above (A) Sebania rostrata
(B) Rhizobium
(B) Sebanea aculeata
2. Manure containing a mixture of (C) Both (A) and (B)
(C) Crotolaria juncea
cattle dung and crop residues is (D) Azotobacter
(D) None of the above
known as—
5. Fertilizers applied to crop plants 8. Biofertilizer present in the roots
(A) Green manure of legume plants is—
pollute—
(B) Farm yard manure (A) Chiefly water resources (A) Azospirillum
(C) Organic manure (B) Chiefly atmosphere (B) Anabaena
(D) None of the above (C) Soil resources (C) Rhizobium
3. Materials of biological origin that (D) Soil resources and water (D) All of the above
are applied commonly to main- resources
tain and improve soil fertility are 6. A green manure— ANSWERS
termed— (A) Protects soil against erosion
(A) Nitrogenous fertilizers and leaching
(B) Biofertilizers (B) Supplies additional nitrogen ●●●

C.S.V. / August / 2009 / 761


Introduction and Occurrence ● The clamp connections are usually formed on the
● Club fungi (division—Basidiomycota), which have terminal cells of the hyphae of the secondary myce-
septate hyphae, include the familiar mushrooms lium.
growing on lawns and the shelf or bracket fungi found ● The clamp connections by some mycologists are con-
on dead trees. sidered to be homologous to the hooks of ascoge-
● Less but well known club fungi are puffballs, bird’s nous hyphae of the Ascomycetes.
nest fungi and stinkhorns. ● Presence of hooklike clamp connection is a safe
● This big group of fungi containing about 16000 spe- criterion for distinguishing a secondary or dikaryotic
cies includes both saprophytic and parasitic species. mycelium from the primary or monokaryotic myce-
● Basidiocarps, also called fruiting bodies, contain the lium.
basidia, club-shaped structures which produce basi-
diospores and from which this division takes this
name (club-fungi).

Mycelium
● The mycelium generally is a weft of interlacing and
anastomosing hyphae. In a few genera, however, the
mycelial hyphae run parallel to one another and get
bundled together to form definite and conspicuous
thick cords called rhizomorphs.
● Most of the Basidiomycetes are heterothallic. It
means primary or homokaryotic mycelium in them is
of two distinct strains which are called plus (+) and A B C D E F
minus (–) strains.
Fig. (A–F) Basidiomycetes. Diagram illustrating the
● The well-developed filamentous mycelium consisting formation of clamp connections.
of a mass of branched, septate hyphae generally
spreads in a fan-shaped manner. Sexual Reproduction
● The mycelium of Basidiomycetes passes through ● Club fungi usually reproduce sexually. However,
three distinct stages—primary, secondary and tertiary development of sex organs, the male antheria and
before the fungus completes its life cycle. female ascogonia are universally absent throughout
the class.
Clamp Connection
● The rudimentary differences in sex, shown at the time
● The dikaryotic cell divides repeatedly by conjugate
of sexual fusion, are designated as plus (+) and
divisions to give rise to a secondary or dikaryotic minus (–) signs. These signs are called sexual
mycelium. During nuclear divisions of the dikaryotic strains. Either of these mycelia, if cultured artificially,
cell special structures called clamp connections are remains sterile. They form no fructifications.
formed.
● Fructifications are formed only if two mycelia of oppo-
Diplodisation (Dikaryotisation) site strains come in contact. The sexual process
being extremely simplified, consists of three funda-
The process by which the primary mycelium is converted mental processes such as karyogamy, sexual fusion
into secondary mycelium or dikaryotic mycelium is called or plasmogamy and meiosis.
diplodisation or dikaryotisation. It may take place by the
following methods— (i) Karyogamy
(i) By hyphal fusion—In this case, fusion occurs between ● The terminal binucleate or dikaryotic cells of the
the vegetative cells of two neighbouring hyphae. hyphae of the secondary mycelium develop into basi-
(ii) By the fusion a germinating basidiospore and a diploid dia.
cell of the basidium.
(iii) By the fusion between the two haploid cells of opposite
● The two nuclei in the dikaryotic cell fuse. This fusion
strains of the basidium. is called karyogamy. The resultant diploid fusion
(iv) By the fusion between germinating odium of one nucleus is called a synkaryon.
strains of the basidium. ● The young basidium containing the synkaryon is
(v) By conjugation of basidiospores—In this case two called probasidium which represents the transitory
basidiospores of opposite strains meet and conjugate. diplophase.

C.S.V. / August / 2009 / 762


Asexual Reproduction
Brand spores
(i) By Conidia :
Dikaryotic Diploid
● The production of conidia is not so common occur-
rence in the Basidiomycetes. They are formed in the
rusts, smuts and some other Basidiomycetes.
C ● In smuts, conidia are budded off from the basidio-
B
spores and the mycelium.
A ● The conidia in Basidiomycetes are produced by the
Young dikaryotic mycelium. They serve to propagate the
basidium
dikaryophase in the life cycle.
(ii) By Oidia :
Basidio-
spore ● Oidia are hyaline, small and thin-walled unicellular
Brand sections or fragments of the mycelium.
F spore
● Oidia may be uni—or binucleate as whether they are
D E
produced by the breaking up of the primary or secon-
Fig. : (A–E) Basidiomycetes. Ustilago sp. (A) Dikaryotic dary mycelium.
hypha forming brand spores; (B–C) brand spores ● Oidia serve a double function; they may either germi-
(dikaryotic and diploid); (D) Germinated diploid
nate to form primary mycelia or bring about diplodi-
brand spore to form an epibasidium with four
haploid nuclei arranged in a row; (E) mature four-
sation. In the latter case the germinating odium acts
celled epibasidium bearing basidiospores; (F) as a spermatium and fuses with the somatic hyphal
Brand spore of Ustilago budding in a nutrient wall of opposite strain.
solution (after Brefeld). ● In some species, the oidia are segmented from spe-
(ii) Plasmogamy cial, short lateral hyphal branches called the oidio-
phores.
● Plasmogamy means the union of two protoplasts
whereby the sexual nuclei of opposite strains come (iii) By Fragmentation
close together in a pair within the same cell. ● Any part detached from parent cell containing coni-
● In Basidiomycetes, plasmogamy is achieved either by diophores become able to form a new mycelium
somatogamy or by spermatization. (plant body).
● The two somatic hyphae of the primary mycelia of
opposite strains come in contact and lie side by side Basidiocarp and Basidium
in the fusion cell. This sexual union or plasmogamy
by fusion of somatic cell is called somatogamy. ● In higher Basidiomycetes (class = Homobasidiomycetidae)
the secondary mycelium develops fruiting bodies called
● In the homothallic species plasmogamy occurs by the basidiocarps.
formation of tubular connections between the somatic
cells of the same mycelium. ● The basidiocarps are usually massive aerial sporophores
which bear basidia.
● Plasmogamy by the union of a spermatium with a
receptive hyphae (female organ) is known as sper- ● The basidia, which are characteristic reproductive struc-
tures of Basidiomycetes, are of two types in general, the
matization.
holobasidium and phragmobasidium.
● Plasmogamy by spermatization exclusively occurs in
● Holobasidia are aseptate and thus unicellular, whereas
the rusts which produce numerous tiny, uninucleate,
phragmobasidia are septate basidia.
nonmotile spore-like bodies called spermatia. They
are formed in flask-shaped organs, the spermato- ● Holobasidia are characteristic of most of the Basid-
gonia. iomycetes particularly the gilled (gill-containing) or fleshy
fungi.
(iii) Meiosis :
● Basidia are developed in a palisade-like layer on the
● The synkaryon in the probasidium undergoes two basidiocarp. This fertile layer is called hymenium.
successive nuclear divisions which constitute meio-
sis. ● Basidia produce basidiospores, which are often wind-
blown, when they germinate, the new haploid mycelia are
● Meiosis restores the haploid condition in the life formed.
cycle.
● The septate basidium or phragmobasidium is typical of
● Both karyogamy and meiosis take place in the basi- rusts and smuts which usually does not form any fructifica-
dium at different stages of development. tion or basidiocarp.

C.S.V. / August / 2009 / 763


Rusts and Smuts are Parasitic Club Fungi
Epiba-
sidium ● Rusts and smuts are club fungi that parasitize cereal
crops, such as corn, wheat, rye and oats.
● They do not form the basidiocarps.
● Some smuts enter seeds of the host and exist inside
the host plant, becoming visible only near maturity.

Epibasidium
Hypobasidium ● In corn smuts, the mycelium grows between the corn
kernel and secretes substances that cause the deve-

Hypobasidium
lopment of tumors on the ears of corn.
● The life cycle of rusts, which may be particularly com-
plex, often requires two different plant host species to
A
C complete the life cycle.
B
Basidiospores ● Black stem rust of wheat uses barberry leaf as an
Sporidia alternate host.

Differences Between Rusts and Smuts


Migrating Rusts Smuts
nucleus
Sterigma 1. Rusts may be heteroe- 1. All smuts are autoe-
cious and autoecious. cious.
For example, wheat
rusts are heteroecious
Epibasidium
and others are autoe-
Brand spore cious.
D 2. The rusts are intercellu- 2. The smuts may be
E
lar and obtain their intercellular or intracel-
Fig. : (A–E) Basidiomycetes. Different types of basidia. (A) nutrition by means of lular (Ustilago maydis ).
Stichobasidial type; (B) Chiastobasidial type; (C) Tuning-fork
haustoria. Haustoria are absent.
type; (D) Holobasidium; (E) Stichobasidial type with a terminal
cluster of septate, sickle-shaped sporidia. 3. Clamp connections on 3. Clamp connections are
the secondary myce- common.
Pileus
lium are rare.
4. The dikaryotic mycel- 4. It produces only one
Gills
ium produces three kind of binucleate
Pores
Annulus kinds of spores : spores called the smut
uredospores and tele- spores which are com-
Stipe utospores on the pri- parable to the teleuto-
Volva mary host and aecio- spores of rust fungi.
A Mycelium spores on the alternate
B
host.
5. Teleutospores are two- 5. The brand spores
celled, stalked; and (teleutospores) are uni-
each cell is binucleate. cellular and binucleate.
D
6. The teleutospores are 6. Smut spores are for-
Pileus
C developed from the med from the inter-
terminal cells of the calary cells.
mycelium.
7. Each cell of the 2- 7. The single-celled, brand
Fruit body celled teleutospores spore germinates to
produces an epibasi- produce a single epiba-
dium which bears four sidium which bears a
G Log of wood basidiospores. They variable number of
Peridiole are borne on sterig- basidiospores. They
Volva mata and are dis- are not borne on sterig-
E F charged violently by the mata nor are they dis-
waterdrop method. charged violently.
Fig. : (A–G) Basidiomycetes. Common types of basidiocarps.
(A) basidiocarp of Agaricus with gills on the underside of
pileus; (B) basidiocarp of Boletus with pores on the underside
Economic Importance
of pileus; (C) basidiocarp of Lycoperdon; (D) basidiocarp of ● Several members of Basidiomycetes are of great
Geaster; (E) fruit bodies of Cyathus; (F) mature fruit body of
economic importance because of their beneficial as
Phallus; (G) fruit bodies of Fomes.
well as harmful nature.

C.S.V. / August / 2009 / 764


● Some of them are causative agents of most destruc- ● Clavatia has medicinal value as it contains anticancer
tive diseases of cereal crops such as ‘smut disease’ substance calvacin.
of oats, wheat, corn, barley as well as the wheat rust. ● Many members of this class form ectomorphic mycor-
● Stem rust of wheat is caused by Puccinia graminis rhizal associations with the roots of forest trees.
tritici. Important Smut Diseases of Cereals
● Some of the higher Basidiomycetes, such as spore Cereals Loose smut Covered smut
fungi, are the common wood rotters; they destroy (Causal organisms) (Causal organisms)
lumbar and timber. Wheat Ustilago nuda (race of Tilletia caries
● Mushrooms which also belong to this group are of U. tritici )
great economic value as food. Oat Ustilago avenae Ustilago horde; ( a
● The young fleshy sporophores of many species of race of U. Kolleri )
‘puff balls’, e.g., Lycoperdon and Clavatia are also Corn Ustilago maydis ——
edible. Barley Ustilago nuda Ustilago hordei

OBJECTIVE QUESTIONS
1. In certain members of Basidio- 7. Conidiospores are formed— 6. During spermatogenesis, there
mycetes, the basidium bears four (A) When nutrients are in short are three phases in which the
spores exogenously, each supply process is completed. The correct
usually at the tip of a minute stalk (B) During sexual reproduction sequence of phases is—
known as— (A) Growth → Multiplication →
(C) By sporangia
(A) Sterigmata Maturation
(D) By sac, club and imperfect
(B) Basidiophore
fungi (B) Multiplication → Growth →
(C) Both (A) and (B)
Maturation
(D) Conidiophore 8. Which of the following group(s)
is/are comprised of club fungi ? (C) Maturation → Multiplication
2. ‘Loose smut’ of corn is caused
by— (A) Mushrooms → Growth
(A) Ustilago maydis (B) Puff balls (D) Multiplication → Maturation
(B) Ustilago avenae (C) Truffles → Growth
(C) Ustilago nuda (D) All of the above 7. Growth phase prepares oogonia
(D) All of the above
9. In which of the following the or spermatogonia for first—
3. Which division of fungi is com- dikaryotic stage is longer- (A) Mitotic division
monly known as club-fungi ? lasting ? (B) Meiotic division
(A) Zygomycetes
(A) Sac fungi (C) Binary fission
(B) Oomycetes
(B) Club fungi (D) Multiple fission
(C) Deuteromycetes
(C) Imperfect fungi
(D) Basidiomycetes 8. The sperm head is covered by a
(D) Zygospore fungi membrane—enclosed structure
4. Clamp connection is characteris-
10. The part of mushroom that is called—
tic of a certain group of—
visible above the ground is— (A) Head (B) Neck
(A) Algae and cyanobacteria
(A) Ascogonium (B) Ascocarp (C) Middle piece (D) Acrosome
(B) Fungi
(C) Zygospore (D) Basidiocarp
(C) Bryophytes 9. A pair of centrioles found in the
(D) None of the above ANSWERS sperm, are located in the region—
5. Puccinia graminis tritici causes— (A) Head (B) Neck
(A) Loose smut of oat (C) Middle piece (D) Tail
(B) Black stem rust of wheat ●●● 10. Which of the following is the
(C) Loose smut of barley (Continued from Page 737 ) genetic part of the sperm ?
(D) Covered smut of oat (A) Head (B) Neck
5. During oogenesis, each haploid
6. Anticancer substance ‘calvacin’ cell produces— (C) Middle piece (D) Tail
is obtained from— (A) Four functional eggs
(A) Club fungi ANSWERS
(B) Two functional eggs
(B) Red algae (C) One functional egg and two
(C) Myxophycean cell polar bodies
(D) None of the above (D) Four functional polar bodies ●●●

C.S.V. / August / 2009 / 765


(C) Both (A) and (B)
(D) None of the above
14. The botanical name of ‘prayer
plant’ is—
(A) Ipomoea
(B) Lantana Camara
1. What happens during photo- 7. The first-formed elements of
(C) Osimum sanctum
respiration when stomates are phloem are called—
closed ? (D) None of the above
(A) Protophloem
(A) The concentration of CO 2 (B) Metaphloem 15. The chloroplast envelope en-
increases in leaves closes a liquid protenaceous
(C) Medullary ray
(B) The concentration of CO 2 substance called—
(D) Both (B) and (C)
decreases in the leaves (A) Protoplasm (B) Cytoplasm
(C) Oxygen, a by-product of 8. Which part of a leaf carries on (C) Granum (D) Stroma
photosynthesis, increases most of the photosynthesis of a
plant ? 16. An agent, such as radiation or a
(D) Both (B) and (C) chemical, that brings about a
(A) Guard cells
2. In which of the following ways mutation is called—
(B) Epidermis
the RNA differs from DNA ? (A) Mutagen (B) Transcription
(C) Epidermal layer
(A) The pentose sugar is ribose, (C) Polysome (D) Intron
(D) Mesophyll
not deoxyribose 17. In Utricularia, leaves are modi-
9. Which of the following is comm- fied into—
(B) The base uracil replaces
only known as ‘thorn apple’ ?
thymine (A) Petiole (B) Bladder
(A) Aegle marmelos
(C) RNA is single stranded (C) Tendril (D) Pitcher
(B) Ananas comosus
(D) All of the above 18. Which of the following scientists
(C) Datura stramonium
proved experimentally that DNA
3. The fibres which are used in the (D) Garcinia mangostana
and not protein is the genetic
manufacture of brooms are material of the T2 bacterio-
10. A change in the amount of
known as— phage ?
energy in the form of heat libe-
(A) Brush fibres rated or absorbed by the system (A) Watson and Crick
(B) Rough weaving fibres during physical or changes is ter- (B) Hershey and Chase
med as—
(C) Soft or bast fibres (C) Jacob and Monod
(A) Enthalpy (D) Miescher and Griffith
(D) None of the above
(B) Entropy
4. The genetic code is a triplet code 19. In shade leaves the ratio of
(C) Ecological energetics chlorophyll to carotenoids appro-
and each codon consists of—
(D) Pyramid of energy aches—
(A) One base
11. Amino acids are converted into (A) 5 : 1 (B) 2 : 3
(B) Two bases
ammonia by a group of bacteria (C) 3 : 2 (D) 1 : 1
(C) Three bases
called— 20. Plasmodesmata are formed
(D) Four bases
(A) Ammonifying bacteria around the elements of—
5. Tendrils of Smilax are homolo- (B) Denitrifying bacteria (A) Golgi bodies
gous to— (B) Chloroplast
(C) Nitrifying bacteria
(A) Axillary buds (D) All of the above (C) Nucleus
(B) Stipules (D) None of the above
12. During the noncyclic electron
(C) Leaf apices pathway, electrons move from— 21. Corymb inflorescence has—
(D) Leaflets (A) Water through PS-II to PS-I (A) Flattened peduncle
6. The internal mechanism by and then to NADP + (B) Long peduncle
which a biological rhythm is (B) Water through PS-I to PS-II (C) Short peduncle
maintained in the absence of and then to NADP + (D) No peduncle
appropriate environmental stimuli (C) PS-I to PS-II
is termed as a— 22. Which of the following is correct
(D) ADP to ATP
regarding aerobic respiration ?
(A) Circardian rhythm
13. The mode of reproduction in (A) It is common in all higher
(B) Sleep movement phycomycetes is— plants
(C) Thigmotropism (A) Asexual (B) Energy is liberated in the
(D) Biological clock (B) Sexual form of ATP

C.S.V. / August / 2009 / 766


(C) End products are H2O and (C) Hatch-Slack cycle 40. How many primary types of his-
CO2 (D) PP-pathway tone molecules are known ?
(D) All of the above (A) One (B) Two
32. An increase in the dry weight of a
23. Cyathium and hypanthodium plant is dependent upon the pre- (C) Four (D) Five
types of inflorescence are similar sence of— 41. Who among the following coined
in having— (A) Oxygen the term cryptovegetarian ?
(A) Unisexual flowers (B) Iron (A) Draghetti
(B) Petaloid bracts (C) Nitrogen (B) Spalanzani
(C) Apical pore (D) Carbon dioxide (C) Lamarck
(D) Nectar glands (D) Darwin
33. RNA retroviruses have a special
24. Which of the following RNA (particular) type of enzyme res- 42. For each of the twenty amino
carries a sequence of codons to ponsible for— acids found in proteins, there
the ribosomes ? (A) Transcribing viral RNA to a should be atleast—
(A) t-RNA (B) m-RNA DNA (A) One t-RNA molecule
(C) r-RNA (D) All of these (B) Polymerizing host DNA (B) Two t-RNA molecules
25. The structure which can show (C) Disintegrating host DNA (C) Three t-RNA molecules
the effect of traits brought by the (D) Translating host DNA (D) Twenty t-RNA molecules
male gamete immediately after
34. Which of the following gives a 43. Cinnamon is obtained from which
its formation is—
possible sequence of organic part of the plant body ?
(A) Plumule (B) Cotyledon chemicals prior to the protocell ?
(C) Endosperm (D) Embryo (A) Bark (B) Root
(A) Inorganic gases, nucleo- (C) Leaf (D) Fruit
26. A microbial metabolite excreted tides, nucleic acids, genes.
or released by lysed cells which (B) Inorganic gases, amino 44. The edible part of tomato is—
in very low concentration is dire- acids, polypeptide, micro- (A) Mesocarp only
ctly toxic to cells of the suscept, sphere (B) Epicarp
is defined as—
(C) Both (A) and (B) (C) Thalamus
(A) Pathogenesis (B) Taxis
(D) Water, salt, protein, oxygen (D) Placenta and pericarp
(C) Toxin (D) Etiology
35. Nitrifying bacteria are kept
27. Guttation is the process of 45. A substance or mixture of subs-
under—
elimination of water from plants tances which prevents, miti-
(A) Vibrio group gates, destroys or repels any
through—
(B) Bacillus group pest is commonly called—
(A) Lenticels (B) Stomata
(C) Coccus group (A) Epinasty
(C) Wounds (D) Hydathodes
(D) None of the above
28. The core of the axis which (B) Growth hormone
includes the vascular system, the 36. Which of the following xylem ele- (C) Antitransparent
interfascicular portion, the pith in ments is living ?
(D) Pesticide
the vicinity of vascular bundles (A) Fibres (B) Vessels
(C) Parenchyma (D) Tracheids 46. Apple and pear are—
(pericycle) is called—
(A) Stele (B) Cortex (A) Schizocarpic and pome fruits
37. A rootless aquatic plant in which
(C) Tracheid (D) Endodermis a portion of leaf is modified to (B) Schizocarpic and pepo fruits
form a bladder for catching small (C) Succulent and pome fruits
29. Which of the following factors is
aquatic animals is— (D) True and schizocarpic fruits
most important in regulating
transpiration ? (A) Utricularia (B) Drosera
47. In which of the following plants,
(C) Nepenthes (D) Dionaea the seed germinates while still
(A) Wind (B) Temperature
(C) Light (D) Humidity 38. Golgi bodies are absent in— attached to the parent (main)
(A) Higher plants plant ?
30. Hira (HD 1941) and Moti (HD
(B) Bacteria (A) Rhizophora
1949) are the varieties of—
(C) Blue green algae (B) Screwpine
(A) Wheat (B) Rice
(D) Both (B) and (C) (C) Mango
(C) Maize (D) Pulse
(D) All of the above
31. Sugarcane, maize and some 39. Herbaceous angiosperms flouri-
other tropical plants have high shed in— 48. Polysome is a group of—
efficiency of CO2 fixation beca- (A) Holocene epoch (A) Nucleolus
use they operate— (B) Pleistocene epoch (B) Endoplasmic reticulum
(A) TCA cycle (C) Pliocene epoch (C) Spherosomes
(B) Calvin cycle (D) Miocene epoch (D) None of the above

C.S.V. / August / 2009 / 767


49. Which of the following plants is
commonly called Swallow wart ?
(A) Calotropis procera
(B) Eucalyptus globulus
(C) Jatropha curcas
(D) Bryophyllum pinnatum
50. An allosteric site on an enzyme
is—
(A) Often involved in feedback
inhibition
(B) Where ATP attaches and
gives up its energy
(C) The same as the active site
(D) All of the above

ANSWERS WITH HINTS

(Continued on Page 790 )

C.S.V. / August / 2009 / 768


(C) Chlamydomonas
(D) Batrachospermum
13. The water held tightly by the soil
particle around them is known
as—
(A) Capillary water
1. Match Column A (Different types 5. The oil is extracted from the (B) Field capacity
of fruits) with Column B (Different seeds of—
(C) Hygroscopic
examples) then select the correct (A) Brassica nigra
answer from the options given (D) None of the above
(B) Brassica campestris
below— 14. Which of the following state-
(C) Both (A) and (B)
Column A Column B ments is not correct regarding
(D) None of the above Cycas ?
(a) Capsule 1. Acer
6. Which of the following is the (A) Cycas plant is arboreal
(b) Lomentum 2. Iris
main internal ground tissue ? (B) It looks like a palm tree
(c) Samara 3. Genarium
(A) Parenchyma (B) Epidermis (C) The stem is columnar nor-
(d) Regma 4. Acacia
(C) Endodermis (D) Pith mally branched with deci-
(a) (b) (c) (d) duous leaf bases
7. The small size of cells is best
(A) 2 4 3 1 correlated with— (D) Both (A) and (C)
(B) 3 2 1 4 (A) The fact that they are self- 15. The sporophyte of Riccia is
(C) 2 4 1 3 reproducing represented by—
(D) 3 4 2 1 (B) An adequate surface area for (A) Spore sac only
exchange of materials (B) Foot, seta and capsule
2. Proteins and polar lipids account
(C) Their prokaryotic versus (C) Foot and capsule
for almost all of the mass of
eukaryotic nature
biological membranes; the small (D) Spores and elaters
amount of carbohydrate present (D) All of the above are correct
16. The process by which DNA of
is generally part of— 8. Which of the following mem- nucleus passes information to
(A) Glycoproteins branes prohibits the entry of both RNA is called—
(B) Glycolipids solvent and solute particles ?
(A) Transcription
(A) Permeable membrane
(C) Both (A) and (B) (B) Translation
(B) Semipermeable membrane
(D) None of the above (C) Transformation
(C) Both (A) and (B)
(D) Transduction
3. Mobilization of stored food in ger- (D) Impermeable membrane
minating seed is triggered by— 17. Chief means of perennation in
9. Which of the following micro- liverworts is—
(A) Gibberellin (B) Auxin organisms charges protein into
(C) Abscisic acid (D) Cytokinin (A) Tuber formation
ammonia ?
(A) Nitrobacter (B) Persistent apices
4. Match Column ‘A’ (Common
(B) Azotobacter (C) Both (A) and (B)
name) with Column ‘B’ (Botani-
cal name) then select the correct (C) Rhizobium (D) None of the above
answer from the options given (D) Bacillus mycoides 18. Dry rot of sugarbeet and internal
below— browning of cauliflower diseases
10. Synthesis of quinine was achie-
Column ‘A’ Column ‘B’ are due to the deficiency of—
ved by—
(a) Palmyra 1. Erianthus (A) Zinc (B) Boron
(A) Alexander Fleming
palm ravennae (C) Copper (D) Manganese
(B) Woodward and Doering
(b) Toddy palm 2. Borassus (C) M.S. Swaminathan 19. Membrane often contains steroid
flabellifer which is a type of lipid that con-
(D) U. Brahmchari
(c) Plum grass 3. Typha tains—
angustata 11. ‘Safed Lerma’ is a new variety
of— (A) No carbon rings
(d) Elephant 4. Caryota (B) Four carbon rings
urens (A) Onion (B) Grape
grass (C) Only one carbon rings
(C) Garlic (D) Wheat
(a) (b) (c) (d) (D) None of the above
12. Which of the following algae is
(A) 4 3 2 1
commonly known as ‘frog- 20. A red pigment found in the fruit
(B) 4 3 1 2 spawn’ ? of tomato is called—
(C) 2 4 1 3 (A) Ulothrix (A) Lycopene
(D) 2 4 3 1 (B) Volvox (B) Xanthophyll

C.S.V. / August / 2009 / 769


(C) Myxoxanthin 29. The cytoplasmic portion of the 39. Which of the following plants
(D) Rhodopsin protoplast is referred to as— does not belong to family Papil-
(A) Endoplasm (B) Cytoplasm ionaceae ?
21. Who coined the term ‘metabolism’
(C) Cytosome (D) Ectoplasm (A) Phaseolus mungo
for all chemical processes carried
on in cells ? (B) Bauhinia variegata
30. Herbicide DCMU kills plants due
(A) Schwann (B) Nawashchin to spoilage of— (C) Crotolaria juncea
(C) Twort (D) Strasburger (A) Photophosphorylation (D) All of the above

22. Inflorescence having a flattened (B) Electron transport 40. Which of the following is a
axis, sessile flowers and a whorl (C) O2-evolution saprophytic angiosperm ?
of involucral bracts is— (D) Rubisco activity (A) Neottia (B) Agaricus
(A) Corymb (B) Head (C) Cuscuta (D) Eucalyptus
31. Trisomy is a type of—
(C) Raceme (D) Umbel (A) Euploidy (B) Polyploidy 41. Autopolyploidy arised by increase
23. For the purpose of gene therapy in the number of chromosome
(C) Hyperploidy (D) Hypoploidy
a retrovirus should be equipped sets of the same species is
32. Balance between CO2 and O2 is called—
with—
maintained by— (A) Intraspecific polyploidy
(A) Recombinant RNA
(A) Photosynthesis (B) Interspecific polyploidy
(B) t RNA
(B) C4-pathway (C) Both (A) and (B)
(C) m RNA
(C) Transpiration (D) Explant
(D) r RNA
(D) Photorespiration 42. Which of the following is a true
24. Non transfer of pollen from hemp ?
33. The potential energy of water is
anther to stigma of the same (A) Cannabis sativa
referred to as—
flower due to mechanical barrier (B) Boehmeria nivea
is— (A) Protoplasmic streaming
(B) Thermodynamics (C) Girardinia heterophylla
(A) Heterostyly (D) Agave sisalana
(C) Water relation
(B) Herkogamy
(D) Water potential 43. Quadrifoliate leaves are found
(C) Dichogamy in—
(D) Cleistogamy 34. From photosystem-I ‘high en-
ergy’ electrons pass to NADP (A) Paris quadrifoliata
25. Regulation of gene activity in where they combine with hydro- (B) Bombax ceiba
prokaryotes usually occurs at the gen ions to form— (C) Hardwickia
level of— (A) NAD (B) ADP (D) All of the above
(A) Translation (C) NADPH2 (D) FAD 44. The meristematic layer between
(B) Transcription bark and the wood in a woody
35. The noncyclic electron pathway
(C) Both (A) and (B) stem is called—
generates—
(D) None of the above (A) Cork cambium
(A) No ATP
26. Genotypically the pollen grains (B) ATP only (B) Zone of cell division
produced by an anther belong (C) NADPH only (C) Vascular cambium
to— (D) None of the above
(D) Both ATP and NADPH
(A) One type
(B) Many types 36. The first person to associate 45. Tribulus fruit is dispersed by
specific gene with a specific means of—
(C) Two types
chromosome is/was— (A) Water (B) Animals
(D) All of the above
(A) Morgan (B) Swaminathan (C) Wind (D) Explosion
27. Cross-pollination within a species (C) Correns (D) Maheswari
is called— 46. In bryophytes, absorbing and
37. The baloonlike outgrowth of attaching organs are—
(A) Explant
parenchyma into the lumen of the
(B) Xenogamy (A) Columella
vessels is known as—
(C) Allopolyploidy (B) Rhizoids
(A) Tunica (B) Tyloses
(D) Autopolyploidy (C) Histogen (D) Phellogen (C) Root hairs
(D) Whole thallus
28. At constant temperature, the rate 38. Drosera catches insects by
of transpiration will be higher means of— 47. The vertical temperature gradient
at— (A) Pitcher over earth’s surface is called—
(A) 1 km below sea level (B) Adhesive disc (A) Phenology
(B) 1 km above sea level (C) Bladder (B) Littoral zone
(C) 2 km above sea level (D) Tentacles secreting shining (C) Sigmoid curve
(D) Sea level liquid (D) Lapse rate

C.S.V. / August / 2009 / 770


48. If concentration of CO2 in the
external atmosphere is increa-
sed, the stomata are closed in—
(A) Dark only
(B) Light only
(C) Both in light and dark
(D) None of the above
49. Leaf primordia are produced by
the—
(A) Vascular bundle
(B) Cambia
(C) Cortex
(D) Apical meristem
50. Carrageenin is used as emulsi-
fying and stabilizing agent in—
(A) Chocolates
(B) Ice creams
(C) Toothpastes
(D) All of the above

ANSWERS WITH HINTS

●●●

C.S.V. / August / 2009 / 771


In each of the following ques- Reason (R) : Light is formed of iodide solution as compared to
tions, a statement of Assertion (A) material particles. pure water.
is given and a corresponding state- (A) (B) (C) (D) (E) Reason (R) : Potassium iodide
ment of Reason (R) is given just 6. Assertion (A) : If there exists solution absorbs iodine forming
below it. Of the statements, mark Coulomb attraction between two compound of potassium in which
the correct answer as— charged bodies, both of them K+ is oxidised to K3 +.
(A) If both A and R are true may not be charged. (A) (B) (C) (D) (E)
and R is the correct expla- Reason (R) : They will be oppo-
nation of A 12. Assertion (A) : At isoelectric
sitely charged.
(B) If both A and R are true but point of an amino acid, it does not
(A) (B) (C) (D) (E)
R is not the correct expla- migrate under the influence of an
nation of A 7. Assertion (A) : White light con- electric field.
(C) If A is true but R is false tains the range of colours in light Reason (R) : At isoelectric point
from violet with a wavelength of an amino acid is totally ionized.
(D) If both A and R are false
4 × 10– 7 m to red light with a (A) (B) (C) (D) (E)
(E) If A is false but R is true
wavelength of 7 × 10– 7 m.
13. Assertion (A) : Mercury metal
PHYSICS Reason (R) : When Young’s starts sticking to the side of the
double slit experiment is carried glass tube on bubbling ozone
1. Assertion (A) : Light is diffracted out with light, multicoloured through it.
around the edges of obstacles fringes are formed.
and the bending is so slight that Reason (R) : In the presence of
(A) (B) (C) (D) (E) ozone, mercury reacts with glass
it is not easily observed.
8. Assertion (A) : To float, a body to form mercury (II) silicide.
Reason (R) : The wavelength of
must displace liquid whose weight (A) (B) (C) (D) (E)
light is very small.
is greater than the actual weight
(A) (B) (C) (D) (E) of the body. 14. Assertion (A) : Methanoic acid
changes mercuric chloride to
2. Assertion (A) : Mass and energy Reason (R) : A floating body will
mercurous chloride on heating
are not conserved separately, but experience no net downward
but acetic acid does not do so
are conserved as a single entity force.
under similar conditions.
called ‘mass energy’. (A) (B) (C) (D) (E)
Reason (R) : Methanoic acid is a
Reason (R) : This is because one 9. Assertion (A) : Dark lines are stronger acid than ethanoic acid.
can be obtained at the cost of observed in the spectrum of light
other as per Einstein’s equation (A) (B) (C) (D) (E)
produced from the sun rays.
E = mc 2. Reason (R) : Dark lines are due 15. Assertion (A) : Sometimes a
(A) (B) (C) (D) (E) to absorption of certain radiations yellow turbidity appears while
by gases in the outer atmos- passing H2S gas even in the
3. Assertion (A) : Coherent sources
phere of the sun. absence of radicals of 2nd group.
generate waves of identical fre-
quencies. (A) (B) (C) (D) (E) Reason (R) : Group IV radicals
10. Assertion (A) : Two sources A are precipitated as their sul-
Reason (R) : Two coherent phides.
sources of circular wave patterns and B each carrying a sound of
400 Hz are standing a few metres (A) (B) (C) (D) (E)
can be produced by passing a
plane wave through narrow slits. apart. When A moves towards B, 16. Assertion (A) : In the electrolysis,
both persons hear the same the quantity of electricity needed
(A) (B) (C) (D) (E)
number of beats per second. to deposit one mole atom of silver
4. Assertion (A) : The force experi- Reason (R) : Doppler shift in is different from that required for
enced by a charged particle frequency of sound is the same depositing one mole atom of
moving in a magnetic field can whether the observer approaches copper.
not do any work. the source or the source Reason (R) : Atomic weights of
Reason (R) : The force does not approaches the observer with silver and copper are different.
displace the charged particle. same speed.
(A) (B) (C) (D) (E)
(A) (B) (C) (D) (E)
(A) (B) (C) (D) (E)
17. Assertion (A) : The gelatinous
5. Assertion (A) : With a white light CHEMISTRY white precipitate of aluminium
source the central fringe is dark hydroxide dissolves both in HCl
in Young’s double slit experi- 11. Assertion (A) : Iodine dissolves as well as in concentrated NaOH
ment. more appreciably in potassium solution.

C.S.V. / August / 2009 / 772


Reason (R) : Aluminium hydro- 24. Assertion (A) : Coenzyme is a BOTANY
xide is a strongly ionic as well as non-protein group without which
covalent in nature. certain enzymes are incomplete 31. Assertion (A) : In Mucor mucedo
(A) (B) (C) (D) (E) or inactive. all of the sporangiophores obtai-
Reason (R) : Coenzymes not only ned from a single germ sporan-
18. Assertion (A) : Nitrogen obtained gium give rise to mycelia which
provide a point of attachment for
by fractional distillation of liquid are of the same mating type. A
the chemical groups being trans-
air is heavier than the nitrogen zygospore germination of this
formed but also influence the pro-
obtained by decomposing nitro- type is known as pure homothal-
perties of the group.
genous compounds. lic.
(A) (B) (C) (D) (E)
Reason (R) : Nitrogen obtained Reason (R) : Sexual repro-
from fractional distillation of liquid 25. Assertion (A) : It is the brain, not
duction in a heterothalic species
air is not completely pure. the sense organs, that interprets
of Mucor is not effected by copu-
(A) (B) (C) (D) (E) the stimulus.
lation of two multinucleate iso-
Reason (R) : Sense organs are gametangia.
19. Assertion (A) : The methyl
transducers, they transform the
carbanion is pyramidal in shape (A) (B) (C) (D) (E)
energy of stimulus to the energy
like the structure of ammonia
of nerve impulses. 32. Assertion (A) : Hershey and
molecule.
(A) (B) (C) (D) (E) Chase turned to blue-green algae
Reason (R) : The carbon atom of
as their experimental material.
methyl carbanion is sp 2 hybridi- 26. Assertion (A) : Jelly fish is not a
sed. Reason (R) : Franklin prepared
true fish which is a vertebrate
an X-ray photograph of DNA that
(A) (B) (C) (D) (E) animal with backbone.
showed its certain dimensions.
20. Assertion (A) : Most of the endo- Reason (R) : The name jelly fish
(A) (B) (C) (D) (E)
thermic reactions are not sponta- is given to invertebrate coelente-
neous under ordinary conditions rate animal, Aurelia, because it is 33. Assertion (A) : The sterile por-
but become so at elevated tem- made up of jelly-like substances. tion of the sporangium in Mucor
perature. (A) (B) (C) (D) (E) is called pileus.
Reason (R) : Entropy of the 27. Assertion (A) : Girls have all Reason (R) : In Mucor, the
system increases with increase in their primary oocytes at birth, remainder of the progametan-
temperature. oogenesis begins at puberty. gium is known as the suspen-
(A) (B) (C) (D) (E) Reason (R) : The first menstrual sor.
cycle is called menopause, and (A) (B) (C) (D) (E)
ZOOLOGY the end of menstrual cycles is 34. Assertion (A) : Archegonium is
called menarche. the female sex organ of liverworts,
21. Assertion (A) : Dramatic popula- (A) (B) (C) (D) (E) ferns and most gymnosperms.
tion growth has occurred in the Reason (R) : Archegonium is
last 300 years, with more growth 28. Assertion (A) : Morphogenesis
involves change in the shape of multicellular of which the swollen
in less developed countries base, i.e., venter contains egg-
(LDCs) than in more developed the embryo.
Reason (R) : Differentiation is the cell.
countries (MDCs). (A) (B) (C) (D) (E)
specialization of cell structure and
Reason (R) : Total fertility rate
function as some genes are 35. Assertion (A) : Transposable
(TFR) is lower in MDCs, due to
turned on and others off. elements can block gene expres-
family planning and access to
(A) (B) (C) (D) (E) sion by inserting into a gene.
contraceptives. Infant mortality is
lower in MDCs than LDCs. 29. Assertion (A) : The sympathetic Reason (R) : Transposable ele-
division acts to mobilize the body ments can move and insert into
(A) (B) (C) (D) (E)
for emergency or quick action— genes, disrupting their expres-
22. Assertion (A) : Capsule of tendon sion.
the ‘fight-or-flight’ response.
is associated with brain. (A) (B) (C) (D) (E)
Reason (R) : It decreases the
Reason (R) : Inherited Rh factor heart rate, respiration rate and 36. Assertion (A) : Daily periodicity
gene is found in Rh – individuals. digestive activity; dilates blood of stomatal movement is not a
(A) (B) (C) (D) (E) vessels of the skeletal muscles, characteristic of succulent plant.
23. Assertion (A) : Baroreceptor is and constricts those of the skin. Reason (R) : In succulent plants
the receptor for hydrostatic pres- (A) (B) (C) (D) (E) stomata are closed at night and
sure of blood. 30. Assertion (A) : Neurosecretion is opened during the day.
Reason (R) : Receptors of atrium pivotal in the vertebrate endocrine (A) (B) (C) (D) (E)
stimulate the Cardio-acceleratory system. 37. Assertion (A) : Scientists use
centre, helping to regulate blood Reason (R) : Hormones are pro- several approaches when they
pressure. duced by endocrine glands. design experiments in order to
(A) (B) (C) (D) (E) (A) (B) (C) (D) (E) make data as valid as possible.

C.S.V. / August / 2009 / 773


Reason (R) : Researchers might most lichens to atmospheric roplast and does not directly
examine hundreds of cells, pollution. require solar energy.
because a large sample size (A) (B) (C) (D) (E) (A) (B) (C) (D) (E)
helps ensure meaningful results. 40. Assertion (A) : Cellular respira-
39. Assertion (A) : The energy cap-
(A) (B) (C) (D) (E) turing portion of photosynthesis tion includes glycolysis and fer-
38. Assertion (A) : Lecanora is a takes place in thylakoid mem- mentation.
genus of lichens in which the branes and can not proceed Reason (R) : Both glycolysis and
thallus is crustose. without solar energy. fermentation are only aerobic
Reason (R) : Some species of Reason (R) : The synthesis part respiration.
Lecanora are more resistant than of photosynthesis occurs in chlo- (A) (B) (C) (D) (E)

ANSWERS WITH HINTS

(Continued on Page 790 )

C.S.V. / August / 2009 / 774


Physics Chemistry
1. A convex lens is made of two different materials. A 16. The structural-pair geometry of xenon difluoride
point is placed on the principal axis. The number of (XeF2) is linear.
images formed by the lens will be two. —T/F
—T/F 17. The heat of hydrogenation of trans-2-butene is more
than that of cis-2-butene.
2. No net force acts on a rectangular coil carrying a
—T/F
steady current when suspended freely in a uniform
18. A balloon is inflated with He to a volume of 4·5 L at
magnetic field.
room temperature. If balloon is taken to the place
—T/F having temperature of – 10°C, the volume of the
3. In an electric field, the electron moves away from balloon remains unchanged.
higher potential to the lower potential. —T/F
—T/F 19. The electronic configuration of fluorine atom is
4. Birds fly off a high tension wire when the current is 1s 2, 2s 2 2px 2 2py 2 2pz 1, and according to Hund’s rule
switched on. the last electron enters to 2 pz orbital with clockwise
—T/F spin.
—T/F
5. Fuse wire must have high resistance and low melting
20. Phenol is a weaker acid than carbonic acid.
point.
—T/F
—T/F 21. A redox reaction in which reactants containing same
6. In a perfectly elastic collision, the kinetic energy may element in different oxidation states react to give a
not remain constant. species in which the element is in same oxidation
—T/F state is called a comproportionation reaction.
7. The sensitivity of a moving coil galvanometer can be —T/F
increased by using a suspension wire of shorter 22. The oxidation of sucrose with conc. HNO3 in
length. presence of V2O5 leads to the formation of 1, 6-
—T/F hexanedioic acid.
8. When a monocycle turns, along the curve, remaining —T/F
vertical at a constant speed, the angular velocity of 23. When one mole electron is passed separately through
the wheel does not change. each of solutions of CuSO4 and AgNO3, the mole ratio
—T/F of Cu and Ag deposited will be 1 : 1.
9. Cathode rays constitute a stream of negatively —T/F
charged particles, called electrons. 24. Colour is shown by a transition metal ion which
—T/F contains all ( n – 1) d vacant orbitals.
10. A piece of red glass is heated till it glows in dark. The —T/F
colour of the glowing piece will be somewhat orange. 25. The vapour density of a substance has no unit and is
independent of temperature and pressure.
—T/F
—T/F
11. A body, whatever its motion, is always at rest in a 26. Alkaline KMnO4 solution is more readily decolourised
frame of reference which is fixed to the body itself.
by benzene than by acetylene.
—T/F —T/F
12. An electron revolves round a nucleus of charges Ze. 27. A gas has vapour density of 17, 10 g of this gas at
In order to excite the electron from the state n = 2 to 17°C and 1 atm will occupy a volume of 7 mL.
n = 3, the energy required is 47·2 eV. Z is equal to 5.
—T/F
—T/F 28. Gatta percha and natural rubber are actually,
13. On reducing the volume of a gas at constant tem- stereoisomers.
perature, the pressure of the gas increases. —T/F
—T/F 29. SnCl 2 is a crystalline solid while SnCl4 is a volatile
14. X-rays cannot be diffracted by means of grating. liquid.
—T/F —T/F
15. Thermal conductivity of air being less than that of felt, 30. Methyl cyanide is soluble in water while methyl
even then we prefer felt to air for thermal insulation. isocyanide is insoluble.
—T/F —T/F

C.S.V. / August / 2009 / 775


Zoology Botany
31. IgE is not fastened by its C-terminal end. 46. Mitochondria supply most of the necessary biological
—T/F energy by breaking down of protein, sugar and
32. The rate of oxygen consumption of an organism or carbohydrates.
tissue is called oxygen quotient. —T/F
—T/F 47. The highest point in the reaction coordinate diagram
33. Osteocytes maintain bone, osteoblasts form new bone represents the idiogram.
and osteoclasts attack and destroy bones. —T/F
—T/F 48. Genes which confer antibiotic resistance on bacteria
34. Pentose phosphate pathway is alternative to are located on plasmid.
glycolysis. —T/F
—T/F 49. Some plants of dry and arid habitats shed their leaves
35. Twelve pairs of cranial nerves and thirty one pairs of to reduce water loss.
spinal nerves comprise the somatic peripheral nerves —T/F
in human nervous system.
50. Colour-blindness is not sex-linked inheritance.
—T/F
—T/F
36. Some developmental structures or processes such as
gill pouches in mammalian embryos are regarded as 51. The edible part of Oryza sativa is epicarp and
phyletic. thalamus.
—T/F —T/F
37. The circulatory system of cockroach, Periplaneta 52. Ergotism is caused by Claviceps purpurea.
americana, is of open type. —T/F
—T/F 53. Plant tissue culture cannot produce haploid plants.
38. The occurrence of different morphological stages —T/F
during the life of an organism is called pleiomorphism.
54. Xylem transports water from the root to the leaf.
—T/F
—T/F
39. Buffers limit pH changes and protect living cells from
injury by strong acids and bases. 55. Stomata arise from the protoderm cells.
—T/F —T/F
40. Prosimian is a group of primates that include apes 56. Stomata in angiosperms open and close owing to
and humans. change in turgor pressure in guard cells.
—T/F —T/F
41. Restriction enzymes are not used to cleave the plas- 57. The pine stem is characterized by the presence of
mid DNA for genetic engineering. conspicuous resin ducts, which are distributed
—T/F throughout the stem.
42. Planarians are free-living turbellarians. —T/F
—T/F 58. The hormone cytokinin is used for early ripening of
43. Epitope is an antibody determinant. fruits.
—T/F —T/F
44. Cyclic AMP is a second messenger within cells. 59. Permeable membranes are those which allow diffusion
—T/F of both solvent and solute molecules through them.
45. Organic evolution is a change in the frequency of —T/F
alleles in a population. 60. Mustard shows hypogeal germination.
—T/F —T/F

ANSWERS WITH HINTS

C.S.V. / August / 2009 / 776


Physics
Q. Can a body have zero velo-
city and constant acceleration ?

Q. What is a solenoid ?
Comment on the magnetic field
around a solenoid ?
Q. What is meant by band- ☞
width ?

Q. What is the radius of gyra-


tion ?
Q. A hydrogen atom contains
one electron. But the spectrum of ☞
hydrogen atom has many lines.
Why ?
Q. What is the difference bet- ☞
ween the interference and diffrac-
tion of light ?

Q. What is axial or pseudo


vector ?

Q. A transistor is a tempera-
ture sensitive device. Why ?

Q. What is Lami’s theorem ?


C.S.V. / August / 2009 / 778


Q. What is law of equipartition
of energy ?

Q. What are gemstones ?


Q. Hydrogen peroxide finds


different uses in different concen-
trations.

Q. What is total internal reflec-


tion ?

Q. What are mustard oils ?

Q. How calcium cyanamide-


carbon mixture acts as a fertilizer ?

Q. What unbreakable plastic-


crockery is made of ?

Q. What is the difference


between mass and weight of an
object ?

Chemistry
Q. How the process of osmo-
sis helps developing adema and
preserving meat and fruits ?

C.S.V. / August / 2009 / 779


Q. Why the product of fre- Zoology
quency and wavelength of infrared
and ultraviolet radiations is always Q. What is perception of
same ? pitch ?
☞ ☞

Q. What are good and bad


cholesterol ?

Q. What is the mass of a


single atom of 12C in grams and
the number of grams per amu ?

Q. What will be the value of


ideal gas constant, R, if exactly
Q. What is teleology ?
1 mol of an ideal gas occupies a

volume of 22·414 litre at 0°° C and
1 atom pressure ?

Q. How do the digestive tracts


of carnivores differ from those of
hervivores ?

C.S.V. / August / 2009 / 780


Q. What is parsnip ? What are
its importance ?

Botany
●●●
Q. What do you mean by pure Q. How are monosaccharides
culture ? formed from polysaccharides ? (Continued from Page 777 )
☞ ☞

Q. Why does coconut oil freeze


during winter ?

Q. Why is CO2 commonly
known as green house gas ?

Q. What do you mean by


Enterobacter ?

Q. What is forest engineering
and its significance ?

●●●

C.S.V. / August / 2009 / 781


1. A police van moving on a high (C) LH = LLi and | EH | > | ELi | 13. Which of the following is/are
way with a speed 30 km h–1 fires (D) LH < LLi and | EH | < | ELi | correct about interclavicular of
a bullet at a thief’s car speeding pigeon ?
6. Which of the following is not
away in the same direction with a (A) It is a median
correctly matched ?
speed of 192 km h–1. If the muzzle (B) It is unpaired
(A) Hexadecane—C6H14
speed of the bullet is 150 ms–1, (C) Connected to the secondary
with what speed does the bullet (B) Heptadecane—C17H36 bronchi of both lungs
hit the thief’s car ? (C) Eicosane—C 20H42 (D) All of the above
(A) 100 ms –1 (B) 105 ms –1 (D) Docosane—C22H46
14. How many nucleotide pairs are
(C) 95 ms –1 (D) None of these 7. Alkenes may be hydrated to reported in β-globin gene of
2. A 4 kg block A is placed on the alcohols by absorption in— mouse by Leder et al ?
top of 8 kg block B which rests (A) Dilute sulphuric acid (A) 116 (B) 216
on a smooth table. A just slips on (B) Dilute hydrochloric acid (C) 96 (D) 106
B when a force of 12N is applied (C) Concentrated sulphuric acid 15. The induction of regional
on A. What is maximum horizon- (D) Concentrated hydrochloric anesthesia by preventing sensory
tal force F required to make both acid nerve impulse from reaching
A and B move together ? centres of consciousness, is
8. ‘Cement fondu’ is a/an—
(A) Hydraulic cement composed known as—
chiefly of calcium aluminate (A) Nerve back
(B) Hydraulic cement composed (B) Nerve entrapment syndrome
chiefly of sodium silicate (C) Nerve growth factor
(C) Portland cement (D) Nephrydrosis
(D) Cement made from granu- 16. Soil is chiefly composed of—
lated blast furnace slag of (A) Only water and organic
(A) 16N (B) 24N fairly low alumina matter
(C) 36N (D) 72N (B) Air + water + organic matter
9. Which of the following is/are said
+ minerals
3. Point P, Q and R are in a vertical to be diamagnetic species ?
(C) Only organic colloids
line such that PQ = QR. A ball at (A) Carbanion
(D) None of the above
P is allowed to fall freely. Find the (B) Carbocation
ratio of time of descent through (C) Singlet carbene 17. In balausta type of fruit, the edible
PQ and QR— part is—
(D) All of the above
1 1 (A) Aril
(A) (B) 10. Phenols are more acidic than (B) Fleshy thalamus
2–1 2–1
alcohols because— (C) Succulent testa
2–1 3 (A) Alcohols do not lose H + ions (D) Succulent sepal
(C) (D)
1 2
(B) Phenoxide ion is stabilized 18. From which one of the following
4. An ideal gas confined to an by resonance flowers is the insecticide
insulated chamber is allowed to (C) Phenoxide ion does not ‘Pyrethrum’ derived ?
enter into an evacuated insulated exhibit resonance (A) Rosa
chamber. If Q, W and ΔE int have (B) Nelumbo
(D) Phenols are more soluble in
the usual meanings, then— polar solvents (C) Iberis
(A) Q = 0, W ≠ 0 (D) Chrysanthemum
(B) W = 0, Q ≠ 0 11. Medrysone is a/an—
(A) Newly discovered mutagen 19. The mutagen proflavin is a/an—
(C) ΔEint = 0, Q ≠ 0
from various insects (A) Base analog
(D) Q = W = ΔFint = 0 (B) Hydroxylating agent
(B) Adrenal corticosteroid drug
5. A hydrogen atom and a doubly (C) Pill for contraceptive treat- (C) Acridine dye
ionised lithium atom are both in ment (D) Alkalylating agent
the second excited state. If LH
(D) Endonuclease enzyme 20. Bud scales are found in—
and LLi respectively represent
(A) Jack fruit
their electronic angular momenta 12. In which of the following orga-
and EH and ELi their energies, nisms, the head is prolonged into (B) Ficus
then— a tubular rostrum ? (C) Magnolia
(A) LH > LLi and | EH | > | ELi | (A) Sea horse (B) Varanus (D) All of the above
(B) LH = LLi and | EH | < | ELi | (C) Heloderma (D) Ophisaurus ●●●

C.S.V. / August / 2009 / 782


Rules for taking part in Quiz Contest
of Competition Science Vision

1. All students or those appearing in competitive


examinations can take part in this contest.
2. Candidates taking part in quiz contest will necessarily
have to send their entries by a fixed date. Entries are
to be sent by ordinary post. Please mark your
envelope 'Quiz–Competition Science Vision' on
the top left hand side.
3. Answers given only on the form of the magazine will Solution to Quiz No. 135
be admissible.
4. In the form there are four squares against each Competition Science Vision
question number. Contestants should put a cross (×)
in the square for the answer they think is correct. Last date for sending 28th August, 2009
Giving more than one answer to a question will
disqualify it. Name Mr./Miss/Mrs. ...........................….........................
5. Contestants should essentially write the number of
questions they have solved. Full Address ...................................…..........................
6. Marks will be deducted for wrong answers. .............................................................…………………
7. The candidate sending the maximum number of ............................................................…………………
correct answers will be given Rs. 600 as first prize.
Next two candidates after that will get Rs. 400 and State .......................Pin Code No.
Rs. 300 as second and third prize respectively. If
there are more than one candidate eligible for a
prize, the amount will be equally distributed among Age.................. Academic Qualification........................
them.
Competition examination for which preparing
8. The decision of the editor will be final and binding in
all cases, and will not be a matter for consideration ....................................................................................
of any court. I have read and understood the rules of quiz con-
test of Competition Science Vision issued by Pratiyogita
Darpan and agree to abide by them.
...................................
(Signature)

RESULT
No. of questions attempted..........................................
No. of correct answers.................................................
No. of wrong answers...................................................
Marks obtained.............................................................

ANSWER FORM
Q. No. A B C D Q. No. A B C D
1. 11.
2. 12.
3. 13.
4. 14.
5. 15.
6. 16.
7. 17.
8. 18.
9. 19.
10. 20.

C.S.V. / August / 2009 / 783


According to the rules of the CSV
Quiz, all entry forms were examined.
As a result, the following participants
have qualified for various prizes. CSV
sends them greetings and good
wishes for their bright future. It also
places on record its appreciation for
their inquisitive nature and expresses
obligation for their co-operation.
PRIZE WINNERS
First Prize
Ravi Jaiswal C/o Gaurav Jaiswal
Room No. 88, A. N. Jha Hostel,
University of Allahabad, Allahabad
U.P.–211 002
Second Prize
Vishal Tiwari
III-B-231
Vidyut Vihar Colony
Shaktinagar, Sonebhadra
U.P.–231 222
Third Prize
1. Gagandeep Singh
C/o Dayaram Verma
L-971, Shastri Nagar, Meerut
U.P.–250 004
2. Sarvdaman Sharma
Lane No. 23
Greater Kailash, Jammu
J & K–180 011
Note : Amount of third prize has been
distributed among two third prize
winners.

C.S.V. / August / 2009 / 785


1. For which one of the following 8. Which one of the following state- 13. Other than India and China
countries, is Spanish not an ments is correct ? which of the following groups of
official language ? (A) Liquid Sodium is employed countries border Myanmar ?
(A) Chile as a coolant in nuclear (A) Bangladesh, Thailand and
(B) Colombia reactors Vietnam
(C) Republic of Congo (B) Calcium Carbonate is an (B) Cambodia, Laos and
(D) Cuba ingredient of toothpaste Malaysia
(C) Bordeaux mixture consists (C) Thailand, Vietnam and
2. For which one of the following, is of Sodium Sulphate and lime Malaysia
Tirupur well known as a huge
(D) Zinc amalgams are used as (D) Thailand, Laos and
exporter to many parts of the
a dental filling Bangladesh
world ?
(A) Gems and Jewellery 9. Diffusion of light in the atmos- 14. Through which one of the
(B) Leather goods phere takes place due to— following groups of countries
(A) Carbon dioxide does the Equator pass ?
(C) Knitted garments
(B) Methane (A) Brazil, Zambia and Malaysia
(D) Handicrafts
(C) Helium (B) Colombia, Kenya and
3. The great Asian river Mekong Indonesia
does not pass through— (D) Water Vapours and Dust
Particle (C) Brazil, Sudan and Malaysia
(A) China (B) Malaysia (D) Venezuela, Ethiopia and
(C) Cambodia (D) Loas 10. In which country is the com- Indonesia
4. Which one of the following does mittee which selects winners for
Nobel Peace Prize located ? 15. Which one of the following pairs
not border Panama ?
is not correctly matched ?
(A) Costa Rica (A) Norway (B) Sweden
(B) Pacific Ocean (C) Finland (D) Denmark Railway Zone Headquarters
(C) Colombia 11. Consider the following state- (A) North Gorakhpur
(D) Venezuela ments— Eastern
Railway
5. The waterfall Victoria is asso- 1. Caffeine a constituent of tea
ciated with the river— and coffee is a diuretic. (B) South Bhubaneshwar
(A) Amazon (B) Missouri 2. Citric acid is used in soft Eastern
(C) St. Laurence (D) Zambezi drinks. Railway

6. Israel has common borders 3. Ascorbic acid is essential for (C) Eastern Kolkata
with— the formation of bones and Railway
(A) Lebanon, Syria, Jordan and teeth. (D) South East Bilaspur
Egypt 4. Citric acid is a good substi- Central
(B) Lebanon, Syria, Turkey and tution for ascorbic acid in our Railway
Jordan nutrition
16. Huangpu River flows through
(C) Cyprus, Turkey, Jordan and Which of the statements given which one of the following cities ?
Egypt above are correct ?
(A) Beijing
(D) Turkey, Syria Iraq and (A) 1 and 2 only (B) Ho Chi Minh City
Yemen (B) 1, 2 and 3 only (C) Shanghai
7. What is the correct sequence of (C) 3 and 4 only (D) Manila
the rivers Godavari, Mahanadi, (D) 1, 2, 3 and 4 only
Narmada and Tapti in the des- 17. Consider the following state-
cending order of their lengths ? 12. Which one of the following ments—
(A) Godavari – Mahanadi – subjects is under the Union list in 1. Dengue is a protozoan
Narmada – Tapti the Seventh Schedule of the disease transmitted by mos-
(B) Godavari – Narmada – Constitution of India ? quitoes
Mahanadi – Tapti (A) Regulation of labour and 2. Retro-orbital pain is not a
(C) Narmada – Godavari – Tapti safety in mines and oil fields symptom of Dengue.
– Mahanadi (B) Agriculture 3. Skin rash and bleeding from
(D) Narmada – Tapti – Godavari (C) Fisheries nose and gums are some of
– Mahanadi (D) Public Health symptoms of Dengue.

C.S.V. / August / 2009 / 786


Which of the statements given 23. Which one of the following types 30. What is the Universal Product
above is/are correct ? is used by computerised tomo- Code (UPC) adopted for ?
(A) 1 and 2 (B) 3 only graphy employed for visualisa- (A) Fire safety code in buildings
(C) 2 only (D) 1 and 3 tion of the internal structure of (B) Earthquake resistant build-
human body ? ing code
18. Which one of the following pairs (A) X-rays (C) Bar code
is not correctly matched ? (B) Sound waves (D) Against adulteration in
(A) Southern Air Command (C) Magnetic resonance eatables
—Thiruvananthapuram (D) Radioisotopes
(B) Eastern Naval Command 24. Production of which one of the
—Vishakhapatnam following is a function of the
(C) Armoured Corps Centre and liver ?
School —Jabalpur (A) Lipase
(D) Army Medical Corps Centre (B) Urea
and School —Lucknow (C) Mucus
19. Itaipu Dam built on the river (D) Hydrochloric acid
Parana is one of the largest dams
25. Which one of the following is not
in the world. Which of the follow-
a digestive enzyme in the human
ing two countries have this as a
system ?
joint project ?
(A) Trypsin (B) Gastrin
(A) Brazil and Peru
(C) Ptyalin (D) Pepsin
(B) Paraguay and Equador
(C) Brazil and Paraguay 26. Which one of the following is
(D) Colombia and Paraguay printed on a commonly used
flourescent tubelight ?
20. Consider the following state- (A) 220 K (B) 273 K
ments— (C) 6500 K (D) 9000 K
1. There are 25 High Courts in
27. What does the 104th Constitution
India. Amendment Bill relate to ?
2. Punjab, Haryana and the (A) Abolition of Legislative
Union Territory of Chandi- Councils in certain states
garh have a common High (B) Introduction of dual citizen-
Court. ship for persons of Indian
3. National Capital Territory of origin living outside India
Delhi has a High Court of its (C) Providing quota to socially
own. and educationally backward
classes in private educational
Which of the statements given institutions
above is/are correct ? (D) Providing quota for religious
(A) 2 and 3 (B) 1 and 2 minorities in the services
(C) 1, 2 and 3 (D) 3 only under the Central Govern-
ment
21. Which one of the following major
28. From North towards South, which
Indian cities is most eastward one of the following is the correct
located ? sequence of the given rivers in
(A) Hyderabad India ?
(B) Bhopal (A) Shyok—Spiti—Zaskar—
Satluj
(C) Lucknow
(B) Shyok—Zaskar—Spiti—
(D) Bengaluru (Bangalore) Satluj
22. Out of the four southern states : (C) Zaskar—Shyok—Satluj—
Andhra Pradesh, Karnataka, Spiti
Kerala and Tamil Nadu which (D) Zaskar—Satluj—Shyok—
one shares boundaries with the Spiti
maximum number of Indian 29. In which state is the Rajiv Gandhi
states ? National Institute of Youth
(A) Andhra Pradesh only Development located ?
(B) Karnataka (A) Tamil Nadu
(C) Andhra Pradesh and (B) Karnataka
Karnataka both (C) Himachal Pradesh
(D) Tamil Nadu and Kerala both (D) Uttarakhand

C.S.V. / August / 2009 / 787


Directions—(Q. 1–7) In each 6. MRTPFW 11. (A) Many pilgrims used Govt.
question below is given a group of (A) 4©35#7 (B) 7©35#4 transport to travel to the holy
letters followed by four combinations (C) 7©35#7 (D) 4©35#4 shrine.
of digits/symbols lettered (A), (B), (C) (E) None of these (B) The cost of travel by private
and (D). You have to find out which of transport is very high.
the combinations correctly represents 7. ENTHWR
the group of letters based on the (A) 29317© (B) 293172 12. (A) There has been heavy rains
coding system and mark the letter of (C) ©9317© (D) ★9317★ in the catchment area of the
that combination as your answer. If (E) None of these lakes supplying drinking
none of the combinations correctly water to the city.
Directions—(Q. 8–12) Below in
represents the group of letters, mark (B) The municipal authority has
each question are given two state-
(E) i.e. ‘None of these’ as your suspended the proposed cut
ments (A) and (B). These statements
answer. in water supply to the city.
may be either independent causes or
Letter : may be effects of independent causes 13. Among P, Q, R, S and T, each
MATWREKIHFUBNP or a common cause. One of these having different weight, R is
Digit/Symbol Code : statements may be the effect of the heavier than only P. S is lighter
other statement. Read both the state- than Q and heavier than T. Who
4@ 3 7 © 2 8%1 # $ 6 9 5
ments and decide which of the follow- among them is the heaviest ?
Conditions :
ing answer choice correctly depicts (A) Q
(i) If both the first and the last letters the relationship between these two (B) P
are consonants, both are to be statements. Give answer—
coded as the code for the first (C) S
(A) if statement (A) is the cause (D) Data inadequate
letter.
and statement (B) is its (E) None of these
(ii) If both the first and the last letters effect.
are vowels, both are to be coded (B) if statement (B) is the cause 14. What should come next in the
as the code for the last letter. and statement (A) is its following number series ?
(iii) If the first letter is a vowel and effect. 87 65432 1 765 4 3 2 1 6 5 4 3 2 1
the last letter is a consonant, the (C) if both the statements (A) (A) 6 (B) 4
codes are to be interchanged. and (B) are independent (C) 7 (D) 5
(iv) If the first letter is a consonant causes. (E) None of these
and the last letter is a vowel, (D) if both the statements (A) 15. In a certain code language, ‘come
both are to be coded as ★. and (B) are effects of inde- now’ is written as ‘ha na’; ‘now
1. AEBRMH pendent causes. and then’ is written as ‘pa da na’
(E) if both the statements (A) and ‘go then’ is written as ‘sa pa’.
(A) @26©41 (B) 126©4@
and (B) are effects of some How is ‘and’ written in that code
(C) 126©41 (D) @26©4@ common cause. language ?
(E) None of these
8. (A) The prices of vegetables (A) sa
2. HBEAFU have increased substantially (B) pa
(A) 162@#$ (B) ★62@#★ during past few weeks.
(C) na
(C) 162@#1 (D) $62@#1 (B) Consumer price index at the
(D) Cannot be determined
(E) None of these end of previous week was
increased by 2 per cent. (E) None of these
3. BKNIRT
(A) 689%©6 (B) 689%©3 9. (A) Many employees of the 16. Each consonant in the word
(C) 698%©3 (D) 389%©3 company proceeded on a DISTEMPER is replaced by the
(E) None of these day’s leave on Friday. next letter in the English alphabet
(B) Both Thursday and Saturday and each vowel in the word is
4. KFUBNA replaced by the previous letter in
were declared holiday by
(A) ★#$69★ (B) 8#$69@ the English alphabet, which of
the company.
(C) 8#$698 (D) @#$69@ the following will be the fourth
10. (A) Many anti-social elements
(E) None of these letter from the right end after the
are caught by the police from
5. IRFAME replacement ?
the locality.
(A) 2©#@4% (B) 2©#@42 (B) Many people in the locality (A) T (B) M
(C) %©#@4% (D) ★©#@4★ are detained by the police for (C) S (D) P
(E) None of these questioning. (E) None of these

C.S.V. / August / 2009 / 788


17. How many meaningful English 23. Which of the following means ‘D 29. Who is to the immediate left of
words can be made with the is maternal uncle of K’ ? M?
letters NDOE using each letter (A) D ÷ N – K (B) D ÷ N + K (A) K (B) W
only once in each word ? (C) D × N – K (D) D × N + K (C) D (D) T
(A) None (B) One (E) None of these (E) None of these
(C) Two (D) Three 24. Which of the following means ‘M 30. In which of the following pairs is
(E) More than three is nephew of R’ ? the second person sitting to the
immediate right of the first
18. Four of the following five are alike (A) M × T + R person ?
in a certain way and so form a (B) R÷J+M×T (A) DT (B) TP
group. Which is the one that (C) R÷J+M (C) PR (D) KW
does not belong to that group ? (E) None of these
(D) R×J+M
(A) BD (B) NQ 31. Who is second to the right of T ?
(E) None of these
(C) RP (D) MK (A) D
(E) FH Directions—(Q. 25–27) Following
(B) K
questions are based on the five three-
19. In a certain code GEAR is written (C) M
digit numbers given below—
as ‘5934’ and RIPE is written as (D) Data inadequate
519 378 436 624 893 (E) None of these
‘4869’. How is PAGE written in
that code ? 25. If the positions of the first and the 32. Who is to the immediate left of
(A) 6359 (B) 6539 third digits within each number R?
(C) 4359 (D) 6459 are interchanged, which of the
(A) W (B) P
following will be the second
(E) None of these (C) K (D) T
smallest number ?
20. How many such digits are there (E) None of these
(A) 519 (B) 378
in the number 64382179 each of (C) 436 (D) 624 Directions—(Q. 33–35) Study
which is as far away from the the following arrangement carefully
(E) 893
beginning of the number as when and answer the questions given
the digits are arranged in ascend- 26. If ‘1’ is subtracted from the first below —
ing order within the number ? digit in each number and ‘1’ is R%E5D2#9AFB@J3IK
(A) None (B) One added to the second digit in each M41WU8V©N★GZδ67
number, which of the following 33. How many such symbols are
(C) Two (D) Three
will be the third digit of the there in the above arrangement,
(E) More than three second highest number ? each of which is immediately
21. Four of the following five are alike (A) 9 (B) 8 followed by a consonant and
in a certain way and so form a (C) 6 (D) 4 immediately preceded by a
group. Which is the one that (E) 3 number ?
does not belong to that group ? (A) None (B) One
27. If the positions of the first and the (C) Two (D) Three
(A) 39 (B) 91 second digits within each number (E) More than three
(C) 78 (D) 52 are interchanged, which of the
34. Which of the following is the fifth
(E) 45 following will be the highest to the left of the sixteenth from
number ? the left end of the above
22. How many such pairs of letters
(A) 519 (B) 378 arrangement ?
are there in the word ORDINAL
each of which has as many letters (C) 436 (D) 624 (A) B (B) U
between them in the word as in (E) 893 (C) W (D) N
the English alphabet ? (E) None of these
Directions—(Q. 28–32) Study
(A) None (B) One the following information carefully and 35. Which of the following is the
answer the questions given below : seventh to the right of the four-
(C) Two (D) Three
M, D, P, K, R, T and W are sitting teenth from the right end ?
(E) More than three (A) B (B) #
around a circle facing at the center. D
Directions—(Q. 23 and 24) Read is second to the right of P who is third (C) N (D) U
the following information carefully and to the right of K. T is third to the right (E) None of these
answer the question which follow— of W who is not an immediate
(i) ‘P × Q’ means ‘P is brother neighbour of D. M is third to the left of
of Q’. R.
(ii) ‘P ÷ Q’ means ‘P is sister of 28. Who is third to the left of D ?
Q’. (A) W
(iii) ‘P – Q’ means ‘P is mother (B) P
of Q’. (C) K
(iv) ‘P + Q’ means ‘P is father of (D) Data inadequate
Q’ (E) None of these

C.S.V. / August / 2009 / 789

Vous aimerez peut-être aussi